Reconciling Bruno's Primitives with Multisense

41 views
Skip to first unread message

Craig Weinberg

unread,
Aug 15, 2012, 5:05:44 AM8/15/12
to everyth...@googlegroups.com
Hi Bruno,

I was thinking about your primitive of arithmetic truth (numbers, 0, +, and *, right?) and then your concept of 'the dreams of numbers', interviewing Lobian Machines, etc and came up with this.

One single irreducible digit ॐ which represents a self-dividing continuum of infinite perpendicular dialectics between eidetic dream states (in which dream~numbers escape their numerical identities as immersive qualitative experiences) and entopic non-dream states (in which number~dreams escape their dream nature as literal algebra-geometries).

This continuum f(ॐ), runs from infinitely solipsistic/private first person subjectivity (calling that Aleph ) to infinitely discrete/public third person mechanism (calling that Omega Ω), so that at ,any given dream is experienced as 99.99...9% dream and 0.00...1% number and at Ω, any given machine or number is presented as 99.99...9% number and 0.00...1% dream.

The halfway point between the and Ω axis is the perpendicular axis f(-ॐ) which is the high and low correspondence between the literal dream and figurative number (or figurative dream and literal number depending on whether you are using the dream-facing epistemology or the number-facing epistemology). This axis runs from tight equivalence ("=") to broadly elliptical potential set membership ("...")

So it looks something like this:

f(ॐ) ⊇ {ℵ "..." ⊥ "=" Ω}

To go further, it could be said that at Ω(Omega), ॐ (Om) expresses as 10|O (one, zero, line segment, circle referring to the quantitative algebraic and geometric perpendicular primitives) while at (Aleph), ॐ(Om) expresses as
יהוה (tetragrammaton or yod, hay, vov, hay, or in perhaps more familiar metaphor, ♣♠♥♦(clubs, spades, hearts, diamonds)

where:

♣ clubs (wands) =Fire, spiritual, tactile
♠ spades (swords) = Air, mental, auditory
♥ hearts (cups) =Water, emotional, visual
♦ diamonds (pentacles/coins) = Earth, physical, olfactory-gustatory

Note that tactile and auditory modalities tune us into ourselves and each others sensemaking (selves and minds), while the visual and olfactory/gustatory sense modalities are about objectifying realism of the world (egos or objectified selves/self-images and bodies). It should be obvious that ♣ clubs (wands) and ♠ spades (swords) are stereotypically masculine and abstracting forces, while ♥ hearts (cups) and ♦ diamonds (pentacles/coins) are stereotypically feminine objectified fields.

Sorry for the mumbo jumbo, but it is the only way to be non-reductive when approaching the qualitative side. We can't pretend to talk about the eidetic, dream like perpendicular of number logic while using the purely empirical terms of arithmetic reduction. We need symbols that can only refer to named qualities rather than enumerated quantities.

Let the ignoring and insulting begin!

Craig

Craig Weinberg

unread,
Aug 15, 2012, 5:21:15 AM8/15/12
to everyth...@googlegroups.com
in case the special characters don't come out...

I was thinking about your primitive of arithmetic truth (numbers, 0, +, and *, right?) and then your concept of ‘the dreams of numbers’, interviewing Lobian Machines, etc and came up with this.

One single irreducible digit ॐ (Om) which represents a self-dividing continuum of infinite perpendicular dialectics between eidetic dream states (in which dream~numbers escape their numerical identities as immersive qualitative experiences) and entopic non-dream states (in which number~dreams escape their dream nature as literal algebra-geometries).

This continuum f (ॐ(Om)), runs from infinitely solipsistic/private first person subjectivity (calling that Aleph )to infinitely discrete/public third person mechanism (calling that Omega Ω), so that at ,any given dream is experienced as 99.99…9% dream and 0.00…1% number and at Ω (Omega), any given machine or number is presented as 99.99…9% number and 0.00…1% dream.

The halfway point between the (Aleph) and Ω (Omega) axis is the perpendicular axis f (-ॐ(Om)) which is the high and low correspondence between the literal dream and figurative number (or figurative dream and literal number depending on whether you are using the dream-facing epistemology or the number-facing epistemology). This axis runs from tight equivalence (“=” equality) to broadly elliptical potential set membership (“…” ellipsis)



So it looks something like this:

f(ॐ) ⊇ {ℵ “…” ⊥ “=” Ω}

function (Om) is superset or equal to the continuum ranging from Aleph to ellipsis perpendicular/orthogonal to the inverse range from equality to Omega).

To go further, it could be said that at Ω(Omega), ॐ (Om) expresses as 10|O (one, zero, line segment, circle referring to the quantitative algebraic and geometric perpendicular primitives) while at (Aleph), ॐ (Om) expresses as


יהוה (tetragrammaton or yod, hay, vov, hay, or in perhaps more familiar metaphor, ♣♠♥♦(clubs, spades, hearts, diamonds)

Roger

unread,
Aug 17, 2012, 9:49:24 AM8/17/12
to everything-list
Hi Craig Weinberg
 
I was into the esoteric a decade ago, including the Tarot, and especially the Yi Ching.
whose ability to transform and embed and interlink metaphors is very powerful.  Being combinatorically
constructed, it is a complete, homogeneous and interlinked (hyperlinked) semantic field (to a certain
resolution).  You can do things with it not even dreamed of in western semantics and language processing. 
Leibniz almost discovered these properties. I developed a theory of story ujsing it (in the form of the Feng Shui).
See
 
 
Similarly I studied the time based version of the Yi Jing called the Tai Xuan Jing (T'ai Hsuan Ching)
which is ternary in form and especially mysterious and beautiful.
 
 
Then I went back tio the Lutheran Church and being conservative, and being advised and believing that such esoteric topics
(unfortunately used in fortune telling, forbidden by the Bible) are not a healthy pursuit, I gave up all of that stuff.
 
 
8/17/2012
Leibniz would say, "If there's no God, we'd have to invent him so everything could function."
----- Receiving the following content -----
Receiver: everything-list
Time: 2012-08-15, 05:05:44
Subject: Reconciling Bruno's Primitives with Multisense

Hi Bruno,

I was thinking about your primitive of arithmetic truth (numbers, 0, +, and *, right?) and then your concept of 'the dreams of numbers', interviewing Lobian Machines, etc and came up with this.

One single irreducible digit 锟斤拷 which represents a self-dividing continuum of infinite perpendicular dialectics between eidetic dream states (in which dream~numbers escape their numerical identities as immersive qualitative experiences) and entopic non-dream states (in which number~dreams escape their dream nature as literal algebra-geometries).

This continuum f(锟斤拷), runs from infinitely solipsistic/private first person subjectivity (calling that Aleph 锟斤拷) to infinitely discrete/public third person mechanism (calling that Omega 锟斤拷), so that at 锟斤拷,any given dream is experienced as 99.99...9% dream and 0.00...1% number and at 锟斤拷, any given machine or number is presented as 99.99...9% number and 0.00...1% dream.

The halfway point between the 锟斤拷 and 锟斤拷 axis is the perpendicular axis f(-锟斤拷) which is the high and low correspondence between the literal dream and figurative number (or figurative dream and literal number depending on whether you are using the dream-facing epistemology or the number-facing epistemology). This axis runs from tight equivalence ("=") to broadly elliptical potential set membership ("...")


So it looks something like this:

f(锟斤拷) 锟斤拷 {锟斤拷 "..." 锟斤拷 "=" 锟斤拷}

To go further, it could be said that at 锟斤拷(Omega), 锟斤拷 (Om) expresses as 10|O (one, zero, line segment, circle referring to the quantitative algebraic and geometric perpendicular primitives) while at 锟斤拷 (Aleph), 锟斤拷(Om) expresses as
锟斤拷锟斤拷锟斤拷锟斤拷 (tetragrammaton or yod, hay, vov, hay, or in perhaps more familiar metaphor, 锟斤拷锟斤拷锟斤拷锟斤拷(clubs, spades, hearts, diamonds)

where:

锟斤拷 clubs (wands) =Fire, spiritual, tactile
锟斤拷 spades (swords) = Air, mental, auditory
锟斤拷 hearts (cups) =Water, emotional, visual
锟斤拷 diamonds (pentacles/coins) = Earth, physical, olfactory-gustatory

Note that tactile and auditory modalities tune us into ourselves and each others sensemaking (selves and minds), while the visual and olfactory/gustatory sense modalities are about objectifying realism of the world (egos or objectified selves/self-images and bodies). It should be obvious that 锟斤拷 clubs (wands) and 锟斤拷 spades (swords) are stereotypically masculine and abstracting forces, while 锟斤拷 hearts (cups) and 锟斤拷 diamonds (pentacles/coins) are stereotypically feminine objectified fields.

Sorry for the mumbo jumbo, but it is the only way to be non-reductive when approaching the qualitative side. We can't pretend to talk about the eidetic, dream like perpendicular of number logic while using the purely empirical terms of arithmetic reduction. We need symbols that can only refer to named qualities rather than enumerated quantities.

Let the ignoring and insulting begin!

Craig

--
You received this message because you are subscribed to the Google Groups "Everything List" group.
To view this discussion on the web visit https://groups.google.com/d/msg/everything-list/-/1BiqAleIH0kJ.
To post to this group, send email to everyth...@googlegroups.com.
To unsubscribe from this group, send email to everything-li...@googlegroups.com.
For more options, visit this group at http://groups.google.com/group/everything-list?hl=en.

Bruno Marchal

unread,
Aug 17, 2012, 10:48:04 AM8/17/12
to everyth...@googlegroups.com
Hi Craig,

On 15 Aug 2012, at 11:21, Craig Weinberg wrote:

> in case the special characters don't come out...
>
> I was thinking about your primitive of arithmetic truth (numbers, 0,
> +, and *, right?) and then your concept of ‘the dreams of
> numbers’, interviewing Lobian Machines, etc and came up with this.
>
> One single irreducible digit ॐ (Om) which represents a self-
> dividing continuum of infinite perpendicular dialectics between
> eidetic dream states (in which dream~numbers escape their numerical
> identities as immersive qualitative experiences) and entopic non-
> dream states (in which number~dreams escape their dream nature as
> literal algebra-geometries).
>
I use such term more literally. I am not sure I can understand this,
even if there is some genuine analogy.

The dreaming number are usually very big concrete number. They dream
by encoding computational state of person, relatively to some
universal number, which are encoding universal machine relatively to
some other one, and the initial one can be chosen arbitrary. Those are
not symbolic number, but real encoding number, a bit like the genome
if you want.



>
> This continuum f (ॐ(Om)), runs from infinitely solipsistic/private
> first person subjectivity (calling that Aleph ℵ)to infinitely
> discrete/public third person mechanism (calling that Omega Ω), so
> that at ℵ,any given dream is experienced as 99.99…9% dream and
> 0.00…1% number and at Ω (Omega), any given machine or number is
> presented as 99.99…9% number and 0.00…1% dream.
>

?
I don't think so. Aristotle invented modal logic to treat in the
quantitative way non reductive qualitative notion.



> We can’t pretend to talk about the eidetic, dream like
> perpendicular of number logic while using the purely empirical terms
> of arithmetic reduction. We need symbols that can only refer to
> named qualities rather than enumerated quantities.
>
This is exactly what happen when you define the first person by the
knower. Bp & p, or if you prefer

provable(p) and true(p),

gives a modality which can provably be shown qualitative, and non
formalizable in arithmetic. It leads to a logic (know as S4Grz) which
describes something which is absolutely impossible to reduce to any
number relations or even anything third person describable notion,
even infinite one.

You might think I just described it, by Bp & p, or by "provable(p) and
true(p)", but this is not the case, as I use some of your intuition
about truth, which cannot be arithmetized by itself, by a famous
result of Gödel and Tarski (independently).
It happens that we do have a good intuition of many truth, and machine
can indeed describe better and better approximations of the truth
concept, but the limit of it, used here, cannot be. So by using both
the comp hypothesis, and by studying simple (Löbian) machine (simpler
than us) we can develop a formal (quantitative in some sense, at some
level, from some point of view) theory concerning the non formal, and
even non-formalizable-at-all-by-the-machine, qualities that machine
can still refer about. And this can be used to explain why machine are
forced to be befuddled by the subjectively-real apparent gap between
third and first person attributes.


> Let the ignoring and insulting begin!
>

We don't need that here, I think, nor anywhere. An insult is almost
always equivalent with "I have no argument".

Bruno


http://iridia.ulb.ac.be/~marchal/



Craig Weinberg

unread,
Aug 17, 2012, 12:35:03 PM8/17/12
to everyth...@googlegroups.com
Thanks Roger,

Your work on this looks very interesting. I think I get the gist of it but I will have to take a closer look.

I wonder how would fortune telling not include weather reports, actuarial tables, financial forecasts, etc? Historically there doesn't seem to be any meaningful correlation between fortune telling and any particular danger to people as a whole. Certainly no more danger than drinking wine or eating ice cream.

Craig

Craig Weinberg

unread,
Aug 17, 2012, 1:15:47 PM8/17/12
to everyth...@googlegroups.com


On Friday, August 17, 2012 10:48:04 AM UTC-4, Bruno Marchal wrote:
Hi Craig,

On 15 Aug 2012, at 11:21, Craig Weinberg wrote:

> in case the special characters don't come out...
>
> I was thinking about your primitive of arithmetic truth (numbers, 0,  
> +, and *, right?) and then your concept of ‘the dreams of  
> numbers’, interviewing Lobian Machines, etc and came up with this.
>
> One single irreducible digit ॐ (Om) which represents a self-
> dividing continuum of infinite perpendicular dialectics between  
> eidetic dream states (in which dream~numbers escape their numerical  
> identities as immersive qualitative experiences) and entopic non-
> dream states (in which number~dreams escape their dream nature as  
> literal algebra-geometries).
>
I use such term more literally. I am not sure I can understand this,  
even if there is some genuine analogy.

Think of it like π, except that instead of circumference and diameter, there is eidetic-figurative and entopic-literal presentation modalities.

The dreaming number are usually very big concrete number. They dream  
by encoding computational state of person, relatively to some  
universal number, which are encoding universal machine relatively to  
some other one, and the initial one can be chosen arbitrary. Those are  
not symbolic number, but real encoding number, a bit like the genome  
if you want.

Why would that result in a dream? It seems shrouded in obfuscating self-reference. Why would anything that has been encoded ever need to be decoded if the machine can fluently process the encoded form? Why would it need any other form - especially if it is all made of numbers?

What I am saying is that if you are going to invoke a possibility of dreams, that has to be grounded in the terms that you are laying out as primitive. Why would dreams leap out of mechanical relations? Even if there was some purpose for it, how could that actually take place - what are the dreamings made of?

My view is that it may be the case that everything that is not matter across space is experience through time - by definition, ontologically. There is no other form or content possible in the cosmos. Numbers are experiences as they must be inferred by computational agents and cannot exist independently of them. What my formulas do is to propose a precise relation between dream-time (including logical algebras) and matter-space (including topological geometries). To do this we need to invoke a continuity between them which is a perpendicular axis which runs from the literal (tight equivalence; induction is accomplished through linear arithmetic logic) to the figurative/metaphorical (loose thematic association; induction is accomplished through linear logic as well as elliptical cross-context leaps).




>
> This continuum f (ॐ(Om)), runs from infinitely solipsistic/private  
> first person subjectivity (calling that Aleph ℵ)to infinitely  
> discrete/public third person mechanism (calling that Omega Ω), so  
> that at ℵ,any given dream is experienced as 99.99…9% dream and  
> 0.00…1% number and at Ω (Omega), any given machine or number is  
> presented as 99.99…9% number and 0.00…1% dream.
>

?


I'm mapping out this literal to figurative axis, as it modifies the axis of subject to object presentations. The more an experience extends figuratively/metaphorically, the less it extends literally/mechanically.


What I am saying though is that *any quantitative treatment of qualitative experience is an unrecoverably catastrophic reduction*. Modal logic can make a toy model of experience, by removing all of the experience and replacing it with a meaningless variable. It is to say 'here, instead of your baby, let's just call it 'consumer of diapers and milk' and design a nursery based on the statistics derived from that consumption. What is needed is a way to reference phenomenological qualities which expresses not merely non-comp, but explicitly asserts quality and a view of the universe from the perspective of irreducible quality.


> We can’t pretend to talk about the eidetic, dream like  
> perpendicular of number logic while using the purely empirical terms  
> of arithmetic reduction. We need symbols that can only refer to  
> named qualities rather than enumerated quantities.
>
This is exactly what happen when you define the first person by the  
knower. Bp & p, or if you prefer

provable(p) and true(p),

provable and true are not first person qualities, they are epistemological quantifications. First person would be more like great(p) and superior(p).


gives a modality which can provably be shown qualitative, and non  
formalizable in arithmetic. It leads to a logic (know as S4Grz) which  
describes something which is absolutely impossible to reduce to any  
number relations or even anything third person describable notion,  
even infinite one.

That doesn't mean it is qualitative, only that it is so obscurely arithmetic that arithmetic itself cannot quantify it. What quality does this modality have? Is it shiny? Is it spicy? Does it get tired?


You might think I just described it, by Bp & p, or by "provable(p) and  
true(p)", but this is not the case, as I use some of your intuition  
about truth, which cannot be arithmetized by itself, by a famous  
result of Gödel and Tarski (independently).
It happens that we do have a good intuition of many truth, and machine  
can indeed describe better and better approximations of the truth  
concept, but the limit of it, used here, cannot be. So by using both  
the comp hypothesis, and by studying simple (Löbian) machine (simpler  
than us) we can develop a formal (quantitative in some sense, at some  
level, from some point of view) theory concerning the non formal, and  
even non-formalizable-at-all-by-the-machine, qualities that machine  
can still refer about. And this can be used to explain why machine are  
forced to be befuddled by the subjectively-real apparent gap between  
third and first person attributes.

It seems like you are missing the obvious. Awareness is not just about knowing and navigating a set of logical conditions. That can be accomplished easily without any awareness. Experienced qualities are orthogonal to knowledge and procedural evaluation.
 


> Let the ignoring and insulting begin!
>

We don't need that here, I think, nor anywhere. An insult is almost  
always equivalent with "I have no argument".

I agree.

Craig
 

Bruno


http://iridia.ulb.ac.be/~marchal/



Stephen P. King

unread,
Aug 17, 2012, 3:04:27 PM8/17/12
to everyth...@googlegroups.com
On 8/17/2012 10:48 AM, Bruno Marchal wrote:
> Hi Craig,
>
> On 15 Aug 2012, at 11:21, Craig Weinberg wrote:
>
>> in case the special characters don't come out...
>>
>> I was thinking about your primitive of arithmetic truth (numbers, 0,
>> +, and *, right?) and then your concept of ‘the dreams of numbers’,
>> interviewing Lobian Machines, etc and came up with this.
>>
>> One single irreducible digit ॐ (Om) which represents a self-dividing
>> continuum of infinite perpendicular dialectics between eidetic dream
>> states (in which dream~numbers escape their numerical identities as
>> immersive qualitative experiences) and entopic non-dream states (in
>> which number~dreams escape their dream nature as literal
>> algebra-geometries).
>>
> I use such term more literally. I am not sure I can understand this,
> even if there is some genuine analogy.
>
> The dreaming number are usually very big concrete number. They dream
> by encoding computational state of person, relatively to some
> universal number, which are encoding universal machine relatively to
> some other one, and the initial one can be chosen arbitrary. Those are
> not symbolic number, but real encoding number, a bit like the genome
> if you want.

Dear Bruno,

Could you elaborate as to how you explain the means by which an
encoding (which is an equivalence relation of sorts between one set and
another) is a generative action such that dreams obtain? I would very
much like to better understand how you obtain the appearance of chance
from purely static relations. I ask this as I simply do not see how you
can claim to explain actions in terms of purely non-active relations.
Craig's ideas assume activity at a primitive level and thus puts his
considerations at odds with yours in an almost irreconcilable way.



snip

--
Onward!

Stephen

"Nature, to be commanded, must be obeyed."
~ Francis Bacon


Roger

unread,
Aug 17, 2012, 5:46:59 PM8/17/12
to everything-list
Hi Craig Weinberg
 
You are right in a sense.  Weather prediction is a form of "fortune-telling".
 
But the reason traditional fortune-telling is frowned on by the Bible is that
it invokes powers outside of God or over God (Thou shalt have no other
God before me).
 
I don't consider weather prediction as a replacement for God, so no problem.
 
A more common false God however is your career, and we're all guilty of that.  
 
 
8/17/2012
Leibniz would say, "If there's no God, we'd have to invent him so everything could function."
----- Receiving the following content -----
Receiver: everything-list
Time: 2012-08-17, 12:35:03
Subject: Re: The I Ching, a cominatorically complete hyperlinked semantic field(mind).

One single irreducible digit which represents a self-dividing continuum of infinite perpendicular dialectics between eidetic dream states (in which dream~numbers escape their numerical identities as immersive qualitative experiences) and entopic non-dream states (in which number~dreams escape their dream nature as literal algebra-geometries).

This continuum f( ), runs from infinitely solipsistic/private first person subjectivity (calling that Aleph ) to infinitely discrete/public third person mechanism (calling that Omega ), so that at ,any given dream is experienced as 99.99...9% dream and 0.00...1% number and at , any given machine or number is presented as 99.99...9% number and 0.00...1% dream.

The halfway point between the and axis is the perpendicular axis f(- ) which is the high and low correspondence between the literal dream and figurative number (or figurative dream and literal number depending on whether you are using the dream-facing epistemology or the number-facing epistemology). This axis runs from tight equivalence ("=") to broadly elliptical potential set membership ("...")


So it looks something like this:

f( ) { "..." "=" }

To go further, it could be said that at (Omega), (Om) expresses as 10|O (one, zero, line segment, circle referring to the quantitative algebraic and geometric perpendicular primitives) while at (Aleph), (Om) expresses as
(tetragrammaton or yod, hay, vov, hay, or in perhaps more familiar metaphor, (clubs, spades, hearts, diamonds)

where:

clubs (wands) =Fire, spiritual, tactile


spades (swords) = Air, mental, auditory

hearts (cups) =Water, emotional, visual

diamonds (pentacles/coins) = Earth, physical, olfactory-gustatory

Note that tactile and auditory modalities tune us into ourselves and each others sensemaking (selves and minds), while the visual and olfactory/gustatory sense modalities are about objectifying realism of the world (egos or objectified selves/self-images and bodies). It should be obvious that clubs (wands) and spades (swords) are stereotypically masculine and abstracting forces, while hearts (cups) and diamonds (pentacles/coins) are stereotypically feminine objectified fields.

Sorry for the mumbo jumbo, but it is the only way to be non-reductive when approaching the qualitative side. We can't pretend to talk about the eidetic, dream like perpendicular of number logic while using the purely empirical terms of arithmetic reduction. We need symbols that can only refer to named qualities rather than enumerated quantities.

Let the ignoring and insulting begin!

Craig

--
You received this message because you are subscribed to the Google Groups "Everything List" group.
To view this discussion on the web visit https://groups.google.com/d/msg/everything-list/-/1BiqAleIH0kJ.
To post to this group, send email to everyth...@googlegroups.com.
To unsubscribe from this group, send email to everything-li...@googlegroups.com.
For more options, visit this group at http://groups.google.com/group/everything-list?hl=en.

--
You received this message because you are subscribed to the Google Groups "Everything List" group.
To view this discussion on the web visit https://groups.google.com/d/msg/everything-list/-/IeeEQl5Je5QJ.

Bruno Marchal

unread,
Aug 18, 2012, 5:59:32 AM8/18/12
to everyth...@googlegroups.com
On 17 Aug 2012, at 19:15, Craig Weinberg wrote:



On Friday, August 17, 2012 10:48:04 AM UTC-4, Bruno Marchal wrote:
Hi Craig,

On 15 Aug 2012, at 11:21, Craig Weinberg wrote:

> in case the special characters don't come out...
>
> I was thinking about your primitive of arithmetic truth (numbers, 0,  
> +, and *, right?) and then your concept of ‘the dreams of  
> numbers’, interviewing Lobian Machines, etc and came up with this.
>
> One single irreducible digit ॐ (Om) which represents a self-
> dividing continuum of infinite perpendicular dialectics between  
> eidetic dream states (in which dream~numbers escape their numerical  
> identities as immersive qualitative experiences) and entopic non-
> dream states (in which number~dreams escape their dream nature as  
> literal algebra-geometries).
>
I use such term more literally. I am not sure I can understand this,  
even if there is some genuine analogy.

Think of it like π, except that instead of circumference and diameter, there is eidetic-figurative and entopic-literal presentation modalities.

Pi = ratio of the length or a circle and its diameter. That is understandable.
"eidetic-figurative and entropic-literal presentation modalities." is not.






The dreaming number are usually very big concrete number. They dream  
by encoding computational state of person, relatively to some  
universal number, which are encoding universal machine relatively to  
some other one, and the initial one can be chosen arbitrary. Those are  
not symbolic number, but real encoding number, a bit like the genome  
if you want.

Why would that result in a dream?

Because I work in the comp theory where we come to the idea that consciousness can be manifested by abstract relation between numbers, as they emulate computation. We have already said "yes" to the doctor.




It seems shrouded in obfuscating self-reference. Why would anything that has been encoded ever need to be decoded if the machine can fluently process the encoded form?

To store what we learn. The DNA plays already such a role at the molecular level. It illustrates also a digital encoding and decoding. 



Why would it need any other form - especially if it is all made of numbers?

Nobody needs a universe. Why do we do babies?
The "truth" is that we have them, we cannot really avoid them. It is like the prime number and the universal machine. Once you have zero and + and *, you get Platonia, and a lot of mess in Platonia. It is a logical consequence.



What I am saying is that if you are going to invoke a possibility of dreams, that has to be grounded in the terms that you are laying out as primitive. Why would dreams leap out of mechanical relations?

It is a logical consequence, once you accept the idea that you might survive with a digital brain.



Even if there was some purpose for it, how could that actually take place - what are the dreamings made of?

Ontologically: nothing take place. All the computations are there. Some emulate self-observing machine and the math explain why they have to be beffudled by existence and conscience.



My view is that it may be the case that everything that is not matter across space is experience through time - by definition, ontologically. There is no other form or content possible in the cosmos. Numbers are experiences as they must be inferred by computational agents and cannot exist independently of them. What my formulas do is to propose a precise relation between dream-time (including logical algebras) and matter-space (including topological geometries). To do this we need to invoke a continuity between them which is a perpendicular axis which runs from the literal (tight equivalence; induction is accomplished through linear arithmetic logic) to the figurative/metaphorical (loose thematic association; induction is accomplished through linear logic as well as elliptical cross-context leaps).

I don't believe in time, space, cosmos, matter, ...
I explain their appearance by the dream property of numbers, relatively to universal numbers.







>
> This continuum f (ॐ(Om)), runs from infinitely solipsistic/private  
> first person subjectivity (calling that Aleph ℵ)to infinitely  
> discrete/public third person mechanism (calling that Omega Ω), so  
> that at ℵ,any given dream is experienced as 99.99…9% dream and  
> 0.00…1% number and at Ω (Omega), any given machine or number is  
> presented as 99.99…9% number and 0.00…1% dream.
>

?


I'm mapping out this literal to figurative axis, as it modifies the axis of subject to object presentations. The more an experience extends figuratively/metaphorically, the less it extends literally/mechanically.

That makes some sense.
I agree with this. Comp prevents to do such a reduction about what a machine can be and can be capable of.



Modal logic can make a toy model of experience, by removing all of the experience and replacing it with a meaningless variable.

Why meaningless?



It is to say 'here, instead of your baby, let's just call it 'consumer of diapers and milk' and design a nursery based on the statistics derived from that consumption.

No, it is the complete contrary. I see your point, but it is eaxtly that type of reduction that is prevented by comp. Even for machine, we can no more reduce them to their third person description. They do have a soul, even after-life, etc.



What is needed is a way to reference phenomenological qualities which expresses not merely non-comp, but explicitly asserts quality and a view of the universe from the perspective of irreducible quality.

You have to postualte them, and to postulate matter, and to postulate a relation, and non-comp, and that is neither satisfying, nor working. It introduces difficulties where there are already enough, imo.




> We can’t pretend to talk about the eidetic, dream like  
> perpendicular of number logic while using the purely empirical terms  
> of arithmetic reduction. We need symbols that can only refer to  
> named qualities rather than enumerated quantities.
>
This is exactly what happen when you define the first person by the  
knower. Bp & p, or if you prefer

provable(p) and true(p),

provable and true are not first person qualities, they are epistemological quantifications. First person would be more like great(p) and superior(p).

?




gives a modality which can provably be shown qualitative, and non  
formalizable in arithmetic. It leads to a logic (know as S4Grz) which  
describes something which is absolutely impossible to reduce to any  
number relations or even anything third person describable notion,  
even infinite one.

That doesn't mean it is qualitative, only that it is so obscurely arithmetic that arithmetic itself cannot quantify it.

But the machine still can refer to it. So I don't see why this would not work. It would not work if you reduce a machine to its body, but the divergence between Bp and Bp & p disallow such a reduction.


What quality does this modality have? Is it shiny? Is it spicy? Does it get tired?

The modality has no quality. It describes qualities. You might do a confusion of level. It is a bit like the confusion between the string "s(s(0))" and the number s(s(0)).





You might think I just described it, by Bp & p, or by "provable(p) and  
true(p)", but this is not the case, as I use some of your intuition  
about truth, which cannot be arithmetized by itself, by a famous  
result of Gödel and Tarski (independently).
It happens that we do have a good intuition of many truth, and machine  
can indeed describe better and better approximations of the truth  
concept, but the limit of it, used here, cannot be. So by using both  
the comp hypothesis, and by studying simple (Löbian) machine (simpler  
than us) we can develop a formal (quantitative in some sense, at some  
level, from some point of view) theory concerning the non formal, and  
even non-formalizable-at-all-by-the-machine, qualities that machine  
can still refer about. And this can be used to explain why machine are  
forced to be befuddled by the subjectively-real apparent gap between  
third and first person attributes.

It seems like you are missing the obvious. Awareness is not just about knowing and navigating a set of logical conditions. That can be accomplished easily without any awareness. Experienced qualities are orthogonal to knowledge and procedural evaluation.

I can understand that "navigating a set of logical conditions" can be done without awareness. Awareness or knowlegde arrives when the navigating embed the navigator in truth, or in a reality. There is a fixed point, and it is explained why this is felt as a personal non communicable experience.

 


> Let the ignoring and insulting begin!
>

We don't need that here, I think, nor anywhere. An insult is almost  
always equivalent with "I have no argument".

I agree.

I know. You are wise. And we agree on many things, but we are working in antipodal theories. I think you have reductionist conception of machine, to be franc.

Bruno



--
You received this message because you are subscribed to the Google Groups "Everything List" group.
To view this discussion on the web visit https://groups.google.com/d/msg/everything-list/-/wpi1rDQ5fT8J.

To post to this group, send email to everyth...@googlegroups.com.
To unsubscribe from this group, send email to everything-li...@googlegroups.com.
For more options, visit this group at http://groups.google.com/group/everything-list?hl=en.

Bruno Marchal

unread,
Aug 18, 2012, 6:20:47 AM8/18/12
to everyth...@googlegroups.com
?


> is a generative action such that dreams obtain?

I work in the comp theory, so I postulate that consciousness can be
manifested through a computation.



> I would very much like to better understand how you obtain the
> appearance of chance from purely static relations. I ask this as I
> simply do not see how you can claim to explain actions in terms of
> purely non-active relations. Craig's ideas assume activity at a
> primitive level and thus puts his considerations at odds with yours
> in an almost irreconcilable way.

There are different form of chance. A "real randomness" is given by
the first person indeterminacy bearing on all computation (aka UD*,
AKA arithmetic).
Yes, Craig's theory is non-comp. I suspect more and more that you
defend also non-comp, but unlike Craig, you seem to want to deny this.

Bruno

http://iridia.ulb.ac.be/~marchal/



Stephen P. King

unread,
Aug 18, 2012, 7:41:40 AM8/18/12
to everyth...@googlegroups.com
On 8/18/2012 6:20 AM, Bruno Marchal wrote:
>
> On 17 Aug 2012, at 21:04, Stephen P. King wrote:
>
>> On 8/17/2012 10:48 AM, Bruno Marchal wrote:
>>> Hi Craig,
>>>
>>> On 15 Aug 2012, at 11:21, Craig Weinberg wrote:
>>>
>>>> in case the special characters don't come out...
>>>>
>>>> I was thinking about your primitive of arithmetic truth (numbers,
>>>> 0, +, and *, right?) and then your concept of ‘the dreams of
>>>> numbers’, interviewing Lobian Machines, etc and came up with this.
>>>>
>>>> One single irreducible digit ॐ (Om) which represents a
>>>> self-dividing continuum of infinite perpendicular dialectics
>>>> between eidetic dream states (in which dream~numbers escape their
>>>> numerical identities as immersive qualitative experiences) and
>>>> entopic non-dream states (in which number~dreams escape their dream
>>>> nature as literal algebra-geometries).
>>>>
>>> I use such term more literally. I am not sure I can understand this,
>>> even if there is some genuine analogy.
>>>
>>> The dreaming number are usually very big concrete number. They dream
>>> by encoding computational state of person, relatively to some
>>> universal number, which are encoding universal machine relatively to
>>> some other one, and the initial one can be chosen arbitrary. Those
>>> are not symbolic number, but real encoding number, a bit like the
>>> genome if you want.
>>
>> Dear Bruno,
>>
>> Could you elaborate as to how you explain the means by which an
>> encoding (which is an equivalence relation of sorts between one set
>> and another)
>
> ?

How do you define "encoding"? What kind of mathematical entity is it?

>
>
>> is a generative action such that dreams obtain?
>
> I work in the comp theory, so I postulate that consciousness can be
> manifested through a computation.

I am thinking that the my self-simulation idea of identity requires
this, so we agree a tiny bit. I do define computations as *any*
transformation of information and information I define as "a difference
between two that makes a difference to a third".

>
>
>
>> I would very much like to better understand how you obtain the
>> appearance of chance from purely static relations. I ask this as I
>> simply do not see how you can claim to explain actions in terms of
>> purely non-active relations. Craig's ideas assume activity at a
>> primitive level and thus puts his considerations at odds with yours
>> in an almost irreconcilable way.
>
> There are different form of chance. A "real randomness" is given by
> the first person indeterminacy bearing on all computation (aka UD*,
> AKA arithmetic).
> Yes, Craig's theory is non-comp. I suspect more and more that you
> defend also non-comp, but unlike Craig, you seem to want to deny this.
>
> Bruno
>
> http://iridia.ulb.ac.be/~marchal/
>
>
>


Bruno Marchal

unread,
Aug 18, 2012, 9:40:01 AM8/18/12
to everyth...@googlegroups.com

On 18 Aug 2012, at 13:41, Stephen P. King wrote:

> On 8/18/2012 6:20 AM, Bruno Marchal wrote:
>>
>> On 17 Aug 2012, at 21:04, Stephen P. King wrote:
>>
>>> On 8/17/2012 10:48 AM, Bruno Marchal wrote:
>>>> <snip>
>>>>
>>>> The dreaming number are usually very big concrete number. They
>>>> dream by encoding computational state of person, relatively to
>>>> some universal number, which are encoding universal machine
>>>> relatively to some other one, and the initial one can be chosen
>>>> arbitrary. Those are not symbolic number, but real encoding
>>>> number, a bit like the genome if you want.
>>>
>>> Dear Bruno,
>>>
>>> Could you elaborate as to how you explain the means by which an
>>> encoding (which is an equivalence relation of sorts between one
>>> set and another)
>>
>> ?
>
> How do you define "encoding"? What kind of mathematical entity is
> it?

I define it by its program, and its semantics. So you can see encoding
as defined by the number k such that phi_k(x) is an encoding function.
So its precise definition can be given by a number:

encoding = s(s(s(s(s(s(s(s(s(s(s(s(s......s(0)))))))) ...)

with the right number of parenthesis.

Equivalently I could define encoding by a SK-combinators, or by a lisp
program. All those definition are provably (in peano arithmetic for
example) equivalent.


>
>>
>>
>>> is a generative action such that dreams obtain?
>>
>> I work in the comp theory, so I postulate that consciousness can be
>> manifested through a computation.
>
> I am thinking that the my self-simulation idea of identity
> requires this, so we agree a tiny bit. I do define computations as
> *any* transformation of information and information I define as "a
> difference between two that makes a difference to a third".

Computation is a more easy concept than "information", which is a bit
a trash word in which people put usually many different things.
Computation admits Church's thesis. Information admits many non
equivalent definitions. It is an important cloud of important notion,
but I would not use it to define computation, which, thanks to CT, is
much more easy to define in a mathematical proper way.

Bruno


>
>>
>>
>>
>>> I would very much like to better understand how you obtain the
>>> appearance of chance from purely static relations. I ask this as I
>>> simply do not see how you can claim to explain actions in terms of
>>> purely non-active relations. Craig's ideas assume activity at a
>>> primitive level and thus puts his considerations at odds with
>>> yours in an almost irreconcilable way.
>>
>> There are different form of chance. A "real randomness" is given by
>> the first person indeterminacy bearing on all computation (aka UD*,
>> AKA arithmetic).
>> Yes, Craig's theory is non-comp. I suspect more and more that you
>> defend also non-comp, but unlike Craig, you seem to want to deny
>> this.
>>
>> Bruno
>>
>> http://iridia.ulb.ac.be/~marchal/
>>
>>
>>
>
>
> --
> Onward!
>
> Stephen
>
> "Nature, to be commanded, must be obeyed."
> ~ Francis Bacon
>
>
> --
> You received this message because you are subscribed to the Google
> Groups "Everything List" group.

Roger

unread,
Aug 18, 2012, 9:50:50 AM8/18/12
to everything-list
Hi Bruno Marchal
 
BTW how can they know if the calculation of pi is very precise if they
don't really know beforehand what its precise value should be ?
 
 
 
8/18/2012
Leibniz would say, "If there's no God, we'd have to invent him so everything could function."
----- Receiving the following content -----
Receiver: everything-list
Time: 2012-08-18, 05:59:32
Subject: Re: Reconciling Bruno's Primitives with Multisense


On 17 Aug 2012, at 19:15, Craig Weinberg wrote:



On Friday, August 17, 2012 10:48:04 AM UTC-4, Bruno Marchal wrote:
Hi Craig,

On 15 Aug 2012, at 11:21, Craig Weinberg wrote:

> in case the special characters don't come out...
>
> I was thinking about your primitive of arithmetic truth (numbers, 0,  
> +, and *, right?) and then your concept of 锟斤拷the dreams of  
> numbers锟斤拷, interviewing Lobian Machines, etc and came up with this.
>
> One single irreducible digit 锟斤拷 (Om) which represents a self-
> dividing continuum of infinite perpendicular dialectics between  
> eidetic dream states (in which dream~numbers escape their numerical  
> identities as immersive qualitative experiences) and entopic non-
> dream states (in which number~dreams escape their dream nature as  
> literal algebra-geometries).
>
I use such term more literally. I am not sure I can understand this,  
even if there is some genuine analogy.

Think of it like 锟斤拷, except that instead of circumference and diameter, there is eidetic-figurative and entopic-literal presentation modalities.

Pi = ratio of the length or a circle and its diameter. That is understandable.
"eidetic-figurative and entropic-literal presentation modalities." is not.






The dreaming number are usually very big concrete number. They dream  
by encoding computational state of person, relatively to some  
universal number, which are encoding universal machine relatively to  
some other one, and the initial one can be chosen arbitrary. Those are  
not symbolic number, but real encoding number, a bit like the genome  
if you want.

Why would that result in a dream?

Because I work in the comp theory where we come to the idea that consciousness can be manifested by abstract relation between numbers, as they emulate computation. We have already said "yes" to the doctor.




It seems shrouded in obfuscating self-reference. Why would anything that has been encoded ever need to be decoded if the machine can fluently process the encoded form?

To store what we learn. The DNA plays already such a role at the molecular level. It illustrates also a digital encoding and decoding. 



Why would it need any other form - especially if it is all made of numbers?

Nobody needs a universe. Why do we do babies?
The "truth" is that we have them, we cannot really avoid them. It is like the prime number and the universal machine. Once you have zero and + and *, you get Platonia, and a lot of mess in Platonia. It is a logical consequence.



What I am saying is that if you are going to invoke a possibility of dreams, that has to be grounded in the terms that you are laying out as primitive. Why would dreams leap out of mechanical relations?

It is a logical consequence, once you accept the idea that you might survive with a digital brain.



Even if there was some purpose for it, how could that actually take place - what are the dreamings made of?

Ontologically: nothing take place. All the computations are there. Some emulate self-observing machine and the math explain why they have to be beffudled by existence and conscience.



My view is that it may be the case that everything that is not matter across space is experience through time - by definition, ontologically. There is no other form or content possible in the cosmos. Numbers are experiences as they must be inferred by computational agents and cannot exist independently of them. What my formulas do is to propose a precise relation between dream-time (including logical algebras) and matter-space (including topological geometries). To do this we need to invoke a continuity between them which is a perpendicular axis which runs from the literal (tight equivalence; induction is accomplished through linear arithmetic logic) to the figurative/metaphorical (loose thematic association; induction is accomplished through linear logic as well as elliptical cross-context leaps).

I don't believe in time, space, cosmos, matter, ...
I explain their appearance by the dream property of numbers, relatively to universal numbers.







>
> This continuum f (锟斤拷(Om)), runs from infinitely solipsistic/private  
> first person subjectivity (calling that Aleph 锟斤拷)to infinitely  
> discrete/public third person mechanism (calling that Omega 锟斤拷), so  
> that at 锟斤拷,any given dream is experienced as 99.99锟斤拷9% dream and  
> 0.00锟斤拷1% number and at 锟斤拷 (Omega), any given machine or number is  
> presented as 99.99锟斤拷9% number and 0.00锟斤拷1% dream.
>

?


I'm mapping out this literal to figurative axis, as it modifies the axis of subject to object presentations. The more an experience extends figuratively/metaphorically, the less it extends literally/mechanically.

That makes some sense.



>
> The halfway point between the 锟斤拷 (Aleph) and 锟斤拷 (Omega) axis is the  
> perpendicular axis f (-锟斤拷(Om)) which is the high and low  
> correspondence between the literal dream and figurative number (or  
> figurative dream and literal number depending on whether you are  
> using the dream-facing epistemology or the number-facing  
> epistemology). This axis runs from tight equivalence (锟斤拷=锟斤拷  
> equality) to broadly elliptical potential set membership (锟斤拷锟斤拷锟斤拷  
> ellipsis)
>
> So it looks something like this:
>
> f(锟斤拷) 锟斤拷 {锟斤拷 锟斤拷锟斤拷锟斤拷 锟斤拷 锟斤拷=锟斤拷 锟斤拷}
>
> function (Om) is superset or equal to the continuum ranging from  
> Aleph to ellipsis perpendicular/orthogonal to the inverse range from  
> equality to Omega).
>
> To go further, it could be said that at 锟斤拷(Omega), 锟斤拷 (Om) expresses  
> as 10|O (one, zero, line segment, circle referring to the  
> quantitative algebraic and geometric perpendicular primitives) while  
> at 锟斤拷 (Aleph), 锟斤拷 (Om) expresses as
> 锟斤拷锟斤拷锟斤拷锟斤拷 (tetragrammaton or yod, hay, vov, hay, or in perhaps more  
> familiar metaphor, 锟斤拷锟斤拷锟斤拷锟斤拷(clubs, spades, hearts, diamonds)
>
> where:
>
> 锟斤拷 clubs (wands) =Fire, spiritual, tactile
> 锟斤拷 spades (swords) = Air, mental, auditory
> 锟斤拷 hearts (cups) =Water, emotional, visual
> 锟斤拷 diamonds (pentacles/coins) = Earth, physical, olfactory-gustatory
>
> Note that tactile and auditory modalities tune us into ourselves and  
> each others sensemaking (selves and minds), while the visual and  
> olfactory/gustatory sense modalities are about objectifying realism  
> of the world (egos or objectified selves/self-images and bodies). It  
> should be obvious that 锟斤拷 clubs (wands) and 锟斤拷 spades (swords) are  
> stereotypically masculine and abstracting forces, while 锟斤拷 hearts  
> (cups) and 锟斤拷 diamonds (pentacles/coins) are stereotypically  
> feminine objectified fields.
>
> Sorry for the mumbo jumbo, but it is the only way to be non-
> reductive when approaching the qualitative side.
>
I don't think so. Aristotle invented modal logic to treat in the  
quantitative way non reductive qualitative notion.


What I am saying though is that *any quantitative treatment of qualitative experience is an unrecoverably catastrophic reduction*.

I agree with this. Comp prevents to do such a reduction about what a machine can be and can be capable of.



Modal logic can make a toy model of experience, by removing all of the experience and replacing it with a meaningless variable.

Why meaningless?



It is to say 'here, instead of your baby, let's just call it 'consumer of diapers and milk' and design a nursery based on the statistics derived from that consumption.

No, it is the complete contrary. I see your point, but it is eaxtly that type of reduction that is prevented by comp. Even for machine, we can no more reduce them to their third person description. They do have a soul, even after-life, etc.



What is needed is a way to reference phenomenological qualities which expresses not merely non-comp, but explicitly asserts quality and a view of the universe from the perspective of irreducible quality.

You have to postualte them, and to postulate matter, and to postulate a relation, and non-comp, and that is neither satisfying, nor working. It introduces difficulties where there are already enough, imo.




> We can锟斤拷t pretend to talk about the eidetic, dream like  
> perpendicular of number logic while using the purely empirical terms  
> of arithmetic reduction. We need symbols that can only refer to  
> named qualities rather than enumerated quantities.
>
This is exactly what happen when you define the first person by the  
knower. Bp & p, or if you prefer

provable(p) and true(p),

provable and true are not first person qualities, they are epistemological quantifications. First person would be more like great(p) and superior(p).

?




gives a modality which can provably be shown qualitative, and non  
formalizable in arithmetic. It leads to a logic (know as S4Grz) which  
describes something which is absolutely impossible to reduce to any  
number relations or even anything third person describable notion,  
even infinite one.

That doesn't mean it is qualitative, only that it is so obscurely arithmetic that arithmetic itself cannot quantify it.

But the machine still can refer to it. So I don't see why this would not work. It would not work if you reduce a machine to its body, but the divergence between Bp and Bp & p disallow such a reduction.


What quality does this modality have? Is it shiny? Is it spicy? Does it get tired?

The modality has no quality. It describes qualities. You might do a confusion of level. It is a bit like the confusion between the string "s(s(0))" and the number s(s(0)).




You might think I just described it, by Bp & p, or by "provable(p) and  
true(p)", but this is not the case, as I use some of your intuition  
about truth, which cannot be arithmetized by itself, by a famous  
result of G锟斤拷del and Tarski (independently).
It happens that we do have a good intuition of many truth, and machine  
can indeed describe better and better approximations of the truth  
concept, but the limit of it, used here, cannot be. So by using both  
the comp hypothesis, and by studying simple (L锟斤拷bian) machine (simpler  
than us) we can develop a formal (quantitative in some sense, at some  
level, from some point of view) theory concerning the non formal, and  
even non-formalizable-at-all-by-the-machine, qualities that machine  
can still refer about. And this can be used to explain why machine are  
forced to be befuddled by the subjectively-real apparent gap between  
third and first person attributes.

It seems like you are missing the obvious. Awareness is not just about knowing and navigating a set of logical conditions. That can be accomplished easily without any awareness. Experienced qualities are orthogonal to knowledge and procedural evaluation.

I can understand that "navigating a set of logical conditions" can be done without awareness. Awareness or knowlegde arrives when the navigating embed the navigator in truth, or in a reality. There is a fixed point, and it is explained why this is felt as a personal non communicable experience.


 


> Let the ignoring and insulting begin!
>

We don't need that here, I think, nor anywhere. An insult is almost  
always equivalent with "I have no argument".

I agree.

I know. You are wise. And we agree on many things, but we are working in antipodal theories. I think you have reductionist conception of machine, to be franc.

Bruno




On Friday, August 17, 2012 10:48:04 AM UTC-4, Bruno Marchal wrote:
Hi Craig,

On 15 Aug 2012, at 11:21, Craig Weinberg wrote:

> in case the special characters don't come out...
>
> I was thinking about your primitive of arithmetic truth (numbers, 0,  
> +, and *, right?) and then your concept of 锟斤拷the dreams of  
> numbers锟斤拷, interviewing Lobian Machines, etc and came up with this.
>
> One single irreducible digit 锟斤拷 (Om) which represents a self-
> dividing continuum of infinite perpendicular dialectics between  
> eidetic dream states (in which dream~numbers escape their numerical  
> identities as immersive qualitative experiences) and entopic non-
> dream states (in which number~dreams escape their dream nature as  
> literal algebra-geometries).
>
I use such term more literally. I am not sure I can understand this,  
even if there is some genuine analogy.

The dreaming number are usually very big concrete number. They dream  
by encoding computational state of person, relatively to some  
universal number, which are encoding universal machine relatively to  
some other one, and the initial one can be chosen arbitrary. Those are  
not symbolic number, but real encoding number, a bit like the genome  
if you want.



>
> This continuum f (锟斤拷(Om)), runs from infinitely solipsistic/private  
> first person subjectivity (calling that Aleph 锟斤拷)to infinitely  
> discrete/public third person mechanism (calling that Omega 锟斤拷), so  
> that at 锟斤拷,any given dream is experienced as 99.99锟斤拷9% dream and  
> 0.00锟斤拷1% number and at 锟斤拷 (Omega), any given machine or number is  
> presented as 99.99锟斤拷9% number and 0.00锟斤拷1% dream.
>

?
>
> The halfway point between the 锟斤拷 (Aleph) and 锟斤拷 (Omega) axis is the  
> perpendicular axis f (-锟斤拷(Om)) which is the high and low  
> correspondence between the literal dream and figurative number (or  
> figurative dream and literal number depending on whether you are  
> using the dream-facing epistemology or the number-facing  
> epistemology). This axis runs from tight equivalence (锟斤拷=锟斤拷  
> equality) to broadly elliptical potential set membership (锟斤拷锟斤拷锟斤拷  
> ellipsis)
>
> So it looks something like this:
>
> f(锟斤拷) 锟斤拷 {锟斤拷 锟斤拷锟斤拷锟斤拷 锟斤拷 锟斤拷=锟斤拷 锟斤拷}
>
> function (Om) is superset or equal to the continuum ranging from  
> Aleph to ellipsis perpendicular/orthogonal to the inverse range from  
> equality to Omega).
>
> To go further, it could be said that at 锟斤拷(Omega), 锟斤拷 (Om) expresses  
> as 10|O (one, zero, line segment, circle referring to the  
> quantitative algebraic and geometric perpendicular primitives) while  
> at 锟斤拷 (Aleph), 锟斤拷 (Om) expresses as
> 锟斤拷锟斤拷锟斤拷锟斤拷 (tetragrammaton or yod, hay, vov, hay, or in perhaps more  
> familiar metaphor, 锟斤拷锟斤拷锟斤拷锟斤拷(clubs, spades, hearts, diamonds)
>
> where:
>
> 锟斤拷 clubs (wands) =Fire, spiritual, tactile
> 锟斤拷 spades (swords) = Air, mental, auditory
> 锟斤拷 hearts (cups) =Water, emotional, visual
> 锟斤拷 diamonds (pentacles/coins) = Earth, physical, olfactory-gustatory
>
> Note that tactile and auditory modalities tune us into ourselves and  
> each others sensemaking (selves and minds), while the visual and  
> olfactory/gustatory sense modalities are about objectifying realism  
> of the world (egos or objectified selves/self-images and bodies). It  
> should be obvious that 锟斤拷 clubs (wands) and 锟斤拷 spades (swords) are  
> stereotypically masculine and abstracting forces, while 锟斤拷 hearts  
> (cups) and 锟斤拷 diamonds (pentacles/coins) are stereotypically  
> feminine objectified fields.
>
> Sorry for the mumbo jumbo, but it is the only way to be non-
> reductive when approaching the qualitative side.
>
I don't think so. Aristotle invented modal logic to treat in the  
quantitative way non reductive qualitative notion.



> We can锟斤拷t pretend to talk about the eidetic, dream like  
> perpendicular of number logic while using the purely empirical terms  
> of arithmetic reduction. We need symbols that can only refer to  
> named qualities rather than enumerated quantities.
>
This is exactly what happen when you define the first person by the  
knower. Bp & p, or if you prefer

provable(p) and true(p),

gives a modality which can provably be shown qualitative, and non  
formalizable in arithmetic. It leads to a logic (know as S4Grz) which  
describes something which is absolutely impossible to reduce to any  
number relations or even anything third person describable notion,  
even infinite one.

You might think I just described it, by Bp & p, or by "provable(p) and  
true(p)", but this is not the case, as I use some of your intuition  
about truth, which cannot be arithmetized by itself, by a famous  
result of G锟斤拷del and Tarski (independently).
It happens that we do have a good intuition of many truth, and machine  
can indeed describe better and better approximations of the truth  
concept, but the limit of it, used here, cannot be. So by using both  
the comp hypothesis, and by studying simple (L锟斤拷bian) machine (simpler  
than us) we can develop a formal (quantitative in some sense, at some  
level, from some point of view) theory concerning the non formal, and  
even non-formalizable-at-all-by-the-machine, qualities that machine  
can still refer about. And this can be used to explain why machine are  
forced to be befuddled by the subjectively-real apparent gap between  
third and first person attributes.


> Let the ignoring and insulting begin!
>

We don't need that here, I think, nor anywhere. An insult is almost  
always equivalent with "I have no argument".

Bruno


http://iridia.ulb.ac.be/~marchal/




--
You received this message because you are subscribed to the Google Groups "Everything List" group.
To view this discussion on the web visit https://groups.google.com/d/msg/everything-list/-/wpi1rDQ5fT8J.
To post to this group, send email to everyth...@googlegroups.com.
To unsubscribe from this group, send email to everything-li...@googlegroups.com.
For more options, visit this group at http://groups.google.com/group/everything-list?hl=en.

Bruno Marchal

unread,
Aug 18, 2012, 10:24:08 AM8/18/12
to everyth...@googlegroups.com
On 18 Aug 2012, at 15:50, Roger wrote:

Hi Bruno Marchal
 
BTW how can they know if the calculation of pi is very precise if they
don't really know beforehand what its precise value should be ?


Because PI is a clear concept, and you can prove that some algorithm computes its exact value, as Archimedes was already aware.

For example, you can prove (convince yourself) that the sequence of [perimeter of regular polygons divided by they greatest diagonal] will get closer and closer to PI when the number of sides is bigger and bigger, and so you can compute PI exactly. Intuitively you might be able to conceive that a regular (symmetrical) polygon having a large number of side looks like a circle, and its greatest diagonal looks like a diameter.

The same for sqrt(2), e, gamma, etc. Those are known as constructive or computable real numbers, and can be (re)defined as computable function from N to N, for example the function given the nth decimal, or more sophisticate one to ensure that the addition and multiplication of constructive real numbers give constructive real numbers (which is not the case with the simple minded definition I just gave).

By Cantor non-enumerability theorem, the computable real numbers constitute a minority among all real numbers.

Bruno



 
 
 
8/18/2012
Leibniz would say, "If there's no God, we'd have to invent him so everything could function."
----- Receiving the following content -----
Receiver: everything-list
Time: 2012-08-18, 05:59:32
Subject: Re: Reconciling Bruno's Primitives with Multisense

On 17 Aug 2012, at 19:15, Craig Weinberg wrote:



On Friday, August 17, 2012 10:48:04 AM UTC-4, Bruno Marchal wrote:
Hi Craig,

On 15 Aug 2012, at 11:21, Craig Weinberg wrote:

> in case the special characters don't come out...
>
> I was thinking about your primitive of arithmetic truth (numbers, 0,  
> +, and *, right?) and then your concept of ‘the dreams of  
> numbers’, interviewing Lobian Machines, etc and came up with this.
>
> One single irreducible digit   (Om) which represents a self-
> dividing continuum of infinite perpendicular dialectics between  
> eidetic dream states (in which dream~numbers escape their numerical  
> identities as immersive qualitative experiences) and entopic non-
> dream states (in which number~dreams escape their dream nature as  
> literal algebra-geometries).
>
I use such term more literally. I am not sure I can understand this,  
even if there is some genuine analogy.

Think of it like π, except that instead of circumference and diameter, there is eidetic-figurative and entopic-literal presentation modalities.

Pi = ratio of the length or a circle and its diameter. That is understandable.
"eidetic-figurative and entropic-literal presentation modalities." is not.






The dreaming number are usually very big concrete number. They dream  
by encoding computational state of person, relatively to some  
universal number, which are encoding universal machine relatively to  
some other one, and the initial one can be chosen arbitrary. Those are  
not symbolic number, but real encoding number, a bit like the genome  
if you want.

Why would that result in a dream?

Because I work in the comp theory where we come to the idea that consciousness can be manifested by abstract relation between numbers, as they emulate computation. We have already said "yes" to the doctor.




It seems shrouded in obfuscating self-reference. Why would anything that has been encoded ever need to be decoded if the machine can fluently process the encoded form?

To store what we learn. The DNA plays already such a role at the molecular level. It illustrates also a digital encoding and decoding. 



Why would it need any other form - especially if it is all made of numbers?

Nobody needs a universe. Why do we do babies?
The "truth" is that we have them, we cannot really avoid them. It is like the prime number and the universal machine. Once you have zero and + and *, you get Platonia, and a lot of mess in Platonia. It is a logical consequence.



What I am saying is that if you are going to invoke a possibility of dreams, that has to be grounded in the terms that you are laying out as primitive. Why would dreams leap out of mechanical relations?

It is a logical consequence, once you accept the idea that you might survive with a digital brain.



Even if there was some purpose for it, how could that actually take place - what are the dreamings made of?

Ontologically: nothing take place. All the computations are there. Some emulate self-observing machine and the math explain why they have to be beffudled by existence and conscience.



My view is that it may be the case that everything that is not matter across space is experience through time - by definition, ontologically. There is no other form or content possible in the cosmos. Numbers are experiences as they must be inferred by computational agents and cannot exist independently of them. What my formulas do is to propose a precise relation between dream-time (including logical algebras) and matter-space (including topological geometries). To do this we need to invoke a continuity between them which is a perpendicular axis which runs from the literal (tight equivalence; induction is accomplished through linear arithmetic logic) to the figurative/metaphorical (loose thematic association; induction is accomplished through linear logic as well as elliptical cross-context leaps).

I don't believe in time, space, cosmos, matter, ...
I explain their appearance by the dream property of numbers, relatively to universal numbers.







>
> This continuum f ( (Om)), runs from infinitely solipsistic/private  
> first person subjectivity (calling that Aleph  )to infinitely  
> discrete/public third person mechanism (calling that Omega Ω), so  
> that at  ,any given dream is experienced as 99.99…9% dream and  
> 0.00…1% number and at Ω (Omega), any given machine or number is  
> presented as 99.99…9% number and 0.00…1% dream.
>

?


I'm mapping out this literal to figurative axis, as it modifies the axis of subject to object presentations. The more an experience extends figuratively/metaphorically, the less it extends literally/mechanically.

That makes some sense.



>
> The halfway point between the   (Aleph) and Ω (Omega) axis is the  
> perpendicular axis f (- (Om)) which is the high and low  
> correspondence between the literal dream and figurative number (or  
> figurative dream and literal number depending on whether you are  
> using the dream-facing epistemology or the number-facing  
> epistemology). This axis runs from tight equivalence (“=”  
> equality) to broadly elliptical potential set membership (“…”  
> ellipsis)
>
> So it looks something like this:
>
> f( )   {  “…” ⊥ “=” Ω}
>
> function (Om) is superset or equal to the continuum ranging from  
> Aleph to ellipsis perpendicular/orthogonal to the inverse range from  
> equality to Omega).
>
> To go further, it could be said that at Ω(Omega),   (Om) expresses  
> as 10|O (one, zero, line segment, circle referring to the  
> quantitative algebraic and geometric perpendicular primitives) while  
> at   (Aleph),   (Om) expresses as
>      (tetragrammaton or yod, hay, vov, hay, or in perhaps more  
> familiar metaphor,     (clubs, spades, hearts, diamonds)
>
> where:
>
>   clubs (wands) =Fire, spiritual, tactile
>   spades (swords) = Air, mental, auditory
>   hearts (cups) =Water, emotional, visual
>   diamonds (pentacles/coins) = Earth, physical, olfactory-gustatory
>
> Note that tactile and auditory modalities tune us into ourselves and  
> each others sensemaking (selves and minds), while the visual and  
> olfactory/gustatory sense modalities are about objectifying realism  
> of the world (egos or objectified selves/self-images and bodies). It  
> should be obvious that   clubs (wands) and   spades (swords) are  
> stereotypically masculine and abstracting forces, while   hearts  
> (cups) and   diamonds (pentacles/coins) are stereotypically  
> feminine objectified fields.
>
> Sorry for the mumbo jumbo, but it is the only way to be non-
> reductive when approaching the qualitative side.
>
I don't think so. Aristotle invented modal logic to treat in the  
quantitative way non reductive qualitative notion.


What I am saying though is that *any quantitative treatment of qualitative experience is an unrecoverably catastrophic reduction*.

I agree with this. Comp prevents to do such a reduction about what a machine can be and can be capable of.



Modal logic can make a toy model of experience, by removing all of the experience and replacing it with a meaningless variable.

Why meaningless?



It is to say 'here, instead of your baby, let's just call it 'consumer of diapers and milk' and design a nursery based on the statistics derived from that consumption.

No, it is the complete contrary. I see your point, but it is eaxtly that type of reduction that is prevented by comp. Even for machine, we can no more reduce them to their third person description. They do have a soul, even after-life, etc.



What is needed is a way to reference phenomenological qualities which expresses not merely non-comp, but explicitly asserts quality and a view of the universe from the perspective of irreducible quality.

You have to postualte them, and to postulate matter, and to postulate a relation, and non-comp, and that is neither satisfying, nor working. It introduces difficulties where there are already enough, imo.




> We can’t pretend to talk about the eidetic, dream like  
> perpendicular of number logic while using the purely empirical terms  
> of arithmetic reduction. We need symbols that can only refer to  
> named qualities rather than enumerated quantities.
>
This is exactly what happen when you define the first person by the  
knower. Bp & p, or if you prefer

provable(p) and true(p),

provable and true are not first person qualities, they are epistemological quantifications. First person would be more like great(p) and superior(p).

?




gives a modality which can provably be shown qualitative, and non  
formalizable in arithmetic. It leads to a logic (know as S4Grz) which  
describes something which is absolutely impossible to reduce to any  
number relations or even anything third person describable notion,  
even infinite one.

That doesn't mean it is qualitative, only that it is so obscurely arithmetic that arithmetic itself cannot quantify it.

But the machine still can refer to it. So I don't see why this would not work. It would not work if you reduce a machine to its body, but the divergence between Bp and Bp & p disallow such a reduction.


What quality does this modality have? Is it shiny? Is it spicy? Does it get tired?

The modality has no quality. It describes qualities. You might do a confusion of level. It is a bit like the confusion between the string "s(s(0))" and the number s(s(0)).




You might think I just described it, by Bp & p, or by "provable(p) and  
true(p)", but this is not the case, as I use some of your intuition  
about truth, which cannot be arithmetized by itself, by a famous  
result of G del and Tarski (independently).
It happens that we do have a good intuition of many truth, and machine  
can indeed describe better and better approximations of the truth  
concept, but the limit of it, used here, cannot be. So by using both  
the comp hypothesis, and by studying simple (L bian) machine (simpler  
than us) we can develop a formal (quantitative in some sense, at some  
level, from some point of view) theory concerning the non formal, and  
even non-formalizable-at-all-by-the-machine, qualities that machine  
can still refer about. And this can be used to explain why machine are  
forced to be befuddled by the subjectively-real apparent gap between  
third and first person attributes.

It seems like you are missing the obvious. Awareness is not just about knowing and navigating a set of logical conditions. That can be accomplished easily without any awareness. Experienced qualities are orthogonal to knowledge and procedural evaluation.

I can understand that "navigating a set of logical conditions" can be done without awareness. Awareness or knowlegde arrives when the navigating embed the navigator in truth, or in a reality. There is a fixed point, and it is explained why this is felt as a personal non communicable experience.


 


> Let the ignoring and insulting begin!
>

We don't need that here, I think, nor anywhere. An insult is almost  
always equivalent with "I have no argument".

I agree.

I know. You are wise. And we agree on many things, but we are working in antipodal theories. I think you have reductionist conception of machine, to be franc.

Bruno




On Friday, August 17, 2012 10:48:04 AM UTC-4, Bruno Marchal wrote:
Hi Craig,

On 15 Aug 2012, at 11:21, Craig Weinberg wrote:

> in case the special characters don't come out...
>
> I was thinking about your primitive of arithmetic truth (numbers, 0,  
> +, and *, right?) and then your concept of ‘the dreams of  
> numbers’, interviewing Lobian Machines, etc and came up with this.
>
> One single irreducible digit   (Om) which represents a self-
> dividing continuum of infinite perpendicular dialectics between  
> eidetic dream states (in which dream~numbers escape their numerical  
> identities as immersive qualitative experiences) and entopic non-
> dream states (in which number~dreams escape their dream nature as  
> literal algebra-geometries).
>
I use such term more literally. I am not sure I can understand this,  
even if there is some genuine analogy.

The dreaming number are usually very big concrete number. They dream  
by encoding computational state of person, relatively to some  
universal number, which are encoding universal machine relatively to  
some other one, and the initial one can be chosen arbitrary. Those are  
not symbolic number, but real encoding number, a bit like the genome  
if you want.



>
> This continuum f ( (Om)), runs from infinitely solipsistic/private  
> first person subjectivity (calling that Aleph  )to infinitely  
> discrete/public third person mechanism (calling that Omega Ω), so  
> that at  ,any given dream is experienced as 99.99…9% dream and  
> 0.00…1% number and at Ω (Omega), any given machine or number is  
> presented as 99.99…9% number and 0.00…1% dream.
>

?
>
> The halfway point between the   (Aleph) and Ω (Omega) axis is the  
> perpendicular axis f (- (Om)) which is the high and low  
> correspondence between the literal dream and figurative number (or  
> figurative dream and literal number depending on whether you are  
> using the dream-facing epistemology or the number-facing  
> epistemology). This axis runs from tight equivalence (“=”  
> equality) to broadly elliptical potential set membership (“…”  
> ellipsis)
>
> So it looks something like this:
>
> f( )   {  “…” ⊥ “=” Ω}
>
> function (Om) is superset or equal to the continuum ranging from  
> Aleph to ellipsis perpendicular/orthogonal to the inverse range from  
> equality to Omega).
>
> To go further, it could be said that at Ω(Omega),   (Om) expresses  
> as 10|O (one, zero, line segment, circle referring to the  
> quantitative algebraic and geometric perpendicular primitives) while  
> at   (Aleph),   (Om) expresses as
>      (tetragrammaton or yod, hay, vov, hay, or in perhaps more  
> familiar metaphor,     (clubs, spades, hearts, diamonds)
>
> where:
>
>   clubs (wands) =Fire, spiritual, tactile
>   spades (swords) = Air, mental, auditory
>   hearts (cups) =Water, emotional, visual
>   diamonds (pentacles/coins) = Earth, physical, olfactory-gustatory
>
> Note that tactile and auditory modalities tune us into ourselves and  
> each others sensemaking (selves and minds), while the visual and  
> olfactory/gustatory sense modalities are about objectifying realism  
> of the world (egos or objectified selves/self-images and bodies). It  
> should be obvious that   clubs (wands) and   spades (swords) are  
> stereotypically masculine and abstracting forces, while   hearts  
> (cups) and   diamonds (pentacles/coins) are stereotypically  
> feminine objectified fields.
>
> Sorry for the mumbo jumbo, but it is the only way to be non-
> reductive when approaching the qualitative side.
>
I don't think so. Aristotle invented modal logic to treat in the  
quantitative way non reductive qualitative notion.



> We can’t pretend to talk about the eidetic, dream like  
> perpendicular of number logic while using the purely empirical terms  
> of arithmetic reduction. We need symbols that can only refer to  
> named qualities rather than enumerated quantities.
>
This is exactly what happen when you define the first person by the  
knower. Bp & p, or if you prefer

provable(p) and true(p),

gives a modality which can provably be shown qualitative, and non  
formalizable in arithmetic. It leads to a logic (know as S4Grz) which  
describes something which is absolutely impossible to reduce to any  
number relations or even anything third person describable notion,  
even infinite one.

You might think I just described it, by Bp & p, or by "provable(p) and  
true(p)", but this is not the case, as I use some of your intuition  
about truth, which cannot be arithmetized by itself, by a famous  
result of G del and Tarski (independently).
It happens that we do have a good intuition of many truth, and machine  
can indeed describe better and better approximations of the truth  
concept, but the limit of it, used here, cannot be. So by using both  
the comp hypothesis, and by studying simple (L bian) machine (simpler  
than us) we can develop a formal (quantitative in some sense, at some  
level, from some point of view) theory concerning the non formal, and  
even non-formalizable-at-all-by-the-machine, qualities that machine  
can still refer about. And this can be used to explain why machine are  
forced to be befuddled by the subjectively-real apparent gap between  
third and first person attributes.


> Let the ignoring and insulting begin!
>

We don't need that here, I think, nor anywhere. An insult is almost  
always equivalent with "I have no argument".

Bruno


http://iridia.ulb.ac.be/~marchal/




--
You received this message because you are subscribed to the Google Groups "Everything List" group.
To view this discussion on the web visit https://groups.google.com/d/msg/everything-list/-/wpi1rDQ5fT8J.
To post to this group, send email to everyth...@googlegroups.com.
To unsubscribe from this group, send email to everything-li...@googlegroups.com.
For more options, visit this group at http://groups.google.com/group/everything-list?hl=en.

--
You received this message because you are subscribed to the Google Groups "Everything List" group.
To post to this group, send email to everyth...@googlegroups.com.
To unsubscribe from this group, send email to everything-li...@googlegroups.com.
For more options, visit this group at http://groups.google.com/group/everything-list?hl=en.

William R. Buckley

unread,
Aug 18, 2012, 10:25:27 AM8/18/12
to everyth...@googlegroups.com

Roger:

 

The same question can be asked of you.  Thus,

 

How can you know if your calculation of pi is precise without your having prior knowledge of that value?

 

By whatever mechanism you use to obtain precious precision, the computer can act the same.

 

wrb

 

 

 

From: everyth...@googlegroups.com [mailto:everyth...@googlegroups.com] On Behalf Of Roger
Sent: Saturday, August 18, 2012 6:51 AM
To: everything-list
Subject: On calculating pi

 

Hi Bruno Marchal

 

BTW how can they know if the calculation of pi is very precise if they

don't really know beforehand what its precise value should be ?

 

 

 

8/18/2012

Leibniz would say, "If there's no God, we'd have to invent him so everything could function."

----- Receiving the following content -----

Receiver: everything-list

Time: 2012-08-18, 05:59:32

Subject: Re: Reconciling Bruno's Primitives with Multisense

 

On 17 Aug 2012, at 19:15, Craig Weinberg wrote:





On Friday, August 17, 2012 10:48:04 AM UTC-4, Bruno Marchal wrote:

Hi Craig,

On 15 Aug 2012, at 11:21, Craig Weinberg wrote:

> in case the special characters don't come out...
>
> I was thinking about your primitive of arithmetic truth (numbers, 0,  

> +, and *, right?) and then your concept of ‘the dreams of  

> numbers’, interviewing Lobian Machines, etc and came up with this.
>
> One single irreducible digit
  (Om) which represents a self-

> dividing continuum of infinite perpendicular dialectics between  
> eidetic dream states (in which dream~numbers escape their numerical  
> identities as immersive qualitative experiences) and entopic non-
> dream states (in which number~dreams escape their dream nature as  
> literal algebra-geometries).
>
I use such term more literally. I am not sure I can understand this,  
even if there is some genuine analogy.


Think of it like π, except that instead of circumference and diameter, there is eidetic-figurative and entopic-literal presentation modalities.

 

Pi = ratio of the length or a circle and its diameter. That is understandable.

"eidetic-figurative and entropic-literal presentation modalities." is not.

 

 

 

 




The dreaming number are usually very big concrete number. They dream  
by encoding computational state of person, relatively to some  
universal number, which are encoding universal machine relatively to  
some other one, and the initial one can be chosen arbitrary. Those are  
not symbolic number, but real encoding number, a bit like the genome  
if you want.


Why would that result in a dream?

 

Because I work in the comp theory where we come to the idea that consciousness can be manifested by abstract relation between numbers, as they emulate computation. We have already said "yes" to the doctor.

 

 

 



It seems shrouded in obfuscating self-reference. Why would anything that has been encoded ever need to be decoded if the machine can fluently process the encoded form?

 

To store what we learn. The DNA plays already such a role at the molecular level. It illustrates also a digital encoding and decoding. 

 

 



Why would it need any other form - especially if it is all made of numbers?

 

Nobody needs a universe. Why do we do babies?

The "truth" is that we have them, we cannot really avoid them. It is like the prime number and the universal machine. Once you have zero and + and *, you get Platonia, and a lot of mess in Platonia. It is a logical consequence.

 




What I am saying is that if you are going to invoke a possibility of dreams, that has to be grounded in the terms that you are laying out as primitive. Why would dreams leap out of mechanical relations?

 

It is a logical consequence, once you accept the idea that you might survive with a digital brain.

 

 



Even if there was some purpose for it, how could that actually take place - what are the dreamings made of?

 

Ontologically: nothing take place. All the computations are there. Some emulate self-observing machine and the math explain why they have to be beffudled by existence and conscience.

 




My view is that it may be the case that everything that is not matter across space is experience through time - by definition, ontologically. There is no other form or content possible in the cosmos. Numbers are experiences as they must be inferred by computational agents and cannot exist independently of them. What my formulas do is to propose a precise relation between dream-time (including logical algebras) and matter-space (including topological geometries). To do this we need to invoke a continuity between them which is a perpendicular axis which runs from the literal (tight equivalence; induction is accomplished through linear arithmetic logic) to the figurative/metaphorical (loose thematic association; induction is accomplished through linear logic as well as elliptical cross-context leaps).

 

I don't believe in time, space, cosmos, matter, ...

I explain their appearance by the dream property of numbers, relatively to universal numbers.

 

 



 




>
> This continuum f (
 (Om)), runs from infinitely solipsistic/private  

> first person subjectivity (calling that Aleph

 )to infinitely  

> discrete/public third person mechanism (calling that Omega Ω), so  

> that at  ,any given dream is experienced as 99.99…9% dream and  

> 0.00…1% number and at Ω (Omega), any given machine or number is  

> presented as 99.99…9% number and 0.00…1% dream.
>

?



I'm mapping out this literal to figurative axis, as it modifies the axis of subject to object presentations. The more an experience extends figuratively/metaphorically, the less it extends literally/mechanically.

 

That makes some sense.

 



 

>
> The halfway point between the
  (Aleph) and Ω (Omega) axis is the  
> perpendicular axis f (-
 (Om)) which is the high and low  

> correspondence between the literal dream and figurative number (or  
> figurative dream and literal number depending on whether you are  
> using the dream-facing epistemology or the number-facing  

> epistemology). This axis runs from tight equivalence (“=”  
> equality) to broadly elliptical potential set membership (“…”  

> ellipsis)
>
> So it looks something like this:
>

> f( )   {  “…” “=” Ω}

>
> function (Om) is superset or equal to the continuum ranging from  
> Aleph to ellipsis perpendicular/orthogonal to the inverse range from  
> equality to Omega).
>

> To go further, it could be said that at Ω(Omega),   (Om) expresses  

> as 10|O (one, zero, line segment, circle referring to the  
> quantitative algebraic and geometric perpendicular primitives) while  

> at   (Aleph),   (Om) expresses as
>
     (tetragrammaton or yod, hay, vov, hay, or in perhaps more  
> familiar metaphor,
    (clubs, spades, hearts, diamonds)
>
> where:
>
>
  clubs (wands) =Fire, spiritual, tactile
>
  spades (swords) = Air, mental, auditory
>
  hearts (cups) =Water, emotional, visual
>
  diamonds (pentacles/coins) = Earth, physical, olfactory-gustatory

>
> Note that tactile and auditory modalities tune us into ourselves and  
> each others sensemaking (selves and minds), while the visual and  
> olfactory/gustatory sense modalities are about objectifying realism  
> of the world (egos or objectified selves/self-images and bodies). It  

> should be obvious that   clubs (wands) and   spades (swords) are  
> stereotypically masculine and abstracting forces, while
  hearts  
> (cups) and
  diamonds (pentacles/coins) are stereotypically  

> feminine objectified fields.
>
> Sorry for the mumbo jumbo, but it is the only way to be non-
> reductive when approaching the qualitative side.
>
I don't think so. Aristotle invented modal logic to treat in the  
quantitative way non reductive qualitative notion.


What I am saying though is that *any quantitative treatment of qualitative experience is an unrecoverably catastrophic reduction*.

 

I agree with this. Comp prevents to do such a reduction about what a machine can be and can be capable of.

 

 



Modal logic can make a toy model of experience, by removing all of the experience and replacing it with a meaningless variable.

 

Why meaningless?

 

 



It is to say 'here, instead of your baby, let's just call it 'consumer of diapers and milk' and design a nursery based on the statistics derived from that consumption.

 

No, it is the complete contrary. I see your point, but it is eaxtly that type of reduction that is prevented by comp. Even for machine, we can no more reduce them to their third person description. They do have a soul, even after-life, etc.

 

 



What is needed is a way to reference phenomenological qualities which expresses not merely non-comp, but explicitly asserts quality and a view of the universe from the perspective of irreducible quality.

 

You have to postualte them, and to postulate matter, and to postulate a relation, and non-comp, and that is neither satisfying, nor working. It introduces difficulties where there are already enough, imo.

 



 


> We can’t pretend to talk about the eidetic, dream like  

> perpendicular of number logic while using the purely empirical terms  
> of arithmetic reduction. We need symbols that can only refer to  
> named qualities rather than enumerated quantities.
>
This is exactly what happen when you define the first person by the  
knower. Bp & p, or if you prefer

provable(p) and true(p),


provable and true are not first person qualities, they are epistemological quantifications. First person would be more like great(p) and superior(p).

 

?

 



 


gives a modality which can provably be shown qualitative, and non  
formalizable in arithmetic. It leads to a logic (know as S4Grz) which  
describes something which is absolutely impossible to reduce to any  
number relations or even anything third person describable notion,  
even infinite one.


That doesn't mean it is qualitative, only that it is so obscurely arithmetic that arithmetic itself cannot quantify it.

 

But the machine still can refer to it. So I don't see why this would not work. It would not work if you reduce a machine to its body, but the divergence between Bp and Bp & p disallow such a reduction.

 



What quality does this modality have? Is it shiny? Is it spicy? Does it get tired?

 

The modality has no quality. It describes qualities. You might do a confusion of level. It is a bit like the confusion between the string "s(s(0))" and the number s(s(0)).

 

 



 


You might think I just described it, by Bp & p, or by "provable(p) and  
true(p)", but this is not the case, as I use some of your intuition  
about truth, which cannot be arithmetized by itself, by a famous  

result of G del and Tarski (independently).

It happens that we do have a good intuition of many truth, and machine  
can indeed describe better and better approximations of the truth  
concept, but the limit of it, used here, cannot be. So by using both  

the comp hypothesis, and by studying simple (L bian) machine (simpler  

than us) we can develop a formal (quantitative in some sense, at some  
level, from some point of view) theory concerning the non formal, and  
even non-formalizable-at-all-by-the-machine, qualities that machine  
can still refer about. And this can be used to explain why machine are  
forced to be befuddled by the subjectively-real apparent gap between  
third and first person attributes.


It seems like you are missing the obvious. Awareness is not just about knowing and navigating a set of logical conditions. That can be accomplished easily without any awareness. Experienced qualities are orthogonal to knowledge and procedural evaluation.

 

I can understand that "navigating a set of logical conditions" can be done without awareness. Awareness or knowlegde arrives when the navigating embed the navigator in truth, or in a reality. There is a fixed point, and it is explained why this is felt as a personal non communicable experience.




 



> Let the ignoring and insulting begin!
>

We don't need that here, I think, nor anywhere. An insult is almost  
always equivalent with "I have no argument".


I agree.

 

I know. You are wise. And we agree on many things, but we are working in antipodal theories. I think you have reductionist conception of machine, to be franc.

 

Bruno

 

 

 


On Friday, August 17, 2012 10:48:04 AM UTC-4, Bruno Marchal wrote:

Hi Craig,

On 15 Aug 2012, at 11:21, Craig Weinberg wrote:

> in case the special characters don't come out...
>
> I was thinking about your primitive of arithmetic truth (numbers, 0,  

> +, and *, right?) and then your concept of ‘the dreams of  

> numbers’, interviewing Lobian Machines, etc and came up with this.
>
> One single irreducible digit
  (Om) which represents a self-

> dividing continuum of infinite perpendicular dialectics between  
> eidetic dream states (in which dream~numbers escape their numerical  
> identities as immersive qualitative experiences) and entopic non-
> dream states (in which number~dreams escape their dream nature as  
> literal algebra-geometries).
>
I use such term more literally. I am not sure I can understand this,  
even if there is some genuine analogy.

The dreaming number are usually very big concrete number. They dream  
by encoding computational state of person, relatively to some  
universal number, which are encoding universal machine relatively to  
some other one, and the initial one can be chosen arbitrary. Those are  
not symbolic number, but real encoding number, a bit like the genome  
if you want.



>

> This continuum f ( (Om)), runs from infinitely solipsistic/private  

> first person subjectivity (calling that Aleph

 )to infinitely  

> discrete/public third person mechanism (calling that Omega Ω), so  

> that at  ,any given dream is experienced as 99.99…9% dream and  

> 0.00…1% number and at Ω (Omega), any given machine or number is  

> presented as 99.99…9% number and 0.00…1% dream.
>

?
>
> The halfway point between the
  (Aleph) and Ω (Omega) axis is the  
> perpendicular axis f (-
 (Om)) which is the high and low  

> correspondence between the literal dream and figurative number (or  
> figurative dream and literal number depending on whether you are  
> using the dream-facing epistemology or the number-facing  

> epistemology). This axis runs from tight equivalence (“=”  
> equality) to broadly elliptical potential set membership (“…”  

> ellipsis)
>
> So it looks something like this:
>

> f( )   {  “…” “=” Ω}

>
> function (Om) is superset or equal to the continuum ranging from  
> Aleph to ellipsis perpendicular/orthogonal to the inverse range from  
> equality to Omega).
>

> To go further, it could be said that at Ω(Omega),   (Om) expresses  

> as 10|O (one, zero, line segment, circle referring to the  
> quantitative algebraic and geometric perpendicular primitives) while  

> at   (Aleph),   (Om) expresses as
>
     (tetragrammaton or yod, hay, vov, hay, or in perhaps more  
> familiar metaphor,
    (clubs, spades, hearts, diamonds)
>
> where:
>
>
  clubs (wands) =Fire, spiritual, tactile
>
  spades (swords) = Air, mental, auditory
>
  hearts (cups) =Water, emotional, visual
>
  diamonds (pentacles/coins) = Earth, physical, olfactory-gustatory

>
> Note that tactile and auditory modalities tune us into ourselves and  
> each others sensemaking (selves and minds), while the visual and  
> olfactory/gustatory sense modalities are about objectifying realism  
> of the world (egos or objectified selves/self-images and bodies). It  

> should be obvious that   clubs (wands) and   spades (swords) are  
> stereotypically masculine and abstracting forces, while
  hearts  
> (cups) and
  diamonds (pentacles/coins) are stereotypically  

> feminine objectified fields.
>
> Sorry for the mumbo jumbo, but it is the only way to be non-
> reductive when approaching the qualitative side.
>
I don't think so. Aristotle invented modal logic to treat in the  
quantitative way non reductive qualitative notion.



> We can’t pretend to talk about the eidetic, dream like  

> perpendicular of number logic while using the purely empirical terms  
> of arithmetic reduction. We need symbols that can only refer to  
> named qualities rather than enumerated quantities.
>
This is exactly what happen when you define the first person by the  
knower. Bp & p, or if you prefer

provable(p) and true(p),

gives a modality which can provably be shown qualitative, and non  
formalizable in arithmetic. It leads to a logic (know as S4Grz) which  
describes something which is absolutely impossible to reduce to any  
number relations or even anything third person describable notion,  
even infinite one.

You might think I just described it, by Bp & p, or by "provable(p) and  
true(p)", but this is not the case, as I use some of your intuition  
about truth, which cannot be arithmetized by itself, by a famous  

result of G del and Tarski (independently).

It happens that we do have a good intuition of many truth, and machine  
can indeed describe better and better approximations of the truth  
concept, but the limit of it, used here, cannot be. So by using both  

the comp hypothesis, and by studying simple (L bian) machine (simpler  

than us) we can develop a formal (quantitative in some sense, at some  
level, from some point of view) theory concerning the non formal, and  
even non-formalizable-at-all-by-the-machine, qualities that machine  
can still refer about. And this can be used to explain why machine are  
forced to be befuddled by the subjectively-real apparent gap between  
third and first person attributes.


> Let the ignoring and insulting begin!
>

We don't need that here, I think, nor anywhere. An insult is almost  
always equivalent with "I have no argument".

Bruno


http://iridia.ulb.ac.be/~marchal/


 

--
You received this message because you are subscribed to the Google Groups "Everything List" group.
To view this discussion on the web visit https://groups.google.com/d/msg/everything-list/-/wpi1rDQ5fT8J.
To post to this group, send email to everyth...@googlegroups.com.
To unsubscribe from this group, send email to everything-li...@googlegroups.com.
For more options, visit this group at http://groups.google.com/group/everything-list?hl=en.

John Clark

unread,
Aug 18, 2012, 11:01:39 AM8/18/12
to everyth...@googlegroups.com
On Sat, Aug 18, 2012 at 9:50 AM, Roger <rcl...@verizon.net> wrote:

>how can they know if the calculation of pi is very precise if they
don't really know beforehand what its precise value should be ?

But we do know the precise value of pi, 250 years ago Euler proved that pi squared divided by 6 is EXACTLY equal to   the infinite series 1+ 1/2 + 1/4 + 1/9 + 1/16 + 1/25 + 1/36 +1/49 +.......   Tell me how close to pi you want to get and a finite number of terms in this sequence can help you produce it.

  John K Clark

Roger

unread,
Aug 18, 2012, 11:19:27 AM8/18/12
to everything-list
Hi Bruno Marchal
 
Yes, you can square the square root of any number to test its accuracy,
but there are a variety of algorithms used to calculate pi.
 
Which is correct ? See
 
 
 
The value obtained is assumed to be true if the infinite series
used to calculate pi converges. But I would think that 
many if not most infinite series should
converge. Which one is the right one ? Is there a unique solution ?
 
 
 
 
8/18/2012
Leibniz would say, "If there's no God, we'd have to invent him so everything could function."
----- Receiving the following content -----
Receiver: everything-list
Time: 2012-08-18, 10:24:08
Subject: Re: On calculating pi


On 18 Aug 2012, at 15:50, Roger wrote:

Hi Bruno Marchal
 
BTW how can they know if the calculation of pi is very precise if they
don't really know beforehand what its precise value should be ?


Because PI is a clear concept, and you can prove that some algorithm computes its exact value, as Archimedes was already aware.

For example, you can prove (convince yourself) that the sequence of [perimeter of regular polygons divided by they greatest diagonal] will get closer and closer to PI when the number of sides is bigger and bigger, and so you can compute PI exactly. Intuitively you might be able to conceive that a regular (symmetrical) polygon having a large number of side looks like a circle, and its greatest diagonal looks like a diameter.

The same for sqrt(2), e, gamma, etc. Those are known as constructive or computable real numbers, and can be (re)defined as computable function from N to N, for example the function given the nth decimal, or more sophisticate one to ensure that the addition and multiplication of constructive real numbers give constructive real numbers (which is not the case with the simple minded definition I just gave).

By Cantor non-enumerability theorem, the computable real numbers constitute a minority among all real numbers.

Bruno



 
 
 
8/18/2012
Leibniz would say, "If there's no God, we'd have to invent him so everything could function."
----- Receiving the following content -----
Receiver: everything-list
Time: 2012-08-18, 05:59:32
Subject: Re: Reconciling Bruno's Primitives with Multisense

On 17 Aug 2012, at 19:15, Craig Weinberg wrote:



On Friday, August 17, 2012 10:48:04 AM UTC-4, Bruno Marchal wrote:
Hi Craig,

On 15 Aug 2012, at 11:21, Craig Weinberg wrote:

> in case the special characters don't come out...
>
> I was thinking about your primitive of arithmetic truth (numbers, 0,  
> +, and *, right?) and then your concept of 锟斤拷the dreams of  
> numbers锟斤拷, interviewing Lobian Machines, etc and came up with this.
>
> One single irreducible digit 锟斤拷 (Om) which represents a self-
> dividing continuum of infinite perpendicular dialectics between  
> eidetic dream states (in which dream~numbers escape their numerical  
> identities as immersive qualitative experiences) and entopic non-
> dream states (in which number~dreams escape their dream nature as  
> literal algebra-geometries).
>
I use such term more literally. I am not sure I can understand this,  
even if there is some genuine analogy.

Think of it like 锟斤拷, except that instead of circumference and diameter, there is eidetic-figurative and entopic-literal presentation modalities.

Pi = ratio of the length or a circle and its diameter. That is understandable.
"eidetic-figurative and entropic-literal presentation modalities." is not.






The dreaming number are usually very big concrete number. They dream  
by encoding computational state of person, relatively to some  
universal number, which are encoding universal machine relatively to  
some other one, and the initial one can be chosen arbitrary. Those are  
not symbolic number, but real encoding number, a bit like the genome  
if you want.

Why would that result in a dream?

Because I work in the comp theory where we come to the idea that consciousness can be manifested by abstract relation between numbers, as they emulate computation. We have already said "yes" to the doctor.




It seems shrouded in obfuscating self-reference. Why would anything that has been encoded ever need to be decoded if the machine can fluently process the encoded form?

To store what we learn. The DNA plays already such a role at the molecular level. It illustrates also a digital encoding and decoding. 



Why would it need any other form - especially if it is all made of numbers?

Nobody needs a universe. Why do we do babies?
The "truth" is that we have them, we cannot really avoid them. It is like the prime number and the universal machine. Once you have zero and + and *, you get Platonia, and a lot of mess in Platonia. It is a logical consequence.



What I am saying is that if you are going to invoke a possibility of dreams, that has to be grounded in the terms that you are laying out as primitive. Why would dreams leap out of mechanical relations?

It is a logical consequence, once you accept the idea that you might survive with a digital brain.



Even if there was some purpose for it, how could that actually take place - what are the dreamings made of?

Ontologically: nothing take place. All the computations are there. Some emulate self-observing machine and the math explain why they have to be beffudled by existence and conscience.



My view is that it may be the case that everything that is not matter across space is experience through time - by definition, ontologically. There is no other form or content possible in the cosmos. Numbers are experiences as they must be inferred by computational agents and cannot exist independently of them. What my formulas do is to propose a precise relation between dream-time (including logical algebras) and matter-space (including topological geometries). To do this we need to invoke a continuity between them which is a perpendicular axis which runs from the literal (tight equivalence; induction is accomplished through linear arithmetic logic) to the figurative/metaphorical (loose thematic association; induction is accomplished through linear logic as well as elliptical cross-context leaps).

I don't believe in time, space, cosmos, matter, ...
I explain their appearance by the dream property of numbers, relatively to universal numbers.







>
> This continuum f (锟斤拷(Om)), runs from infinitely solipsistic/private  
> first person subjectivity (calling that Aleph 锟斤拷)to infinitely  
> discrete/public third person mechanism (calling that Omega 锟斤拷), so  
> that at 锟斤拷,any given dream is experienced as 99.99锟斤拷9% dream and  
> 0.00锟斤拷1% number and at 锟斤拷 (Omega), any given machine or number is  
> presented as 99.99锟斤拷9% number and 0.00锟斤拷1% dream.
>

?


I'm mapping out this literal to figurative axis, as it modifies the axis of subject to object presentations. The more an experience extends figuratively/metaphorically, the less it extends literally/mechanically.

That makes some sense.



>
> The halfway point between the 锟斤拷 (Aleph) and 锟斤拷 (Omega) axis is the  
> perpendicular axis f (-锟斤拷(Om)) which is the high and low  
> correspondence between the literal dream and figurative number (or  
> figurative dream and literal number depending on whether you are  
> using the dream-facing epistemology or the number-facing  
> epistemology). This axis runs from tight equivalence (锟斤拷=锟斤拷  
> equality) to broadly elliptical potential set membership (锟斤拷锟斤拷锟斤拷  
> ellipsis)
>
> So it looks something like this:
>
> f(锟斤拷) 锟斤拷 {锟斤拷 锟斤拷锟斤拷锟斤拷 锟斤拷 锟斤拷=锟斤拷 锟斤拷}
>
> function (Om) is superset or equal to the continuum ranging from  
> Aleph to ellipsis perpendicular/orthogonal to the inverse range from  
> equality to Omega).
>
> To go further, it could be said that at 锟斤拷(Omega), 锟斤拷 (Om) expresses  
> as 10|O (one, zero, line segment, circle referring to the  
> quantitative algebraic and geometric perpendicular primitives) while  
> at 锟斤拷 (Aleph), 锟斤拷 (Om) expresses as
> 锟斤拷锟斤拷锟斤拷锟斤拷 (tetragrammaton or yod, hay, vov, hay, or in perhaps more  
> familiar metaphor, 锟斤拷锟斤拷锟斤拷锟斤拷(clubs, spades, hearts, diamonds)
>
> where:
>
> 锟斤拷 clubs (wands) =Fire, spiritual, tactile
> 锟斤拷 spades (swords) = Air, mental, auditory
> 锟斤拷 hearts (cups) =Water, emotional, visual
> 锟斤拷 diamonds (pentacles/coins) = Earth, physical, olfactory-gustatory
>
> Note that tactile and auditory modalities tune us into ourselves and  
> each others sensemaking (selves and minds), while the visual and  
> olfactory/gustatory sense modalities are about objectifying realism  
> of the world (egos or objectified selves/self-images and bodies). It  
> should be obvious that 锟斤拷 clubs (wands) and 锟斤拷 spades (swords) are  
> stereotypically masculine and abstracting forces, while 锟斤拷 hearts  
> (cups) and 锟斤拷 diamonds (pentacles/coins) are stereotypically  
> feminine objectified fields.
>
> Sorry for the mumbo jumbo, but it is the only way to be non-
> reductive when approaching the qualitative side.
>
I don't think so. Aristotle invented modal logic to treat in the  
quantitative way non reductive qualitative notion.


What I am saying though is that *any quantitative treatment of qualitative experience is an unrecoverably catastrophic reduction*.

I agree with this. Comp prevents to do such a reduction about what a machine can be and can be capable of.



Modal logic can make a toy model of experience, by removing all of the experience and replacing it with a meaningless variable.

Why meaningless?



It is to say 'here, instead of your baby, let's just call it 'consumer of diapers and milk' and design a nursery based on the statistics derived from that consumption.

No, it is the complete contrary. I see your point, but it is eaxtly that type of reduction that is prevented by comp. Even for machine, we can no more reduce them to their third person description. They do have a soul, even after-life, etc.



What is needed is a way to reference phenomenological qualities which expresses not merely non-comp, but explicitly asserts quality and a view of the universe from the perspective of irreducible quality.

You have to postualte them, and to postulate matter, and to postulate a relation, and non-comp, and that is neither satisfying, nor working. It introduces difficulties where there are already enough, imo.




> We can锟斤拷t pretend to talk about the eidetic, dream like  
> perpendicular of number logic while using the purely empirical terms  
> of arithmetic reduction. We need symbols that can only refer to  
> named qualities rather than enumerated quantities.
>
This is exactly what happen when you define the first person by the  
knower. Bp & p, or if you prefer

provable(p) and true(p),

provable and true are not first person qualities, they are epistemological quantifications. First person would be more like great(p) and superior(p).

?




gives a modality which can provably be shown qualitative, and non  
formalizable in arithmetic. It leads to a logic (know as S4Grz) which  
describes something which is absolutely impossible to reduce to any  
number relations or even anything third person describable notion,  
even infinite one.

That doesn't mean it is qualitative, only that it is so obscurely arithmetic that arithmetic itself cannot quantify it.

But the machine still can refer to it. So I don't see why this would not work. It would not work if you reduce a machine to its body, but the divergence between Bp and Bp & p disallow such a reduction.


What quality does this modality have? Is it shiny? Is it spicy? Does it get tired?

The modality has no quality. It describes qualities. You might do a confusion of level. It is a bit like the confusion between the string "s(s(0))" and the number s(s(0)).




You might think I just described it, by Bp & p, or by "provable(p) and  
true(p)", but this is not the case, as I use some of your intuition  
about truth, which cannot be arithmetized by itself, by a famous  
result of G锟斤拷del and Tarski (independently).
It happens that we do have a good intuition of many truth, and machine  
can indeed describe better and better approximations of the truth  
concept, but the limit of it, used here, cannot be. So by using both  
the comp hypothesis, and by studying simple (L锟斤拷bian) machine (simpler  
than us) we can develop a formal (quantitative in some sense, at some  
level, from some point of view) theory concerning the non formal, and  
even non-formalizable-at-all-by-the-machine, qualities that machine  
can still refer about. And this can be used to explain why machine are  
forced to be befuddled by the subjectively-real apparent gap between  
third and first person attributes.

It seems like you are missing the obvious. Awareness is not just about knowing and navigating a set of logical conditions. That can be accomplished easily without any awareness. Experienced qualities are orthogonal to knowledge and procedural evaluation.

I can understand that "navigating a set of logical conditions" can be done without awareness. Awareness or knowlegde arrives when the navigating embed the navigator in truth, or in a reality. There is a fixed point, and it is explained why this is felt as a personal non communicable experience.


 


> Let the ignoring and insulting begin!
>

We don't need that here, I think, nor anywhere. An insult is almost  
always equivalent with "I have no argument".

I agree.

I know. You are wise. And we agree on many things, but we are working in antipodal theories. I think you have reductionist conception of machine, to be franc.

Bruno




On Friday, August 17, 2012 10:48:04 AM UTC-4, Bruno Marchal wrote:
Hi Craig,

On 15 Aug 2012, at 11:21, Craig Weinberg wrote:

> in case the special characters don't come out...
>
> I was thinking about your primitive of arithmetic truth (numbers, 0,  
> +, and *, right?) and then your concept of 锟斤拷the dreams of  
> numbers锟斤拷, interviewing Lobian Machines, etc and came up with this.
>
> One single irreducible digit 锟斤拷 (Om) which represents a self-
> dividing continuum of infinite perpendicular dialectics between  
> eidetic dream states (in which dream~numbers escape their numerical  
> identities as immersive qualitative experiences) and entopic non-
> dream states (in which number~dreams escape their dream nature as  
> literal algebra-geometries).
>
I use such term more literally. I am not sure I can understand this,  
even if there is some genuine analogy.

The dreaming number are usually very big concrete number. They dream  
by encoding computational state of person, relatively to some  
universal number, which are encoding universal machine relatively to  
some other one, and the initial one can be chosen arbitrary. Those are  
not symbolic number, but real encoding number, a bit like the genome  
if you want.



>
> This continuum f (锟斤拷(Om)), runs from infinitely solipsistic/private  
> first person subjectivity (calling that Aleph 锟斤拷)to infinitely  
> discrete/public third person mechanism (calling that Omega 锟斤拷), so  
> that at 锟斤拷,any given dream is experienced as 99.99锟斤拷9% dream and  
> 0.00锟斤拷1% number and at 锟斤拷 (Omega), any given machine or number is  
> presented as 99.99锟斤拷9% number and 0.00锟斤拷1% dream.
>

?
>
> The halfway point between the 锟斤拷 (Aleph) and 锟斤拷 (Omega) axis is the  
> perpendicular axis f (-锟斤拷(Om)) which is the high and low  
> correspondence between the literal dream and figurative number (or  
> figurative dream and literal number depending on whether you are  
> using the dream-facing epistemology or the number-facing  
> epistemology). This axis runs from tight equivalence (锟斤拷=锟斤拷  
> equality) to broadly elliptical potential set membership (锟斤拷锟斤拷锟斤拷  
> ellipsis)
>
> So it looks something like this:
>
> f(锟斤拷) 锟斤拷 {锟斤拷 锟斤拷锟斤拷锟斤拷 锟斤拷 锟斤拷=锟斤拷 锟斤拷}
>
> function (Om) is superset or equal to the continuum ranging from  
> Aleph to ellipsis perpendicular/orthogonal to the inverse range from  
> equality to Omega).
>
> To go further, it could be said that at 锟斤拷(Omega), 锟斤拷 (Om) expresses  
> as 10|O (one, zero, line segment, circle referring to the  
> quantitative algebraic and geometric perpendicular primitives) while  
> at 锟斤拷 (Aleph), 锟斤拷 (Om) expresses as
> 锟斤拷锟斤拷锟斤拷锟斤拷 (tetragrammaton or yod, hay, vov, hay, or in perhaps more  
> familiar metaphor, 锟斤拷锟斤拷锟斤拷锟斤拷(clubs, spades, hearts, diamonds)
>
> where:
>
> 锟斤拷 clubs (wands) =Fire, spiritual, tactile
> 锟斤拷 spades (swords) = Air, mental, auditory
> 锟斤拷 hearts (cups) =Water, emotional, visual
> 锟斤拷 diamonds (pentacles/coins) = Earth, physical, olfactory-gustatory
>
> Note that tactile and auditory modalities tune us into ourselves and  
> each others sensemaking (selves and minds), while the visual and  
> olfactory/gustatory sense modalities are about objectifying realism  
> of the world (egos or objectified selves/self-images and bodies). It  
> should be obvious that 锟斤拷 clubs (wands) and 锟斤拷 spades (swords) are  
> stereotypically masculine and abstracting forces, while 锟斤拷 hearts  
> (cups) and 锟斤拷 diamonds (pentacles/coins) are stereotypically  
> feminine objectified fields.
>
> Sorry for the mumbo jumbo, but it is the only way to be non-
> reductive when approaching the qualitative side.
>
I don't think so. Aristotle invented modal logic to treat in the  
quantitative way non reductive qualitative notion.



> We can锟斤拷t pretend to talk about the eidetic, dream like  
> perpendicular of number logic while using the purely empirical terms  
> of arithmetic reduction. We need symbols that can only refer to  
> named qualities rather than enumerated quantities.
>
This is exactly what happen when you define the first person by the  
knower. Bp & p, or if you prefer

provable(p) and true(p),

gives a modality which can provably be shown qualitative, and non  
formalizable in arithmetic. It leads to a logic (know as S4Grz) which  
describes something which is absolutely impossible to reduce to any  
number relations or even anything third person describable notion,  
even infinite one.

You might think I just described it, by Bp & p, or by "provable(p) and  
true(p)", but this is not the case, as I use some of your intuition  
about truth, which cannot be arithmetized by itself, by a famous  
result of G锟斤拷del and Tarski (independently).
It happens that we do have a good intuition of many truth, and machine  
can indeed describe better and better approximations of the truth  
concept, but the limit of it, used here, cannot be. So by using both  
the comp hypothesis, and by studying simple (L锟斤拷bian) machine (simpler  
than us) we can develop a formal (quantitative in some sense, at some  
level, from some point of view) theory concerning the non formal, and  
even non-formalizable-at-all-by-the-machine, qualities that machine  
can still refer about. And this can be used to explain why machine are  
forced to be befuddled by the subjectively-real apparent gap between  
third and first person attributes.


> Let the ignoring and insulting begin!
>

We don't need that here, I think, nor anywhere. An insult is almost  
always equivalent with "I have no argument".

Bruno


http://iridia.ulb.ac.be/~marchal/




--
You received this message because you are subscribed to the Google Groups "Everything List" group.
To view this discussion on the web visit https://groups.google.com/d/msg/everything-list/-/wpi1rDQ5fT8J.
To post to this group, send email to everyth...@googlegroups.com.
To unsubscribe from this group, send email to everything-li...@googlegroups.com.
For more options, visit this group at http://groups.google.com/group/everything-list?hl=en.

--
You received this message because you are subscribed to the Google Groups "Everything List" group.
To post to this group, send email to everyth...@googlegroups.com.
To unsubscribe from this group, send email to everything-li...@googlegroups.com.
For more options, visit this group at http://groups.google.com/group/everything-list?hl=en.

Roger

unread,
Aug 18, 2012, 11:47:11 AM8/18/12
to everything-list
Hi John Clark
 
Thanks for that. I guess that the various approximations are
supposedly faster ways to get to that value.  Thanks again.
 
 
 
 
8/18/2012
Leibniz would say, "If there's no God, we'd have to invent him so everything could function."
----- Receiving the following content -----
From: John Clark
Receiver: everything-list
Time: 2012-08-18, 11:01:39
Subject: Re: On calculating pi

On Sat, Aug 18, 2012 at 9:50 AM, Roger <rcl...@verizon.net> wrote:

>how can they know if the calculation of pi is very precise if they
don't really know beforehand what its precise value should be ?

But we do know the precise value of pi, 250 years ago Euler proved that pi squared divided by 6 is EXACTLY equal to � the infinite series 1+ 1/2 + 1/4 + 1/9 + 1/16 + 1/25 + 1/36 +1/49 +.......锟斤拷 Tell me how close to pi you want to get and a finite number of terms in this sequence can help you produce it.

� John K Clark

--
You received this message because you are subscribed to the Google Groups "Everything List" group.

Craig Weinberg

unread,
Aug 18, 2012, 12:15:46 PM8/18/12
to everyth...@googlegroups.com


On Saturday, August 18, 2012 5:59:32 AM UTC-4, Bruno Marchal wrote:

On 17 Aug 2012, at 19:15, Craig Weinberg wrote:



On Friday, August 17, 2012 10:48:04 AM UTC-4, Bruno Marchal wrote:
Hi Craig,

On 15 Aug 2012, at 11:21, Craig Weinberg wrote:

> in case the special characters don't come out...
>
> I was thinking about your primitive of arithmetic truth (numbers, 0,  
> +, and *, right?) and then your concept of ‘the dreams of  
> numbers’, interviewing Lobian Machines, etc and came up with this.
>
> One single irreducible digit ॐ (Om) which represents a self-
> dividing continuum of infinite perpendicular dialectics between  
> eidetic dream states (in which dream~numbers escape their numerical  
> identities as immersive qualitative experiences) and entopic non-
> dream states (in which number~dreams escape their dream nature as  
> literal algebra-geometries).
>
I use such term more literally. I am not sure I can understand this,  
even if there is some genuine analogy.

Think of it like π, except that instead of circumference and diameter, there is eidetic-figurative and entopic-literal presentation modalities.

Pi = ratio of the length or a circle and its diameter. That is understandable.
"eidetic-figurative and entropic-literal presentation modalities." is not.

If it isn't understand it, how do you explain that I understand it? Straightness and circularity are two presentations with very narrow, precise, and literal characteristics. They can be said to oppose each other in one sense and be mutually exclusive forms, but the larger the circle, the straighter the curve of it's circumference (I know you have a better way of saying that). All possible geometric forms and topologies however would be categorized as 'entopic-literal' forms, as they are shapes which refer only to themselves. Entopic hallucinations are those which are repeating geometries, escalating into full blown ecstasies of Platonic psychedelia.

The other major category of hallucination (and therefore nakedly exposed consciousness) is eidetic - faces in the trees, butterflies in the inkblot, etc. Instead of referring literally to a shape of ink on paper or light and dark regions of bark, they present characters or creatures, mythical themes, etc. This is, like linearity is to circularity, the opposite potential of awareness - one that simplifies a complex figurative identity into a gestalt or algebra (algebra in the etymological sense of a "reunion of broken parts") rather than a topological form that is literal. This ratio, between poetic, dreamlike involvement and literal, mathematical observation I am saying forms the grand continuum of the cosmos.

 






The dreaming number are usually very big concrete number. They dream  
by encoding computational state of person, relatively to some  
universal number, which are encoding universal machine relatively to  
some other one, and the initial one can be chosen arbitrary. Those are  
not symbolic number, but real encoding number, a bit like the genome  
if you want.

Why would that result in a dream?

Because I work in the comp theory where we come to the idea that consciousness can be manifested by abstract relation between numbers, as they emulate computation. We have already said "yes" to the doctor.


You don't really 'come to the idea' at all though, you assume it from the start. There is no theory for why or how numbers would dream, only the assumption that they do.
 
 


It seems shrouded in obfuscating self-reference. Why would anything that has been encoded ever need to be decoded if the machine can fluently process the encoded form?

To store what we learn. The DNA plays already such a role at the molecular level. It illustrates also a digital encoding and decoding. 

Why would storage imply any kind of encoding? Without physics to constrain resource requirements, there would be no need to compress information. I can't think of any reason that a Turing machine would need to create an abstraction layer, especially if you could control the speed of the tape. If for any reason you needed subroutines to synchronize, you could run those parts of the tape faster or slower - infinitely fast if you like. Without physics, time is relative and unbounded. A Turing machine needs no programming language, it doesn't even need bytes. You can do all of your addressing on the fly with dynamically sized memory chunks. All of these formalisms, codes, architectures, etc are purely in the service of the limitations imposed by physics and human convenience. We get tired of writing a thousand lines of code every time we want to transform some dataset, but the computer doesn't care. It will run a trillion lines of code just to add the same number to itself, it doesn't care how long it takes or how wasteful it seems to us.


Why would it need any other form - especially if it is all made of numbers?

Nobody needs a universe. Why do we do babies?
The "truth" is that we have them, we cannot really avoid them. It is like the prime number and the universal machine. Once you have zero and + and *, you get Platonia, and a lot of mess in Platonia. It is a logical consequence.

But a universe is not a consequence of Platonia. Since we know that we have a universe that means that it is Platonia which needs to be justified intellectually in terms of contributing to the universe, not the other way around.
 



What I am saying is that if you are going to invoke a possibility of dreams, that has to be grounded in the terms that you are laying out as primitive. Why would dreams leap out of mechanical relations?

It is a logical consequence, once you accept the idea that you might survive with a digital brain.

It's the logic of begging the question. I ask you why numbers dream and you say it's logical if you accept that dreams can survive as digital process. I don't though. If human dreams could exist in something other than humans, then they would already. On the sands of the beaches, in the odd complicated cloud formation. I would need a compelling reason to believe otherwise. Why would I give the benefit of the doubt to machines when machines have clearly shown no signs of feeling thus far? Not just that they might not have feelings, but that they are universally known for lacking any capacity for feeling whatsoever - that is what defines the nature of machines. When you understand the continuum of eidetic to entopic, figurative to literal, you will understand why this makes perfect sense. A machine is an assembly of logical forms. That does not produce any phenomenology by itself because it is constructed from the outside in. Living organisms build themselves from the inside out, from their own native sensitivities and motivations. They are not having an alien script intentionally imposed upon their behavior.
 



Even if there was some purpose for it, how could that actually take place - what are the dreamings made of?

Ontologically: nothing take place. All the computations are there. Some emulate self-observing machine and the math explain why they have to be beffudled by existence and conscience.


That may be, but why in the world would computational befuddlement be expressed as personality and realism?



My view is that it may be the case that everything that is not matter across space is experience through time - by definition, ontologically. There is no other form or content possible in the cosmos. Numbers are experiences as they must be inferred by computational agents and cannot exist independently of them. What my formulas do is to propose a precise relation between dream-time (including logical algebras) and matter-space (including topological geometries). To do this we need to invoke a continuity between them which is a perpendicular axis which runs from the literal (tight equivalence; induction is accomplished through linear arithmetic logic) to the figurative/metaphorical (loose thematic association; induction is accomplished through linear logic as well as elliptical cross-context leaps).

I don't believe in time, space, cosmos, matter, ...
I explain their appearance by the dream property of numbers, relatively to universal numbers.


That's what I am showing you, is how the dream property of numbers (eidetic-figurative) and the number property of dreams (entopic-literal) are two polar regions of the same (involuted) continuum.







>
> This continuum f (ॐ(Om)), runs from infinitely solipsistic/private  
> first person subjectivity (calling that Aleph ℵ)to infinitely  
> discrete/public third person mechanism (calling that Omega Ω), so  
> that at ℵ,any given dream is experienced as 99.99…9% dream and  
> 0.00…1% number and at Ω (Omega), any given machine or number is  
> presented as 99.99…9% number and 0.00…1% dream.
>

?


I'm mapping out this literal to figurative axis, as it modifies the axis of subject to object presentations. The more an experience extends figuratively/metaphorically, the less it extends literally/mechanically.

That makes some sense.


That's what I'm saying. Quanta is qualia that has been flattened until it spreads out like with a rolling pin to the point of universality. Qualia is the dough that has no extension into public space.
 

Ok, but we have to connect that irreducible quality with quantity, and I think we can do that with this symmetrical continuum of literal figures and figurative literacy.

Modal logic can make a toy model of experience, by removing all of the experience and replacing it with a meaningless variable.

Why meaningless?

Because that's what variables do, they generalize all potential content-meanings under single quantifiable term.
 



It is to say 'here, instead of your baby, let's just call it 'consumer of diapers and milk' and design a nursery based on the statistics derived from that consumption.

No, it is the complete contrary. I see your point, but it is eaxtly that type of reduction that is prevented by comp. Even for machine, we can no more reduce them to their third person description. They do have a soul, even after-life, etc.
 
If machines have souls, then there has to be a way that the two are connected. There has to be some kind of rationale for it.


What is needed is a way to reference phenomenological qualities which expresses not merely non-comp, but explicitly asserts quality and a view of the universe from the perspective of irreducible quality.

You have to postualte them, and to postulate matter, and to postulate a relation, and non-comp, and that is neither satisfying, nor working. It introduces difficulties where there are already enough, imo.

I don't think you have to postulate matter, you only have to formalize what you already assume about dreaming numbers.
 




> We can’t pretend to talk about the eidetic, dream like  
> perpendicular of number logic while using the purely empirical terms  
> of arithmetic reduction. We need symbols that can only refer to  
> named qualities rather than enumerated quantities.
>
This is exactly what happen when you define the first person by the  
knower. Bp & p, or if you prefer

provable(p) and true(p),

provable and true are not first person qualities, they are epistemological quantifications. First person would be more like great(p) and superior(p).

?

A square peg either fits in a particular sized round hole or not. That is a true/false. It can be proved by actually trying to fit the peg in the hole. Those are third person public conditions. Whether one peg or hole seems 'better' than another is a first person kind of distinction.
 




gives a modality which can provably be shown qualitative, and non  
formalizable in arithmetic. It leads to a logic (know as S4Grz) which  
describes something which is absolutely impossible to reduce to any  
number relations or even anything third person describable notion,  
even infinite one.

That doesn't mean it is qualitative, only that it is so obscurely arithmetic that arithmetic itself cannot quantify it.

But the machine still can refer to it. So I don't see why this would not work. It would not work if you reduce a machine to its body, but the divergence between Bp and Bp & p disallow such a reduction.

I don't really understand what modal logic has to do with the possibility of quality.
 


What quality does this modality have? Is it shiny? Is it spicy? Does it get tired?

The modality has no quality. It describes qualities. You might do a confusion of level. It is a bit like the confusion between the string "s(s(0))" and the number s(s(0)).

 
How is it describing qualities? It seems to me to be describing quantitative formalism. How does one describe red logically?




You might think I just described it, by Bp & p, or by "provable(p) and  
true(p)", but this is not the case, as I use some of your intuition  
about truth, which cannot be arithmetized by itself, by a famous  
result of Gödel and Tarski (independently).
It happens that we do have a good intuition of many truth, and machine  
can indeed describe better and better approximations of the truth  
concept, but the limit of it, used here, cannot be. So by using both  
the comp hypothesis, and by studying simple (Löbian) machine (simpler  
than us) we can develop a formal (quantitative in some sense, at some  
level, from some point of view) theory concerning the non formal, and  
even non-formalizable-at-all-by-the-machine, qualities that machine  
can still refer about. And this can be used to explain why machine are  
forced to be befuddled by the subjectively-real apparent gap between  
third and first person attributes.

It seems like you are missing the obvious. Awareness is not just about knowing and navigating a set of logical conditions. That can be accomplished easily without any awareness. Experienced qualities are orthogonal to knowledge and procedural evaluation.

I can understand that "navigating a set of logical conditions" can be done without awareness. Awareness or knowlegde arrives when the navigating embed the navigator in truth, or in a reality. There is a fixed point, and it is explained why this is felt as a personal non communicable experience.

The fact that there would be data that cannot be communicated from a fixed vector doesn't imply to me anything specifically personal or experiential. What is relevant about consciousness is not only *that* it is private, but what the content of that privacy is. The form of it is trivial, except that it helps us understand the relation to exterior realism. Using modal logic to describe experience makes a formula out of the shape of a can and presents it as a way of creating canned food.
 

 


> Let the ignoring and insulting begin!
>

We don't need that here, I think, nor anywhere. An insult is almost  
always equivalent with "I have no argument".

I agree.

I know. You are wise. And we agree on many things, but we are working in antipodal theories. I think you have reductionist conception of machine, to be franc.

You are wise too. I think that your view of my view of machines is more of a projection than you think. While I admit, certainly, to being ignorant of the particular details of mathematical principles which give you a more generous view of machine consciousness than I have, I don't limit the capabilities of machines in the way that you assume. If quanta is flat qualia, then machine intelligence can extend infinitely farther than organic intelligence, because of the horizontal universality of it. What I am saying is that you can't build qualia out of quanta, because there is no universal qualia. Qualia is othrogonal to quanta - it is the manifestation of non-universality itself.

Craig

 

Alberto G. Corona

unread,
Aug 18, 2012, 4:55:53 PM8/18/12
to everyth...@googlegroups.com
I can not resit to say something here.

1)Adivination may be very dangerous, because adivination can be a powerful way of manipulation. an autoproclaimed adivine can manipulate you or your society if the the adivine is a powerful person.  It can gain the a status of living god. It can even be a philosopher or a scientist.

2) very related with this, it is very plausible that by natural selection applied to the need to coordinate societies for common goals, we have the capability and the unavoidable neccesity, by instinct to deify something or someone and to hypostasize it, that is to give it a personal nature it it has not. this may derive from the cult to the authority of the founder of the ancient tribe of our ancestors. 

 If we have this instinct and this is unavoidable, the best use of it, to avoid the manipulation of  those who want to ascend  in the mind of the people, to what was in the past reserved for the gods,  is to adore a transcendent personal god that represent the unknown. T

There is no theology here. I´m, talking in practical terms. although it may be considered of what Saint Thomas would call "natural revelation". I´m fascinated o how much specifically Christian apologetics can be derived from the apparently antireligious, simple and egoistic notion of natural selection, which is none of the three. But this is not the place to discuss that.

2012/8/17 Roger <rcl...@verizon.net>

Alberto G. Corona

unread,
Aug 18, 2012, 5:04:28 PM8/18/12
to everyth...@googlegroups.com
I can not resit to say something here.

1)Adivination may be very dangerous, because adivination can be a powerful way of manipulation. an autoproclaimed adivine can manipulate you or your society if the the adivine is a powerful person.  It can gain the a status of living god. In the past they were adivines or magicians, later they were the philosophers. Nowadays they are mostly scientists. I´m talking about the bad people of these groups.

2) very related with this, it is very plausible that by natural selection applied to the need to coordinate societies for common goals, we have the capability and the unavoidable necesity, by instinct to deify something or someone and to hypostasize it, that is to give it a personal nature it it has not. this may derive from the cult to the authority of the founder of the ancient tribe of our ancestors. 

 If we have this instinct and this is unavoidable, the best use of it, to avoid the manipulation of  those who want to ascend  in the mind of the people, to what was in the past reserved for the gods,  is to adore a transcendent personal god that represent the unknown. 

There is no theology here. I´m, talking in practical terms. although it may be considered of what Saint Thomas would call "natural revelation". I´m fascinated o how much specifically Christian apologetics can be derived from the apparently antireligious, simple and egoistic notion of natural selection, which is none of the three. But this is not the place to discuss that.

2012/8/17 Roger <rcl...@verizon.net>
Hi Craig Weinberg

Craig Weinberg

unread,
Aug 18, 2012, 10:37:11 PM8/18/12
to everyth...@googlegroups.com


On Saturday, August 18, 2012 5:04:28 PM UTC-4, Alberto G.Corona wrote:
I can not resit to say something here.

1)Adivination may be very dangerous, because adivination can be a powerful way of manipulation. an autoproclaimed adivine can manipulate you or your society if the the adivine is a powerful person.  It can gain the a status of living god. In the past they were adivines or magicians, later they were the philosophers. Nowadays they are mostly scientists. I´m talking about the bad people of these groups.

Sure, but doesn't the abuse of adivination pale in comparison to most other forms of political device? Has the Bible or Koran every been used as a powerful way to manipulate societies? Has financial power ever giving someone the status of a living god? To me, especially in a modern context, using divination is seen as a huge liability. The Wall Street wizards of quant magic are far more influential than any Bronze Age warlock ever dreamed of being.


2) very related with this, it is very plausible that by natural selection applied to the need to coordinate societies for common goals, we have the capability and the unavoidable necesity, by instinct to deify something or someone and to hypostasize it, that is to give it a personal nature it it has not. this may derive from the cult to the authority of the founder of the ancient tribe of our ancestors. 

 If we have this instinct and this is unavoidable, the best use of it, to avoid the manipulation of  those who want to ascend  in the mind of the people, to what was in the past reserved for the gods,  is to adore a transcendent personal god that represent the unknown. 

There is no theology here. I´m, talking in practical terms. although it may be considered of what Saint Thomas would call "natural revelation". I´m fascinated o how much specifically Christian apologetics can be derived from the apparently antireligious, simple and egoistic notion of natural selection, which is none of the three. But this is not the place to discuss that.


I think that our contemporary culture shows that society can bond to abstract conceptual brands just as well as an anthropomorphized personality. Our Gods are commercial abstractions of status. Any fears of charismatic religious power in the West are probably hysterical exaggerations at this point. Relatively few people care about someone claiming to speak for an omniscient God anymore - it's who speaks for financial success that matters.

Craig

Alberto G. Corona

unread,
Aug 19, 2012, 5:15:08 AM8/19/12
to everyth...@googlegroups.com
The barrier between religion and ordinary life, like the one that suppossedly exist between gods and ordinary life is conventiona. If it is true that men have an instinct for religion, this is not governed by a switch that is put on when in a temple or when it is reading esoteric teachings. It is on all the time and in everyone.

What produces this need of the soul or this innate instinct of the human nature?. It may produce organized relgion, but also politics and ideology.  The brain areas excited by the appearance of the Pope in a group of believers are the same that are excited in ecologists when Al Gore appears. In the past there were no separation between both phenomena. This is an mostly Occidental division. The cult of personality in socialist countries and the sectarian movements (either political or religious) are new editions of the fundamentally Unitarian nature of religion and politics.

So, then, gods and adivines have been and will be here forever. When a name for them is discredited, they appear with new names and within new organization.  The modern Global warming alarmism is an  episode of adivination by makin illegitimate use of science. the Marxism was a scholastic school of Masters of Reality that claimed predicitive powers over the story of Humanity. The gigantic photographs of Marx Lenin in the URSS parliament is an example of religious temple of Atheism. But also the small photograph or a loving one in the dormitory carries out a religious sense, Specially if it passed away and it was a greath influence in our lives. Religion is everywhere and forever.

2012/8/18 Alberto G. Corona <agoc...@gmail.com>

Bruno Marchal

unread,
Aug 19, 2012, 5:59:02 AM8/19/12
to everyth...@googlegroups.com
On 18 Aug 2012, at 17:19, Roger wrote:

Hi Bruno Marchal
 
Yes, you can square the square root of any number to test its accuracy,
but there are a variety of algorithms used to calculate pi.
 
Which is correct ? See
 
 
 
The value obtained is assumed to be true if the infinite series
used to calculate pi converges. But I would think that 
many if not most infinite series should
converge. Which one is the right one ? Is there a unique solution ?

Most series would not converge. In this case they all converge to Pi, as they have been designed for that. Some just converge more quickly than others.

Bruno



Bruno Marchal

unread,
Aug 19, 2012, 7:57:32 AM8/19/12
to everyth...@googlegroups.com
On 18 Aug 2012, at 18:15, Craig Weinberg wrote:



On Saturday, August 18, 2012 5:59:32 AM UTC-4, Bruno Marchal wrote:

On 17 Aug 2012, at 19:15, Craig Weinberg wrote:



On Friday, August 17, 2012 10:48:04 AM UTC-4, Bruno Marchal wrote:
Hi Craig,

On 15 Aug 2012, at 11:21, Craig Weinberg wrote:

> in case the special characters don't come out...
>
> I was thinking about your primitive of arithmetic truth (numbers, 0,  
> +, and *, right?) and then your concept of ‘the dreams of  
> numbers’, interviewing Lobian Machines, etc and came up with this.
>
> One single irreducible digit ॐ (Om) which represents a self-
> dividing continuum of infinite perpendicular dialectics between  
> eidetic dream states (in which dream~numbers escape their numerical  
> identities as immersive qualitative experiences) and entopic non-
> dream states (in which number~dreams escape their dream nature as  
> literal algebra-geometries).
>
I use such term more literally. I am not sure I can understand this,  
even if there is some genuine analogy.

Think of it like π, except that instead of circumference and diameter, there is eidetic-figurative and entopic-literal presentation modalities.

Pi = ratio of the length or a circle and its diameter. That is understandable.
"eidetic-figurative and entropic-literal presentation modalities." is not.

If it isn't understand it, how do you explain that I understand it?

I understand that 2+2 = 4.
I still cannot explain how and why I understand "2+2 = 4".
"2+2=4" is easy.
"I understand 2+2=4" is quasi infinitely more complex.





Straightness and circularity are two presentations with very narrow, precise, and literal characteristics. They can be said to oppose each other in one sense and be mutually exclusive forms, but the larger the circle, the straighter the curve of it's circumference (I know you have a better way of saying that). All possible geometric forms and topologies however would be categorized as 'entopic-literal' forms, as they are shapes which refer only to themselves. Entopic hallucinations are those which are repeating geometries, escalating into full blown ecstasies of Platonic psychedelia.

The other major category of hallucination (and therefore nakedly exposed consciousness) is eidetic - faces in the trees, butterflies in the inkblot, etc. Instead of referring literally to a shape of ink on paper or light and dark regions of bark, they present characters or creatures, mythical themes, etc. This is, like linearity is to circularity, the opposite potential of awareness - one that simplifies a complex figurative identity into a gestalt or algebra (algebra in the etymological sense of a "reunion of broken parts") rather than a topological form that is literal. This ratio, between poetic, dreamlike involvement and literal, mathematical observation I am saying forms the grand continuum of the cosmos.

Hmm...




 






The dreaming number are usually very big concrete number. They dream  
by encoding computational state of person, relatively to some  
universal number, which are encoding universal machine relatively to  
some other one, and the initial one can be chosen arbitrary. Those are  
not symbolic number, but real encoding number, a bit like the genome  
if you want.

Why would that result in a dream?

Because I work in the comp theory where we come to the idea that consciousness can be manifested by abstract relation between numbers, as they emulate computation. We have already said "yes" to the doctor.


You don't really 'come to the idea' at all though, you assume it from the start. There is no theory for why or how numbers would dream, only the assumption that they do.


Here I absolutely disagree. The theory is that I am a material machine. The conclusion is that matter is an hallucination, yet a lawful one. It looks like you have not yet take a deep look on UDA.



 
 


It seems shrouded in obfuscating self-reference. Why would anything that has been encoded ever need to be decoded if the machine can fluently process the encoded form?

To store what we learn. The DNA plays already such a role at the molecular level. It illustrates also a digital encoding and decoding. 

Why would storage imply any kind of encoding? Without physics to constrain resource requirements, there would be no need to compress information.

The numbers do that all the time. There is no need, but they cannot not doing it, because it follows from addition and multiplication. There is no need for Saturn's ring either.



I can't think of any reason that a Turing machine would need to create an abstraction layer, especially if you could control the speed of the tape. If for any reason you needed subroutines to synchronize, you could run those parts of the tape faster or slower - infinitely fast if you like. Without physics, time is relative and unbounded. A Turing machine needs no programming language, it doesn't even need bytes. You can do all of your addressing on the fly with dynamically sized memory chunks. All of these formalisms, codes, architectures, etc are purely in the service of the limitations imposed by physics and human convenience.

Not if you bet that we are machine. In that cse the UD reasoning shows that what you describe is only the appernt view from inside, but that eventually the physical resource constraints arise from the mathematical computer science constraints, that is, from arithmetic.



We get tired of writing a thousand lines of code every time we want to transform some dataset, but the computer doesn't care. It will run a trillion lines of code just to add the same number to itself, it doesn't care how long it takes or how wasteful it seems to us.

Not with comp, as I am a person which manifest itself through a computer, and persons care.






Why would it need any other form - especially if it is all made of numbers?

Nobody needs a universe. Why do we do babies?
The "truth" is that we have them, we cannot really avoid them. It is like the prime number and the universal machine. Once you have zero and + and *, you get Platonia, and a lot of mess in Platonia. It is a logical consequence.

But a universe is not a consequence of Platonia.

It is. Or your non-comp assumption is correct, but I do not work in the theory non-comp. Actually, I have not yet seen a non-comp *theory*. Only philosophical argument against comp, but no concrete replacement.


Since we know that we have a universe

We know only that we are conscious. How could we know that there is a (physical) universe? 
We don't even *know* that there is a mathematical universe, or even an arithmetical universe. We bet on it.





that means that it is Platonia which needs to be justified intellectually in terms of contributing to the universe, not the other way around.

But with comp, we do got an explanation of the physical universe(s). And we got both quanta and qualia. It might be false, but the explanation is there.



 



What I am saying is that if you are going to invoke a possibility of dreams, that has to be grounded in the terms that you are laying out as primitive. Why would dreams leap out of mechanical relations?

It is a logical consequence, once you accept the idea that you might survive with a digital brain.

It's the logic of begging the question. I ask you why numbers dream and you say it's logical if you accept that dreams can survive as digital process. I don't though. If human dreams could exist in something other than humans, then they would already.

And that is the case already, indeed. That follows from the reasoning.



On the sands of the beaches, in the odd complicated cloud formation. I would need a compelling reason to believe otherwise. Why would I give the benefit of the doubt to machines when machines have clearly shown no signs of feeling thus far?

But molecular biology, quantum mechanics, etc. suggest strongly that we are machine. The rest is the logical consequence. There is no evidence for non-comp. So we do have compelling reason to think that machine can have feeling. The conjunction of our own experience, and the study of our bodies.

But I don't want to defend comp. That's the kind of philosophy I do not work on. I just show the consequence of being material machine. The consequences are that we are already in a "matrix", whose structure can be mathematically handled, so that we can constructively derive physics, and compare it to the empirical physics, so that comp is shown testable. (To be short).


Not just that they might not have feelings, but that they are universally known for lacking any capacity for feeling whatsoever - that is what defines the nature of machines. When you understand the continuum of eidetic to entopic, figurative to literal, you will understand why this makes perfect sense

But what you say makes sense. The comp theory already explains why comp is hard (impossible, even)  to believe, and why it has to be highly counter-intuitive. 


A machine is an assembly of logical forms. That does not produce any phenomenology by itself because it is constructed from the outside in. Living organisms build themselves from the inside out, from their own native sensitivities and motivations. They are not having an alien script intentionally imposed upon their behavior.

I don't know that. Genetic illustrates that nature do scripts. But math shows also that very short scripts, like "help yourself" can lead to tremendous richness and complexity. 



 



Even if there was some purpose for it, how could that actually take place - what are the dreamings made of?

Ontologically: nothing take place. All the computations are there. Some emulate self-observing machine and the math explain why they have to be beffudled by existence and conscience.


That may be, but why in the world would computational befuddlement be expressed as personality and realism?

Because there is, very roughly, 60000^10000000000 brain states possible, that makes a lot of subjectivity possible, and they are multiplied and selected through a filtering on a continuum of computations (first person indeterminacy). 






My view is that it may be the case that everything that is not matter across space is experience through time - by definition, ontologically. There is no other form or content possible in the cosmos. Numbers are experiences as they must be inferred by computational agents and cannot exist independently of them. What my formulas do is to propose a precise relation between dream-time (including logical algebras) and matter-space (including topological geometries). To do this we need to invoke a continuity between them which is a perpendicular axis which runs from the literal (tight equivalence; induction is accomplished through linear arithmetic logic) to the figurative/metaphorical (loose thematic association; induction is accomplished through linear logic as well as elliptical cross-context leaps).

I don't believe in time, space, cosmos, matter, ...
I explain their appearance by the dream property of numbers, relatively to universal numbers.


That's what I am showing you, is how the dream property of numbers (eidetic-figurative) and the number property of dreams (entopic-literal) are two polar regions of the same (involuted) continuum.

I am a simple minded logician. If you use a technical term, you have to explain it to me in a precise frame theory.
I don't ask you to formalize it in first order logic, but to succeed in explaining this enough precisely so that a poor logician like can understand that with time and patience we can get such a formalization. If not it looks like you can say anything you want, and very often, it makes sense in machine first person terms, and thus fail to convey your non-comp intuition more than the natural non-comp intuition of *all* correct machine.
Comp shows intuition locally correct, locally useful, locally unavoidable, but globally wrong.










>
> This continuum f (ॐ(Om)), runs from infinitely solipsistic/private  
> first person subjectivity (calling that Aleph ℵ)to infinitely  
> discrete/public third person mechanism (calling that Omega Ω), so  
> that at ℵ,any given dream is experienced as 99.99…9% dream and  
> 0.00…1% number and at Ω (Omega), any given machine or number is  
> presented as 99.99…9% number and 0.00…1% dream.
>

?


I'm mapping out this literal to figurative axis, as it modifies the axis of subject to object presentations. The more an experience extends figuratively/metaphorically, the less it extends literally/mechanically.

That makes some sense.


That's what I'm saying. Quanta is qualia that has been flattened until it spreads out like with a rolling pin to the point of universality. Qualia is the dough that has no extension into public space.

OK.
Perhaps. But you should do it, then, or convince others to do it.




Modal logic can make a toy model of experience, by removing all of the experience and replacing it with a meaningless variable.

Why meaningless?

Because that's what variables do, they generalize all potential content-meanings under single quantifiable term.

Hmm... OK. But the person is in the box (B), not in the variable, which is usually used for some unspecified arithmetic proposition, or memories content, etc.


 



It is to say 'here, instead of your baby, let's just call it 'consumer of diapers and milk' and design a nursery based on the statistics derived from that consumption.

No, it is the complete contrary. I see your point, but it is eaxtly that type of reduction that is prevented by comp. Even for machine, we can no more reduce them to their third person description. They do have a soul, even after-life, etc.
 
If machines have souls, then there has to be a way that the two are connected. There has to be some kind of rationale for it.

Nice to hear that. I agree. 




What is needed is a way to reference phenomenological qualities which expresses not merely non-comp, but explicitly asserts quality and a view of the universe from the perspective of irreducible quality.

You have to postualte them, and to postulate matter, and to postulate a relation, and non-comp, and that is neither satisfying, nor working. It introduces difficulties where there are already enough, imo.

I don't think you have to postulate matter, you only have to formalize what you already assume about dreaming numbers.

OK. That's the point. In the comp theory.


 




> We can’t pretend to talk about the eidetic, dream like  
> perpendicular of number logic while using the purely empirical terms  
> of arithmetic reduction. We need symbols that can only refer to  
> named qualities rather than enumerated quantities.
>
This is exactly what happen when you define the first person by the  
knower. Bp & p, or if you prefer

provable(p) and true(p),

provable and true are not first person qualities, they are epistemological quantifications. First person would be more like great(p) and superior(p).

?

A square peg either fits in a particular sized round hole or not. That is a true/false. It can be proved by actually trying to fit the peg in the hole. Those are third person public conditions. Whether one peg or hole seems 'better' than another is a first person kind of distinction.
 




gives a modality which can provably be shown qualitative, and non  
formalizable in arithmetic. It leads to a logic (know as S4Grz) which  
describes something which is absolutely impossible to reduce to any  
number relations or even anything third person describable notion,  
even infinite one.

That doesn't mean it is qualitative, only that it is so obscurely arithmetic that arithmetic itself cannot quantify it.

But the machine still can refer to it. So I don't see why this would not work. It would not work if you reduce a machine to its body, but the divergence between Bp and Bp & p disallow such a reduction.

I don't really understand what modal logic has to do with the possibility of quality.

The machine modal logic of self-reference, Bp, when conjuncted with the non nameable Truth, leads the machine to discover true propositions, in a sort of immediate way, which they cannot justify nor even describe, except by projecting them on similar machines. 
The redness quality is like that. I have no doubt that I experience redness, but I am unable to communicate it to a blind rationalist. I can point on something red, and hope my fellow has a sufficiently similar experience, so that the word "red" will have a sufficiently close meaning to mine. 



 


What quality does this modality have? Is it shiny? Is it spicy? Does it get tired?

The modality has no quality. It describes qualities. You might do a confusion of level. It is a bit like the confusion between the string "s(s(0))" and the number s(s(0)).

 
How is it describing qualities? It seems to me to be describing quantitative formalism. How does one describe red logically?

By explaining it exists necessarily and that it cannot be describe logically. Modal logic, in the machine context are "meta". It does not explain the quality red. It explains that quality exists and are NOT describable quantitatively. 

The "universal soul", Bp & p, or the S4Grz logic, is a sort of mathematical tour-de-force: it is a formal logic describing the logic of content which are impossible to formalize ever. But we can formalize this at the metalevel.







You might think I just described it, by Bp & p, or by "provable(p) and  
true(p)", but this is not the case, as I use some of your intuition  
about truth, which cannot be arithmetized by itself, by a famous  
result of Gödel and Tarski (independently).
It happens that we do have a good intuition of many truth, and machine  
can indeed describe better and better approximations of the truth  
concept, but the limit of it, used here, cannot be. So by using both  
the comp hypothesis, and by studying simple (Löbian) machine (simpler  
than us) we can develop a formal (quantitative in some sense, at some  
level, from some point of view) theory concerning the non formal, and  
even non-formalizable-at-all-by-the-machine, qualities that machine  
can still refer about. And this can be used to explain why machine are  
forced to be befuddled by the subjectively-real apparent gap between  
third and first person attributes.

It seems like you are missing the obvious. Awareness is not just about knowing and navigating a set of logical conditions. That can be accomplished easily without any awareness. Experienced qualities are orthogonal to knowledge and procedural evaluation.

I can understand that "navigating a set of logical conditions" can be done without awareness. Awareness or knowlegde arrives when the navigating embed the navigator in truth, or in a reality. There is a fixed point, and it is explained why this is felt as a personal non communicable experience.

The fact that there would be data that cannot be communicated from a fixed vector doesn't imply to me anything specifically personal or experiential. What is relevant about consciousness is not only *that* it is private, but what the content of that privacy is.

Of course.



The form of it is trivial, except that it helps us understand the relation to exterior realism. Using modal logic to describe experience makes a formula out of the shape of a can and presents it as a way of creating canned food.

See above.


 

 


> Let the ignoring and insulting begin!
>

We don't need that here, I think, nor anywhere. An insult is almost  
always equivalent with "I have no argument".

I agree.

I know. You are wise. And we agree on many things, but we are working in antipodal theories. I think you have reductionist conception of machine, to be franc.

You are wise too. I think that your view of my view of machines is more of a projection than you think.

Well, if I remember well, you are the one who refuse to sell a steak to my daugher's husband, who said "yes" to a doctor, and is, by construction, a machine.




While I admit, certainly, to being ignorant of the particular details of mathematical principles which give you a more generous view of machine consciousness than I have, I don't limit the capabilities of machines in the way that you assume. If quanta is flat qualia,

Quanta are more sort of sharable qualia. First person plural construct brought by the many-world internal interpretation of arithmetic by arithmetical beings.



then machine intelligence can extend infinitely farther than organic intelligence, because of the horizontal universality of it.

Yes, indeed.



What I am saying is that you can't build qualia out of quanta,

I agree with this. Qualia emerges logically before quanta, but it seems the contrary. That can perhaps be related to Stephen-Pratt reversal of arrows in the mind body relation, or to the duality between thrid person (Bp) and the Theaetetical first person (Bp & p).



because there is no universal qualia.

This I doubt. More from personal reasons than comp, though.


Qualia is othrogonal to quanta - it is the manifestation of non-universality itself.

Well, with comp, quanta is the tiny emerging part of the qualia-consciousness part of the arithmetical platonia, seen from inside.

May be the crux of our difference (beyond comp/non-comp) is that, as many people, you seem to believe that lind and matter are symmetrical and have same caliber. But comp is still open too full platonism, in which matter, and physical universe are more like a divine bug, a temporary misleading; hding the truth more than paving a way to it. Some platonist identify matter with bad, or the devil. this is of course an open problem with comp, but what can already been argued for is that the mindscape is vastly bigger than the physical scape. The comp mindscape or personscape is vast. *Very* vast. It escapes the math. It violates physicalism, but also mathematicalism, despite the ontology is quite little (a tiny segment of arithmetical truth). 

Bruno



Bruno Marchal

unread,
Aug 19, 2012, 8:26:10 AM8/19/12
to everyth...@googlegroups.com

On 19 Aug 2012, at 11:15, Alberto G. Corona wrote:

> The barrier between religion and ordinary life, like the one that
> suppossedly exist between gods and ordinary life is conventiona. If
> it is true that men have an instinct for religion, this is not
> governed by a switch that is put on when in a temple or when it is
> reading esoteric teachings. It is on all the time and in everyone.

I agree. I make a case that all correct machine are theological. The
reason is that such machine, when looking inward (as they can do by
self-reference) can guess that there is something transcending them.



>
> What produces this need of the soul or this innate instinct of the
> human nature?. It may produce organized relgion, but also politics
> and ideology. The brain areas excited by the appearance of the Pope
> in a group of believers are the same that are excited in ecologists
> when Al Gore appears. In the past there were no separation between
> both phenomena. This is an mostly Occidental division.

But it is also a natural division. When machine get theological, from
their perspective it looks like those kind of things are different.
And at some level they are. I think that the conflict is already
reflected in the left brain / right brain difference. Perhaps between
woman and man, east and west, yin and yang.

Take any machine, she will develop those two poles. the "schizophreny
appears only when one pole believes to be more right than the other
pole.



> The cult of personality in socialist countries and the sectarian
> movements (either political or religious) are new editions of the
> fundamentally Unitarian nature of religion and politics.
>
> So, then, gods and adivines have been and will be here forever.

I concur.



> When a name for them is discredited, they appear with new names and
> within new organization.

Absolutely. Some atheists sects can copy some clergy ritual at the
level of the microcospic details, and also the authoritative
arguments. I am thinking to some atheist masonic lodges (not all).



> The modern Global warming alarmism is an episode of adivination by
> makin illegitimate use of science. the Marxism was a scholastic
> school of Masters of Reality that claimed predicitive powers over
> the story of Humanity. The gigantic photographs of Marx Lenin in the
> URSS parliament is an example of religious temple of Atheism. But
> also the small photograph or a loving one in the dormitory carries
> out a religious sense, Specially if it passed away and it was a
> greath influence in our lives. Religion is everywhere and forever.

OK. But it can progress. The authoritative argument in science and
religion is a rest of our mammals reflex. Dogs and wolves needs
leaders, for reason of a long biological past story. It makes sense
for short term goal, like it makes sense to "obey" to orders in the
military situation. But it is really an handicap for the long run.

And that means that authoritative arguments will disappear, in the
long run, or we will disappear, like the dinosaurs. Natural selection
can select good things for the short terms, and throw them away later.
What will not disappear is science and religion. Religion and
spirituality will be more and more prevalent, and play a role of
private goal, and science will be more and more understood as the best
tool to approximate that spiritual goal. I think.

To fight fundamentalism in religion, theology should go back to the
academy (which like democracy is the worst institution except for all
others!).

Bruno

http://iridia.ulb.ac.be/~marchal/



smi...@zonnet.nl

unread,
Aug 19, 2012, 11:22:42 AM8/19/12
to everyth...@googlegroups.com
Citeren Bruno Marchal <mar...@ulb.ac.be>:
And even divergent series can be resummed to yield a finite answer,
sometimes even using just a few terms.


Saibal

Craig Weinberg

unread,
Aug 19, 2012, 12:39:30 PM8/19/12
to everyth...@googlegroups.com
 
To me, objects ÷ space subjects * time = realism is easy also.


Straightness and circularity are two presentations with very narrow, precise, and literal characteristics. They can be said to oppose each other in one sense and be mutually exclusive forms, but the larger the circle, the straighter the curve of it's circumference (I know you have a better way of saying that). All possible geometric forms and topologies however would be categorized as 'entopic-literal' forms, as they are shapes which refer only to themselves. Entopic hallucinations are those which are repeating geometries, escalating into full blown ecstasies of Platonic psychedelia.

The other major category of hallucination (and therefore nakedly exposed consciousness) is eidetic - faces in the trees, butterflies in the inkblot, etc. Instead of referring literally to a shape of ink on paper or light and dark regions of bark, they present characters or creatures, mythical themes, etc. This is, like linearity is to circularity, the opposite potential of awareness - one that simplifies a complex figurative identity into a gestalt or algebra (algebra in the etymological sense of a "reunion of broken parts") rather than a topological form that is literal. This ratio, between poetic, dreamlike involvement and literal, mathematical observation I am saying forms the grand continuum of the cosmos.

Hmm...




 






The dreaming number are usually very big concrete number. They dream  
by encoding computational state of person, relatively to some  
universal number, which are encoding universal machine relatively to  
some other one, and the initial one can be chosen arbitrary. Those are  
not symbolic number, but real encoding number, a bit like the genome  
if you want.

Why would that result in a dream?

Because I work in the comp theory where we come to the idea that consciousness can be manifested by abstract relation between numbers, as they emulate computation. We have already said "yes" to the doctor.


You don't really 'come to the idea' at all though, you assume it from the start. There is no theory for why or how numbers would dream, only the assumption that they do.


Here I absolutely disagree. The theory is that I am a material machine. The conclusion is that matter is an hallucination, yet a lawful one. It looks like you have not yet take a deep look on UDA.

I conclude that matter is a hallucination also, but not our hallucination. Matter begins as the (shared, intentional) hallucination of our molecular subselves. The key though, is that the extent to which our experience is distant from our molecular sub-realities is directly proportional to the realism and involuntary nature of our experience with matter.

It's confusing to me when you say that we are a material machine, yet matter is a hallucination, so that means we are a hallucination machine - which is ok by me, but why bring matter into it at all? What makes some hallucinations into matter? My view is that the same thing that gives the hallucinations significance (makes them more narrative and eidetic, more pull toward gestalt coherence) has a symmetric exhaust in the form of entropy...which is space. It is space/entropy which provides Cantor-set-like statistical gaps in which the various layers of realism can break down in peace. The gaps are where the dreams hide their unrealism and forgives their continuity errors. The world of matter looks like it makes perfect sense, but to our molecular selves, it may be a timeless chaos of conflicting orders.
 



 
 


It seems shrouded in obfuscating self-reference. Why would anything that has been encoded ever need to be decoded if the machine can fluently process the encoded form?

To store what we learn. The DNA plays already such a role at the molecular level. It illustrates also a digital encoding and decoding. 

Why would storage imply any kind of encoding? Without physics to constrain resource requirements, there would be no need to compress information.

The numbers do that all the time. There is no need, but they cannot not doing it, because it follows from addition and multiplication. There is no need for Saturn's ring either.

What I am saying is that the reason they do it all the time is that numbers are different than you assume. They have no existence that is independent from private time and public space. There is no Platonia, only the sense of the whole which resides inherently in every apartness.
 



I can't think of any reason that a Turing machine would need to create an abstraction layer, especially if you could control the speed of the tape. If for any reason you needed subroutines to synchronize, you could run those parts of the tape faster or slower - infinitely fast if you like. Without physics, time is relative and unbounded. A Turing machine needs no programming language, it doesn't even need bytes. You can do all of your addressing on the fly with dynamically sized memory chunks. All of these formalisms, codes, architectures, etc are purely in the service of the limitations imposed by physics and human convenience.

Not if you bet that we are machine. In that cse the UD reasoning shows that what you describe is only the appernt view from inside, but that eventually the physical resource constraints arise from the mathematical computer science constraints, that is, from arithmetic.

Why would any constraints arise purely from arithmetic that would be resolved by encoding?
 



We get tired of writing a thousand lines of code every time we want to transform some dataset, but the computer doesn't care. It will run a trillion lines of code just to add the same number to itself, it doesn't care how long it takes or how wasteful it seems to us.

Not with comp, as I am a person which manifest itself through a computer, and persons care.

Comp begs the question. At what point would a computation start to invent 'care' as a meta-computation? If there were arithmetic resources to consider, surely the addition of encoding, decoding, and compiling  would use more resources rather than less.







Why would it need any other form - especially if it is all made of numbers?

Nobody needs a universe. Why do we do babies?
The "truth" is that we have them, we cannot really avoid them. It is like the prime number and the universal machine. Once you have zero and + and *, you get Platonia, and a lot of mess in Platonia. It is a logical consequence.

But a universe is not a consequence of Platonia.

It is. Or your non-comp assumption is correct, but I do not work in the theory non-comp. Actually, I have not yet seen a non-comp *theory*. Only philosophical argument against comp, but no concrete replacement.

My theory is not non-comp, it is the reconciliation of comp and non-comp. Part of it is to explain why any theory by definition can only apply to comp. As Goethe said, “My friend, all theory is gray, and the Golden tree of life is green.”



Since we know that we have a universe

We know only that we are conscious. How could we know that there is a (physical) universe? 

How could we not know that there is a physical (part) of the universe? We know it with the same certainty that we know 2+2=4. We can tell when we finally do wake up in the truly public space of the universe, that it is a different experience from even the most convincing dream.
 
We don't even *know* that there is a mathematical universe, or even an arithmetical universe. We bet on it.


Then that relativistic bet becomes what we mean by *knowing*. There is only seems like, and physics is a seems like which pretends to be the opposite. Matter seems like it 'simply is.'






that means that it is Platonia which needs to be justified intellectually in terms of contributing to the universe, not the other way around.

But with comp, we do got an explanation of the physical universe(s). And we got both quanta and qualia. It might be false, but the explanation is there.

Haha, the 'it might be false' part doesn't concern you? Shouldn't the fact that it doesn't concern you concern you? But I don't see that comp gives you qualia. I don't see red or pain, just ineffable namespaces. A hole where qualia can be inferred, but only if we already have the example of qualia to plug into the explanation.

 



 



What I am saying is that if you are going to invoke a possibility of dreams, that has to be grounded in the terms that you are laying out as primitive. Why would dreams leap out of mechanical relations?

It is a logical consequence, once you accept the idea that you might survive with a digital brain.

It's the logic of begging the question. I ask you why numbers dream and you say it's logical if you accept that dreams can survive as digital process. I don't though. If human dreams could exist in something other than humans, then they would already.

And that is the case already, indeed. That follows from the reasoning.

Where else do human dreams exist except in human experience?
 



On the sands of the beaches, in the odd complicated cloud formation. I would need a compelling reason to believe otherwise. Why would I give the benefit of the doubt to machines when machines have clearly shown no signs of feeling thus far?

But molecular biology, quantum mechanics, etc. suggest strongly that we are machine. The rest is the logical consequence.

They suggest a mechanistic level of what we are, but given that consciousness can't be detected externally even at the macrocosmic level, there is no hope that any other level besides the lowest quantitative mechanistic level would be revealed. You can't find irony and love with a microscope. The logic is biased because it assumes a voyeuristic access to reality rather than an integrated perception of multiple subjects. We are machine, but we are also the opposite of machine, perhaps even a machine that specifically amplifies the opposition to mechanism to a higher degree than any other.

 
There is no evidence for non-comp.

We are the evidence for non-comp. Not our bodies but our individuality and creativity, humor, passion, initiative, etc. The microbiology is the boring part. The interesting part is impossible to see directly because it is what we are, but it can be understood by inference of symmetry.

 
So we do have compelling reason to think that machine can have feeling. The conjunction of our own experience, and the study of our bodies.

You have a compelling reason to think that, but I see the clear error of painting ourselves into a corner based on faulty assumptions. You assume that our being identical with our body means that who we are must be reduced to body-like process, whereas the fact that we can choose to change the chemistry of our brain, move our voluntary muscles, etc, tells me that there is another side to the story: The body must also be seen as a reduced shadow of processes which are us. If we are made of neurons, then neurons, at least to some extent feel and see also, and if neurons feel and see then molecules do something similar. The fact that subjectivity is private by nature makes it unsurprising that we would be reluctant to consider that this is the case, especially since the body is the comp-space-matter extension that evacuates all appearance of subjectivity. Subjectivity does not survive objectification, which means that subjects can't be reduced to objects.


But I don't want to defend comp. That's the kind of philosophy I do not work on. I just show the consequence of being material machine. The consequences are that we are already in a "matrix", whose structure can be mathematically handled, so that we can constructively derive physics, and compare it to the empirical physics, so that comp is shown testable. (To be short).


Not just that they might not have feelings, but that they are universally known for lacking any capacity for feeling whatsoever - that is what defines the nature of machines. When you understand the continuum of eidetic to entopic, figurative to literal, you will understand why this makes perfect sense

But what you say makes sense. The comp theory already explains why comp is hard (impossible, even)  to believe, and why it has to be highly counter-intuitive. 

Comp isn't hard for me to believe at all, I just think that if we play out the axioms of comp alone, we do not wind up with realism or a universe.  We have to begin from our whole experience and make sense of arithmetic in that context and not assume that arithmetic can be its own independent context (because why would it need or want any other?).



A machine is an assembly of logical forms. That does not produce any phenomenology by itself because it is constructed from the outside in. Living organisms build themselves from the inside out, from their own native sensitivities and motivations. They are not having an alien script intentionally imposed upon their behavior.

I don't know that. Genetic illustrates that nature do scripts. But math shows also that very short scripts, like "help yourself" can lead to tremendous richness and complexity. 


Nature can do scripts, but that doesn't mean that the scriptee has the level of awareness used by the scripter. Nature is full of exploitation.
 


 



Even if there was some purpose for it, how could that actually take place - what are the dreamings made of?

Ontologically: nothing take place. All the computations are there. Some emulate self-observing machine and the math explain why they have to be beffudled by existence and conscience.


That may be, but why in the world would computational befuddlement be expressed as personality and realism?

Because there is, very roughly, 60000^10000000000 brain states possible, that makes a lot of subjectivity possible, and they are multiplied and selected through a filtering on a continuum of computations (first person indeterminacy). 

That's a naive anthropomorphizing of computation though. What does Platonia care for 'a lot'? It has infinity and eternity.
 






My view is that it may be the case that everything that is not matter across space is experience through time - by definition, ontologically. There is no other form or content possible in the cosmos. Numbers are experiences as they must be inferred by computational agents and cannot exist independently of them. What my formulas do is to propose a precise relation between dream-time (including logical algebras) and matter-space (including topological geometries). To do this we need to invoke a continuity between them which is a perpendicular axis which runs from the literal (tight equivalence; induction is accomplished through linear arithmetic logic) to the figurative/metaphorical (loose thematic association; induction is accomplished through linear logic as well as elliptical cross-context leaps).

I don't believe in time, space, cosmos, matter, ...
I explain their appearance by the dream property of numbers, relatively to universal numbers.


That's what I am showing you, is how the dream property of numbers (eidetic-figurative) and the number property of dreams (entopic-literal) are two polar regions of the same (involuted) continuum.

I am a simple minded logician. If you use a technical term, you have to explain it to me in a precise frame theory.
I don't ask you to formalize it in first order logic, but to succeed in explaining this enough precisely so that a poor logician like can understand that with time and patience we can get such a formalization. If not it looks like you can say anything you want, and very often, it makes sense in machine first person terms, and thus fail to convey your non-comp intuition more than the natural non-comp intuition of *all* correct machine.
Comp shows intuition locally correct, locally useful, locally unavoidable, but globally wrong.

Likewise non-comp shows the logic of 'correct and useful' to be trivially valuable but profoundly inadequate. We can use different terms, as long as one side is literally figurative (numbers are actually figures) and the other figuratively literal (dreams seem like reality to some degree). I keep throwing out different terms, which ones are you comfortable with?

Oriental - Western.
Subjective - Objective.
Algebraic-Topological.
Dreamtime - Spacetime.
Interior - Exterior.
Proprietary - Generic.
Signyfing - Generic.
Personal - Universal.
Private - Public.
Eidetic - Entopic
Narrative - Scenic
Portrait - Landscape
Vertical - Horizontal
Solipsistic - Mechanistic
I - it

ℵ-Ω

ॐ - (ॐ - ॐ)

 

I think I am doing that. I don't know that others can be convinced though, they have to convince themselves.
 



Modal logic can make a toy model of experience, by removing all of the experience and replacing it with a meaningless variable.

Why meaningless?

Because that's what variables do, they generalize all potential content-meanings under single quantifiable term.

Hmm... OK. But the person is in the box (B), not in the variable, which is usually used for some unspecified arithmetic proposition, or memories content, etc.

If the box is B and the person is in the box, then the person is in B, which is a variable containing no possibility for content (other than to refer to a potential for content...B stands for a menu, not a meal).
 

The incommunicable nature of qualities like red is not sufficient to point to anything like red though. Water is wet, but so are a lot of things. Wetness doesn't make it water and ineffability by itself doesn't generate an explanation for qualia.
 
 

 


What quality does this modality have? Is it shiny? Is it spicy? Does it get tired?

The modality has no quality. It describes qualities. You might do a confusion of level. It is a bit like the confusion between the string "s(s(0))" and the number s(s(0)).

 
How is it describing qualities? It seems to me to be describing quantitative formalism. How does one describe red logically?

By explaining it exists necessarily and that it cannot be describe logically. Modal logic, in the machine context are "meta". It does not explain the quality red. It explains that quality exists and are NOT describable quantitatively. 

The "universal soul", Bp & p, or the S4Grz logic, is a sort of mathematical tour-de-force: it is a formal logic describing the logic of content which are impossible to formalize ever. But we can formalize this at the metalevel.


 
But it formalizes red out of existence. It makes it #FF0000.


I don't deny that we are machines, only that we are entirely machines. My point is that we can just as easily be described as non-machines with machine-like tendencies as the other way around. The symmetry between the two and the sense between them is what I am interested in.
 




While I admit, certainly, to being ignorant of the particular details of mathematical principles which give you a more generous view of machine consciousness than I have, I don't limit the capabilities of machines in the way that you assume. If quanta is flat qualia,

Quanta are more sort of sharable qualia.

I agree, but what I am adding to that is that the more sharable they are, the less qualitative they have to be by definition. If you want the most universal truths, they must be shallow and meaningless. If you want deep and meaningful truths then they must be personal (to match up with a lifetime's worth of personal associations and experiences).

 
First person plural construct brought by the many-world internal interpretation of arithmetic by arithmetical beings.

 I think MWI is a plug for the inadequacy of comp. With sense, you don't need to formally execute all possible computations, you can use elliptical (...) nesting to fill in for infinities and therefore retain the coherence of a single-world interpretation, or as I call it, a Sole Entropy Well (single source of almost zero entropy - pure signal-ness, which perpetually 'floats on top' of all subordinate causes and conditions...like God, but without the beard.)




then machine intelligence can extend infinitely farther than organic intelligence, because of the horizontal universality of it.

Yes, indeed.



What I am saying is that you can't build qualia out of quanta,

I agree with this. Qualia emerges logically before quanta, but it seems the contrary. That can perhaps be related to Stephen-Pratt reversal of arrows in the mind body relation, or to the duality between thrid person (Bp) and the Theaetetical first person (Bp & p).


Sounds promising?
 


because there is no universal qualia.

This I doubt. More from personal reasons than comp, though.

Universal qualia I suppose is qualia of the unity/symmetry between qualia and quanta.
 


Qualia is othrogonal to quanta - it is the manifestation of non-universality itself.

Well, with comp, quanta is the tiny emerging part of the qualia-consciousness part of the arithmetical platonia, seen from inside.

May be the crux of our difference (beyond comp/non-comp) is that, as many people, you seem to believe that lind and matter are symmetrical and have same caliber. But comp is still open too full platonism, in which matter, and physical universe are more like a divine bug, a temporary misleading; hding the truth more than paving a way to it. Some platonist identify matter with bad, or the devil. this is of course an open problem with comp, but what can already been argued for is that the mindscape is vastly bigger than the physical scape. The comp mindscape or personscape is vast. *Very* vast. It escapes the math. It violates physicalism, but also mathematicalism, despite the ontology is quite little (a tiny segment of arithmetical truth). 

Bruno


I think of matter as bad i

What's wrong with seeing mind and matter as symmetric? I wouldn't say mind, because there are many more ways to experience than through a cognitive mind, but to me the symmetry is the sense of it it all. Good and bad are broadly associated with self and matter, but only in the sense that matter is the resistance against significance, and therefore entropy, loss, degeneration, and death. Of course that is simplistic, and in reality, matter is the collection of significance itself, just seen from the outside perspective which can only appreciate a minimal trace of the significance it represents in the fullness of time. Matter amplifies significance by making it more dear...the vastness blackness of space making the stars more brilliant etc.

Craig
 

Bruno

 

Stephen P. King

unread,
Aug 19, 2012, 3:51:37 PM8/19/12
to everyth...@googlegroups.com
Dear Bruno,

    As I see it, the quasi-infiitely more complex aspect of "I understand that 2+2=4" follows, at least, from the requirement that many entities capable of making such statements can point to examples of 2+2=4 and communicate about such statements with each other however far away in space and time they are from each other. We can ignore the fact that there is a collection of entities to whom the statement "I understand that 2+2=4" has a meaning. You need to get a grip on the nature of meaningfulness. Searle has tried to do this with his Chinese Room idea but failed to communicate the concept. :_(

snip






The dreaming number are usually very big concrete number. They dream  
by encoding computational state of person, relatively to some  
universal number, which are encoding universal machine relatively to  
some other one, and the initial one can be chosen arbitrary. Those are  
not symbolic number, but real encoding number, a bit like the genome  
if you want.

Why would that result in a dream?

Because I work in the comp theory where we come to the idea that consciousness can be manifested by abstract relation between numbers, as they emulate computation. We have already said "yes" to the doctor.


You don't really 'come to the idea' at all though, you assume it from the start. There is no theory for why or how numbers would dream, only the assumption that they do.


Here I absolutely disagree. The theory is that I am a material machine. The conclusion is that matter is an hallucination, yet a lawful one. It looks like you have not yet take a deep look on UDA.

    No Bruno, you do not *just* have a theory that "I am a machine". There are many assumptions buried in the UDA that are ignored. One of them is that there are a pre-existing collection of distinguishable entities capable of comprehending what is an abstract theory. This includes the blackboards on which you write your "chicken-scratch" and the internet and the computers that we are using to write, send and receive these emails. There is an entire physical world that you want to ignore, mostly, I suspect, because you do not want to deal with the interaction/concurrency problem. The conclusion that "matter is a hallucination", while correct IMHO, does not eliminate the impact of this pre-existing collection and their capacities.



 
 


It seems shrouded in obfuscating self-reference. Why would anything that has been encoded ever need to be decoded if the machine can fluently process the encoded form?

To store what we learn. The DNA plays already such a role at the molecular level. It illustrates also a digital encoding and decoding. 

Why would storage imply any kind of encoding? Without physics to constrain resource requirements, there would be no need to compress information.

The numbers do that all the time. There is no need, but they cannot not doing it, because it follows from addition and multiplication. There is no need for Saturn's ring either.

    Ultimately one could argue that anything is "no needed" and yet such arguments are wrong headed.

    Numbers are not isolate entities! They do not "do" anything in and from themselves. There is not even an "self" in a number alone unless we think of them as a fixed point of a class of transformations of a quasi-infinite collection. You seem to be using a Sorites paradox type of argument here to make a false point.
http://plato.stanford.edu/entries/sorites-paradox/
"The sorites paradox is the name given to a class of paradoxical arguments, also known as little-by-little arguments, which arise as a result of the indeterminacy surrounding limits of application of the predicates involved. For example, the concept of a heap appears to lack sharp boundaries and, as a consequence of the subsequent indeterminacy surrounding the extension of the predicate ‘is a heap’, no one grain of wheat can be identified as making the difference between being a heap and not being a heap. Given then that one grain of wheat does not make a heap, it would seem to follow that two do not, thus three do not, and so on. In the end it would appear that no amount of wheat can make a heap. We are faced with paradox since from apparently true premises by seemingly uncontroversial reasoning we arrive at an apparently false conclusion.
This phenomenon at the heart of the paradox is now recognized as the phenomenon of vagueness (see the entry on vagueness). Though initially identified with the indeterminacy surrounding limits of application of a predicate along some dimension, vagueness can be seen to be a feature of syntactic categories other than predicates. Names, adjectives, adverbs and so on are all susceptible to paradoxical sorites reasoning in a derivative sense".



I can't think of any reason that a Turing machine would need to create an abstraction layer, especially if you could control the speed of the tape. If for any reason you needed subroutines to synchronize, you could run those parts of the tape faster or slower - infinitely fast if you like. Without physics, time is relative and unbounded. A Turing machine needs no programming language, it doesn't even need bytes. You can do all of your addressing on the fly with dynamically sized memory chunks. All of these formalisms, codes, architectures, etc are purely in the service of the limitations imposed by physics and human convenience.

Not if you bet that we are machine. In that cse the UD reasoning shows that what you describe is only the appernt view from inside, but that eventually the physical resource constraints arise from the mathematical computer science constraints, that is, from arithmetic.

    Sure, we have evidence that our experiences of the world are not direct contact with the objects making up the world but are a form of virtual reality generated by our brain. Therefore we can make good bets that more often than not are good such that they pay off, but the computation that the brain generates that our experiences supervene upon simply does not happen in the absence of physical processing that fall under the rule of "thermodynamics". There is no knowledge for free. Arithmetic is the product of many such brains interacting with each other *and* it is a relational structure that exists in its own right. We cannot remove the former of this pair and imagine that the latter is left to float free and continue with all of its abilities intact.



We get tired of writing a thousand lines of code every time we want to transform some dataset, but the computer doesn't care. It will run a trillion lines of code just to add the same number to itself, it doesn't care how long it takes or how wasteful it seems to us.

Not with comp, as I am a person which manifest itself through a computer, and persons care.

    Certainly, but this is because of the vast constellation of other persons that you are capable of not only betting to exist and have some particular set of properties, but also cannot just theorize out of existence. COMP does not exist in isolation. It is a concept that exists in (all possible versions of ) our records of it (which include our internal brain memories).




Why would it need any other form - especially if it is all made of numbers?

Nobody needs a universe. Why do we do babies?
The "truth" is that we have them, we cannot really avoid them. It is like the prime number and the universal machine. Once you have zero and + and *, you get Platonia, and a lot of mess in Platonia. It is a logical consequence.

But a universe is not a consequence of Platonia.

It is. Or your non-comp assumption is correct, but I do not work in the theory non-comp. Actually, I have not yet seen a non-comp *theory*. Only philosophical argument against comp, but no concrete replacement.

    I cannot speak for Craig directly, but where we have a disagreement is in the things that COMP ignores. The larger universe within which it has meaningful expression.



Since we know that we have a universe

We know only that we are conscious. How could we know that there is a (physical) universe?

    Because we can kick the wall and watch our foot rebound. There is overwhelming evidence that there is a physical universe. The very act of exchanging these emails is some of that evidence! Can we be certain of what it is like in-itself? No! We cannot! But we can form theories about what it might be and test these. This fact alone tells us that there is "something" that is there and it is not just the figment of my (singular) imagination.


We don't even *know* that there is a mathematical universe, or even an arithmetical universe. We bet on it.

    Yes, and we can do that only because of the prior possibility of being able to represent facts about that universe to ourselves and each other.



that means that it is Platonia which needs to be justified intellectually in terms of contributing to the universe, not the other way around.

But with comp, we do got an explanation of the physical universe(s). And we got both quanta and qualia. It might be false, but the explanation is there.

    Only to you is there such a complete explanation. We simply cannot ignore the fact that we are "not-Bruno" and that only "Bruno bets correctly that comp is true". This is like the sentence: "Stephen Paul King cannot prove that this statement is true". I can only bet on it one way or another, but anyone else can prove things about it.



 


What I am saying is that if you are going to invoke a possibility of dreams, that has to be grounded in the terms that you are laying out as primitive. Why would dreams leap out of mechanical relations?

It is a logical consequence, once you accept the idea that you might survive with a digital brain.

It's the logic of begging the question. I ask you why numbers dream and you say it's logical if you accept that dreams can survive as digital process. I don't though. If human dreams could exist in something other than humans, then they would already.

And that is the case already, indeed. That follows from the reasoning.

    :-P



On the sands of the beaches, in the odd complicated cloud formation. I would need a compelling reason to believe otherwise. Why would I give the benefit of the doubt to machines when machines have clearly shown no signs of feeling thus far?

But molecular biology, quantum mechanics, etc. suggest strongly that we are machine.

    You are not just talking about particular formal theories! You are also tacitly taking into account the vast world of observations using those theories that find them to be correct. This does not deny the suggestion that we are machine, but it does not allow a free floating abstract representation of The Machine to exist *and* do things that only the universe of interacting entities can do.


The rest is the logical consequence. There is no evidence for non-comp.

    This email *is* evidence! That I can coherently  (in spite of my dyslexia) understand what you might mean with those symbols) such that I can respond in kind is evidence that not-comp. But I am only focusing on the error in step 8 that makes comp fail for me, and thus make a non-comp claim.



So we do have compelling reason to think that machine can have feeling. The conjunction of our own experience, and the study of our bodies.

    No no no no no! We cannot ever have any reason, it is not 1p and thus cannot be considered as knowledge! It is not possible to exactly represent the 1p, we can only approximate it. We can get very very very close to it, but we can never get "it". My experience of what it is like to be me is not sharable, and thus cannot be copied, coded or transferred. I can only copy, code and share representations of it. The map is only the territory when the territory is the map. Otherwise please do not conflate or confuse the two.




But I don't want to defend comp. That's the kind of philosophy I do not work on. I just show the consequence of being material machine. The consequences are that we are already in a "matrix", whose structure can be mathematically handled, so that we can constructively derive physics, and compare it to the empirical physics, so that comp is shown testable. (To be short).


Not just that they might not have feelings, but that they are universally known for lacking any capacity for feeling whatsoever - that is what defines the nature of machines. When you understand the continuum of eidetic to entopic, figurative to literal, you will understand why this makes perfect sense

But what you say makes sense. The comp theory already explains why comp is hard (impossible, even)  to believe, and why it has to be highly counter-intuitive.

    Only because of the very abstract math involved, otherwise it is not complicated at all! Otherwise the "explain it to your grandmother" is impossible.



A machine is an assembly of logical forms. That does not produce any phenomenology by itself because it is constructed from the outside in. Living organisms build themselves from the inside out, from their own native sensitivities and motivations. They are not having an alien script intentionally imposed upon their behavior.

I don't know that. Genetic illustrates that nature do scripts. But math shows also that very short scripts, like "help yourself" can lead to tremendous richness and complexity.

    Umm, this is a subject that we should elaborate on all on its own. The sentence "help yourself" is not just 12 symbols in some particular order!



 

Even if there was some purpose for it, how could that actually take place - what are the dreamings made of?

Ontologically: nothing take place. All the computations are there. Some emulate self-observing machine and the math explain why they have to be beffudled by existence and conscience.


That may be, but why in the world would computational befuddlement be expressed as personality and realism?

Because there is, very roughly, 60000^10000000000 brain states possible, that makes a lot of subjectivity possible, and they are multiplied and selected through a filtering on a continuum of computations (first person indeterminacy).
    Sure, a lot of difference that make a difference can be coded with those many brain states. But you need to show why they have the particular rules that they have. We simply cannot ignore the body problem!






My view is that it may be the case that everything that is not matter across space is experience through time - by definition, ontologically. There is no other form or content possible in the cosmos. Numbers are experiences as they must be inferred by computational agents and cannot exist independently of them. What my formulas do is to propose a precise relation between dream-time (including logical algebras) and matter-space (including topological geometries). To do this we need to invoke a continuity between them which is a perpendicular axis which runs from the literal (tight equivalence; induction is accomplished through linear arithmetic logic) to the figurative/metaphorical (loose thematic association; induction is accomplished through linear logic as well as elliptical cross-context leaps).

I don't believe in time, space, cosmos, matter, ...
I explain their appearance by the dream property of numbers, relatively to universal numbers.


That's what I am showing you, is how the dream property of numbers (eidetic-figurative) and the number property of dreams (entopic-literal) are two polar regions of the same (involuted) continuum.

I am a simple minded logician. If you use a technical term, you have to explain it to me in a precise frame theory.
   
    Yes, please explain this more Craig.


I don't ask you to formalize it in first order logic, but to succeed in explaining this enough precisely so that a poor logician like can understand that with time and patience we can get such a formalization. If not it looks like you can say anything you want, and very often, it makes sense in machine first person terms, and thus fail to convey your non-comp intuition more than the natural non-comp intuition of *all* correct machine.
Comp shows intuition locally correct, locally useful, locally unavoidable, but globally wrong.

    But this "globally wrong" is just an abstraction itself. We are assuming a semiotic theory here! Another aspect of the body problem...
    Mhm!




Modal logic can make a toy model of experience, by removing all of the experience and replacing it with a meaningless variable.

Why meaningless?

Because that's what variables do, they generalize all potential content-meanings under single quantifiable term.

Hmm... OK. But the person is in the box (B), not in the variable, which is usually used for some unspecified arithmetic proposition, or memories content, etc.

    No! A person is not "in the box (B)" at all! This is a map-territory conflation without explanation!




 

It is to say 'here, instead of your baby, let's just call it 'consumer of diapers and milk' and design a nursery based on the statistics derived from that consumption.

No, it is the complete contrary. I see your point, but it is eaxtly that type of reduction that is prevented by comp. Even for machine, we can no more reduce them to their third person description. They do have a soul, even after-life, etc.
 
If machines have souls, then there has to be a way that the two are connected. There has to be some kind of rationale for it.

Nice to hear that. I agree.

    As do I.





What is needed is a way to reference phenomenological qualities which expresses not merely non-comp, but explicitly asserts quality and a view of the universe from the perspective of irreducible quality.

You have to postualte them, and to postulate matter, and to postulate a relation, and non-comp, and that is neither satisfying, nor working. It introduces difficulties where there are already enough, imo.

I don't think you have to postulate matter, you only have to formalize what you already assume about dreaming numbers.

OK. That's the point. In the comp theory.

    But comp theory does not float free of the world within which it has a meaning! Just as how "17 is prime" has a unique truth value only in a world where that set of physical pattern convey a meaning. In the absence of that world there is not meaning nor truth value.



 



> We can’t pretend to talk about the eidetic, dream like  
> perpendicular of number logic while using the purely empirical terms  
> of arithmetic reduction. We need symbols that can only refer to  
> named qualities rather than enumerated quantities.
>
This is exactly what happen when you define the first person by the  
knower. Bp & p, or if you prefer

provable(p) and true(p),

provable and true are not first person qualities, they are epistemological quantifications. First person would be more like great(p) and superior(p).

?

A square peg either fits in a particular sized round hole or not. That is a true/false. It can be proved by actually trying to fit the peg in the hole. Those are third person public conditions. Whether one peg or hole seems 'better' than another is a first person kind of distinction.

    Sentences are true (or false) only in worlds that have demonstrations of those sentences. Truth is not independent of context. It is independent of any particular representation within a set of mutually accessible worlds. We cannot imagine that entities have some particular set of properties (which include truth values) in the absence of the possibility to verify such and verification is the quintessence of the physical world, unless we are OK with incoherent thinking....


 



gives a modality which can provably be shown qualitative, and non  
formalizable in arithmetic. It leads to a logic (know as S4Grz) which  
describes something which is absolutely impossible to reduce to any  
number relations or even anything third person describable notion,  
even infinite one.

That doesn't mean it is qualitative, only that it is so obscurely arithmetic that arithmetic itself cannot quantify it.

But the machine still can refer to it. So I don't see why this would not work. It would not work if you reduce a machine to its body, but the divergence between Bp and Bp & p disallow such a reduction.

I don't really understand what modal logic has to do with the possibility of quality.

The machine modal logic of self-reference, Bp, when conjuncted with the non nameable Truth, leads the machine to discover true propositions, in a sort of immediate way, which they cannot justify nor even describe, except by projecting them on similar machines.

    Truth is not an object per se. It can be identified with an object, but it itself is not. The map is only the territory when the territory is the map. Truth is not a person and thus has no particular name, thus it cannot beconjuncted with anything else. It has no peer!


The redness quality is like that. I have no doubt that I experience redness, but I am unable to communicate it to a blind rationalist. I can point on something red, and hope my fellow has a sufficiently similar experience, so that the word "red" will have a sufficiently close meaning to mine.

    You cannot communicate redness at all! Qualia are strictly 1p. We can talk about representations of 1p, but never 1p itself. This is why we need something like bisimulation to represent communication and interactions in general.



 

What quality does this modality have? Is it shiny? Is it spicy? Does it get tired?

The modality has no quality. It describes qualities. You might do a confusion of level. It is a bit like the confusion between the string "s(s(0))" and the number s(s(0)).

 
How is it describing qualities? It seems to me to be describing quantitative formalism. How does one describe red logically?

By explaining it exists necessarily and that it cannot be describe logically. Modal logic, in the machine context are "meta". It does not explain the quality red. It explains that quality exists and are NOT describable quantitatively.

    OK, then please be consistent! Our explanations only touch the representations of the 1p, we never touch our 1p themselves.



The "universal soul", Bp & p, or the S4Grz logic, is a sort of mathematical tour-de-force: it is a formal logic describing the logic of content which are impossible to formalize ever. But we can formalize this at the metalevel.

    We have formal representations of representations of the concept of "universal soul"... The concept is a collectively agreed upon representation and that collective agreement only happens because of the physical world.







You might think I just described it, by Bp & p, or by "provable(p) and  
true(p)", but this is not the case, as I use some of your intuition  
about truth, which cannot be arithmetized by itself, by a famous  
result of Gödel and Tarski (independently).
It happens that we do have a good intuition of many truth, and machine  
can indeed describe better and better approximations of the truth  
concept, but the limit of it, used here, cannot be. So by using both  
the comp hypothesis, and by studying simple (Löbian) machine (simpler  
than us) we can develop a formal (quantitative in some sense, at some  
level, from some point of view) theory concerning the non formal, and  
even non-formalizable-at-all-by-the-machine, qualities that machine  
can still refer about. And this can be used to explain why machine are  
forced to be befuddled by the subjectively-real apparent gap between  
third and first person attributes.

It seems like you are missing the obvious. Awareness is not just about knowing and navigating a set of logical conditions. That can be accomplished easily without any awareness. Experienced qualities are orthogonal to knowledge and procedural evaluation.

I can understand that "navigating a set of logical conditions" can be done without awareness. Awareness or knowlegde arrives when the navigating embed the navigator in truth, or in a reality. There is a fixed point, and it is explained why this is felt as a personal non communicable experience.

The fact that there would be data that cannot be communicated from a fixed vector doesn't imply to me anything specifically personal or experiential. What is relevant about consciousness is not only *that* it is private, but what the content of that privacy is.

Of course.

    But please do not miss the point here, Bruno! You are ignoring the fact that representations can be of other representations but not only of other representations. Somewhere some how, there must be a persistent demonstration of the referent.




The form of it is trivial, except that it helps us understand the relation to exterior realism. Using modal logic to describe experience makes a formula out of the shape of a can and presents it as a way of creating canned food.

See above.

    Yes, please do, Bruno! Maps of maps of maps of maps of ... only converge in a representation of the stream of mappings... and such a concept has a meaning because there is something that it is like that is a 1p.




 

 


> Let the ignoring and insulting begin!
>

We don't need that here, I think, nor anywhere. An insult is almost  
always equivalent with "I have no argument".

I agree.

I know. You are wise. And we agree on many things, but we are working in antipodal theories. I think you have reductionist conception of machine, to be franc.

You are wise too. I think that your view of my view of machines is more of a projection than you think.

Well, if I remember well, you are the one who refuse to sell a steak to my daugher's husband, who said "yes" to a doctor, and is, by construction, a machine.

    Umm, are you going to sue him for illegal acts of discrimination? Horrors, Craig is a criminal! Am I one as well for trying hard to actually understand what he is trying to communicate? To be serious, Bruno, plese consider that Craig and I have theories about machines and are not reductionists. We are just considering a wider context than you do with COMP. You are only considering formal theories that can be expressed in sigma_1 sentences, no?



While I admit, certainly, to being ignorant of the particular details of mathematical principles which give you a more generous view of machine consciousness than I have, I don't limit the capabilities of machines in the way that you assume. If quanta is flat qualia,

Quanta are more sort of sharable qualia.

    Please explain this sentence. It assumes a theory of bodies that you have, so far, only been coy about.


First person plural construct brought by the many-world internal interpretation of arithmetic by arithmetical beings.

    For there to be a "many-world internal interpretation" there must be something that is a world and a reason for many of them - not just one. These are the sort of things that you do not discuss at length that are at the heart of my own critique of comp step 8.



then machine intelligence can extend infinitely farther than organic intelligence, because of the horizontal universality of it.

Yes, indeed.

    See! We are not reductionist.



What I am saying is that you can't build qualia out of quanta,

I agree with this. Qualia emerges logically before quanta, but it seems the contrary. That can perhaps be related to Stephen-Pratt reversal of arrows in the mind body relation, or to the duality between thrid person (Bp) and the Theaetetical first person (Bp & p).

    Indeed! The reversal of the arrows is a symmetry that is only possible because of the postulation that numbers and physical matter are on the same ontological level (neither is primitive). They emerge from each other.



because there is no universal qualia.

This I doubt. More from personal reasons than comp, though.

    Please elaborate on this!


Qualia is orthogonal to quanta - it is the manifestation of non-universality itself.

Well, with comp, quanta is the tiny emerging part of the qualia-consciousness part of the arithmetical platonia, seen from inside.

    I would agree with this if and only if you concede that the particularity of the quanta are the result of infinitely many bisimulations. This is the solution to the body problem that I propose. You need to see where bisimulation fits in comp. AFAIK, there is no room in COMP for bisimulation because it is the theory of a single monad.



May be the crux of our difference (beyond comp/non-comp) is that, as many people, you seem to believe that mind and matter are symmetrical and have same caliber. But comp is still open too full platonism, in which matter, and physical universe are more like a divine bug, a temporary misleading; hding the truth more than paving a way to it. Some platonist identify matter with bad, or the devil. this is of course an open problem with comp, but what can already been argued for is that the mindscape is vastly bigger than the physical scape. The comp mindscape or personscape is vast. *Very* vast. It escapes the math. It violates physicalism, but also mathematicalism, despite the ontology is quite little (a tiny segment of arithmetical truth). 


    There is no asymmetry between the mental and the material! You only think this because you ignore the uncountable infinity of possible physical worlds. It is the possibility of the thinking that "all is matter" or "all is mind" that is the problem when misunderstood. There is so much more that I need to write on this but I have no more time to write...

meekerdb

unread,
Aug 19, 2012, 4:30:25 PM8/19/12
to everyth...@googlegroups.com
On 8/19/2012 12:51 PM, Stephen P. King wrote:
I understand that 2+2 = 4.
I still cannot explain how and why I understand "2+2 = 4".
"2+2=4" is easy.
"I understand 2+2=4" is quasi infinitely more complex.

Dear Bruno,

    As I see it, the quasi-infiitely more complex aspect of "I understand that 2+2=4" follows, at least, from the requirement that many entities capable of making such statements can point to examples of 2+2=4 and communicate about such statements with each other however far away in space and time they are from each other. We can ignore the fact that there is a collection of entities to whom the statement "I understand that 2+2=4" has a meaning. You need to get a grip on the nature of meaningfulness. Searle has tried to do this with his Chinese Room idea but failed to communicate the concept. :_(

Maybe Bruno will introduce a new modality to his logic Up="Understands p".  :-)

Brent

Stephen P. King

unread,
Aug 19, 2012, 5:43:33 PM8/19/12
to everyth...@googlegroups.com
--


Hi Brent,

    That would be wonderful if possible. AFAIK, understanding is contingent on demonstrability, e.g. I understand p if and only if I can demonstrate that p implies q and q is not trivial and q is true in the same context as p. I think that Bruno's idea of "interviewing a machine" is a form of demonstration as I am trying to define it here. In my thesis, demonstrability requires that the model to be demonstrated is actually implemented in at least one possible physical world (i.e. satisfies thermodynamic laws and Shannon information theory) otherwise it could be used to implement a Maxwell Demon.

    BTW, it was an analysis of Maxwell's Demon that lead me to my current ideas, that abstract computation requires that at least one physical system actually can implement it. This is not ultrafinitism since I am allowing for an uncountable infinity of physical worlds, but almost none of them are accessible to each other (there exist event horizons, etc.).
    Consider the case where a computation X is generating an exact simulation of the behavior of molecules in a two compartment tank with a valve and there exists a computer Y that can use the output of X to control the valve. We can easily see that X could be a subroutine of Y. If the control of Y leads to an exact partition of the fast (hot) and slow (cold) molecules and this difference can be used to run Y then some might argue that we would have a computation for free situation. The problem is that for the hot/cold difference to be exploited to do work the entire apparatus would have to be coupled to a heat reservoir that would absorb the waste energy generated by the work.  Heat Reservoirs are interesting beasts....

See http://www.nature.com/news/the-unavoidable-cost-of-computation-revealed-1.10186 for more on this.

    We ignore the role played by our physical world in our philosophical/mathematical/logical discussions to our peril!

meekerdb

unread,
Aug 19, 2012, 6:03:19 PM8/19/12
to everyth...@googlegroups.com
On 8/19/2012 2:43 PM, Stephen P. King wrote:
On 8/19/2012 4:30 PM, meekerdb wrote:
On 8/19/2012 12:51 PM, Stephen P. King wrote:
I understand that 2+2 = 4.
I still cannot explain how and why I understand "2+2 = 4".
"2+2=4" is easy.
"I understand 2+2=4" is quasi infinitely more complex.

Dear Bruno,

    As I see it, the quasi-infiitely more complex aspect of "I understand that 2+2=4" follows, at least, from the requirement that many entities capable of making such statements can point to examples of 2+2=4 and communicate about such statements with each other however far away in space and time they are from each other. We can ignore the fact that there is a collection of entities to whom the statement "I understand that 2+2=4" has a meaning. You need to get a grip on the nature of meaningfulness. Searle has tried to do this with his Chinese Room idea but failed to communicate the concept. :_(

Maybe Bruno will introduce a new modality to his logic Up="Understands p".  :-)

Brent
--


Hi Brent,

    That would be wonderful if possible. AFAIK, understanding is contingent on demonstrability, e.g. I understand p if and only if I can demonstrate that p implies q and q is not trivial and q is true in the same context as p. I think that Bruno's idea of "interviewing a machine" is a form of demonstration as I am trying to define it here. In my thesis, demonstrability requires that the model to be demonstrated is actually implemented in at least one possible physical world (i.e. satisfies thermodynamic laws and Shannon information theory) otherwise it could be used to implement a Maxwell Demon.

    BTW, it was an analysis of Maxwell's Demon that lead me to my current ideas, that abstract computation requires that at least one physical system actually can implement it. This is not ultrafinitism since I am allowing for an uncountable infinity of physical worlds, but almost none of them are accessible to each other (there exist event horizons, etc.).
    Consider the case where a computation X is generating an exact simulation of the behavior of molecules in a two compartment tank with a valve and there exists a computer Y that can use the output of X to control the valve. We can easily see that X could be a subroutine of Y. If the control of Y leads to an exact partition of the fast (hot) and slow (cold) molecules and this difference can be used to run Y then some might argue that we would have a computation for free situation. The problem is that for the hot/cold difference to be exploited to do work the entire apparatus would have to be coupled to a heat reservoir that would absorb the waste energy generated by the work.  Heat Reservoirs are interesting beasts....

If your computer simulation is acting as Maxwell's demon then you don't need a heat reservoir.  The demon makes one tank hot an the other cold so a heat engine runs on the difference.  Unfortunately this is impossible because such a simulation would require defining the initial state of the particle's position and momentum in the two tanks.  This is not available for free.  To determine it by measurement takes at least as much free energy as can be recovered after implementing Maxwell's demon.
But if you're doing a calculation once on a given machine it's not necessary to erase the result.  In Feynman's paper on quantum computing he note this gets around Landauer's limit.  So long as the evolution of the computation is unitary no energy need be dissipated.  So I don't see how the result is relevant to Bruno's UD.

Brent


    We ignore the role played by our physical world in our philosophical/mathematical/logical discussions to our peril!

-- 
Onward!

Stephen

"Nature, to be commanded, must be obeyed." 
~ Francis Bacon
--
You received this message because you are subscribed to the Google Groups "Everything List" group.

Stephen P. King

unread,
Aug 20, 2012, 1:02:41 AM8/20/12
to everyth...@googlegroups.com
On 8/19/2012 6:03 PM, meekerdb wrote:
On 8/19/2012 2:43 PM, Stephen P. King wrote:
On 8/19/2012 4:30 PM, meekerdb wrote:
On 8/19/2012 12:51 PM, Stephen P. King wrote:
I understand that 2+2 = 4.
I still cannot explain how and why I understand "2+2 = 4".
"2+2=4" is easy.
"I understand 2+2=4" is quasi infinitely more complex.

Dear Bruno,

    As I see it, the quasi-infiitely more complex aspect of "I understand that 2+2=4" follows, at least, from the requirement that many entities capable of making such statements can point to examples of 2+2=4 and communicate about such statements with each other however far away in space and time they are from each other. We can ignore the fact that there is a collection of entities to whom the statement "I understand that 2+2=4" has a meaning. You need to get a grip on the nature of meaningfulness. Searle has tried to do this with his Chinese Room idea but failed to communicate the concept. :_(

Maybe Bruno will introduce a new modality to his logic Up="Understands p".  :-)

Brent
--


Hi Brent,

    That would be wonderful if possible. AFAIK, understanding is contingent on demonstrability, e.g. I understand p if and only if I can demonstrate that p implies q and q is not trivial and q is true in the same context as p. I think that Bruno's idea of "interviewing a machine" is a form of demonstration as I am trying to define it here. In my thesis, demonstrability requires that the model to be demonstrated is actually implemented in at least one possible physical world (i.e. satisfies thermodynamic laws and Shannon information theory) otherwise it could be used to implement a Maxwell Demon.

    BTW, it was an analysis of Maxwell's Demon that lead me to my current ideas, that abstract computation requires that at least one physical system actually can implement it. This is not ultrafinitism since I am allowing for an uncountable infinity of physical worlds, but almost none of them are accessible to each other (there exist event horizons, etc.).
    Consider the case where a computation X is generating an exact simulation of the behavior of molecules in a two compartment tank with a valve and there exists a computer Y that can use the output of X to control the valve. We can easily see that X could be a subroutine of Y. If the control of Y leads to an exact partition of the fast (hot) and slow (cold) molecules and this difference can be used to run Y then some might argue that we would have a computation for free situation. The problem is that for the hot/cold difference to be exploited to do work the entire apparatus would have to be coupled to a heat reservoir that would absorb the waste energy generated by the work.  Heat Reservoirs are interesting beasts....

If your computer simulation is acting as Maxwell's demon then you don't need a heat reservoir.

 Hi Brent,

    Good point. I stand corrected! But did my remark about understanding make any sense to you? I am trying to work out the implication of the idea of Boolean algebras as entities capable of evolving and interacting as it is a key postulate of the idea that I am researching. The Maxwell Demon is just a nice and handy toy model of this idea, IMHO. Could the Maxwell Computational Demon "understand" what it is doing? We could add the capacity to have a self-model as a subroutine and thus a way to gauge its actual efficiency against a theoretical standard as a way to implement a "choice" mechanism... See http://www.youtube.com/watch?v=ehno85yI-sA for a discussion of this self-modeling idea.


The demon makes one tank hot an the other cold so a heat engine runs on the difference.

    Yes, the demon would act in a cycle: Compute the simulation to operate the valve to segregate the hot from cold and then use the heat engine to charge a battery, discharging the difference in temperatures. Can this run forever? No, given real world things like friction and the wearing out of parts, but in the idea case it might seem to be able to run for ever.

Unfortunately this is impossible because such a simulation would require defining the initial state of the particle's position and momentum in the two tanks.  This is not available for free.  To determine it by measurement takes at least as much free energy as can be recovered after implementing Maxwell's demon.

    The idea case would shift the initial position/momentum question into a synchronization question: how is a measurement different from the "inverse" of a simulation? I do not have any good words to express my thought here... Let's see where the discussion takes us.





See http://www.nature.com/news/the-unavoidable-cost-of-computation-revealed-1.10186 for more on this.

But if you're doing a calculation once on a given machine it's not necessary to erase the result.  In Feynman's paper on quantum computing he note this gets around Landauer's limit.  So long as the evolution of the computation is unitary no energy need be dissipated.  So I don't see how the result is relevant to Bruno's UD.

    The reversibility argument only works if there is sufficient black memory to work with such that erasure never is necessary. This is just trading off the recource of energy for the resource of a read/write medium. Given the wearing out of parts situation, could this be dealt with so that it is not a problem for the idea case aka no friction, no loss of heat to an external world...

See also http://www.csupomona.edu/~hsleff/MD-power-time.pdf for a nice discussion...

    I am trying to met Bruno half-way in his COMP idea... I just can't let go of the apparent necessity of actual physical implementation, even given that I really like his immaterialist hypothesis. It is too much like Leibniz' PEH and its reliance on the logically impossible. How is Bruno's idea not a proverbial floating castle in the sky?
   


Brent

Stephen P. King

unread,
Aug 20, 2012, 1:42:48 AM8/20/12
to everyth...@googlegroups.com
On 8/19/2012 12:39 PM, Craig Weinberg wrote:
You don't really 'come to the idea' at all though, you assume it from the start. There is no theory for why or how numbers would dream, only the assumption that they do.

[BM]
Here I absolutely disagree. The theory is that I am a material machine. The conclusion is that matter is an hallucination, yet a lawful one. It looks like you have not yet take a deep look on UDA.
[Craig]

I conclude that matter is a hallucination also, but not our hallucination. Matter begins as the (shared, intentional) hallucination of our molecular subselves. The key though, is that the extent to which our experience is distant from our molecular sub-realities is directly proportional to the realism and involuntary nature of our experience with matter.

Dear Craig and Bruno,

    This just might be a way of dealing with the "substitution level" question if we go a bit deeper to the atomic! If we think of the material world as defined at the atomic level and consider them as the basic entity having a model logic ala comp, Craig use of electromagnetic forces would be even more justified. We could use a physicist's thoughts on this...



It's confusing to me when you say that we are a material machine, yet matter is a hallucination, so that means we are a hallucination machine - which is ok by me, but why bring matter into it at all? What makes some hallucinations into matter?

    I answer this question by noting that if multiple entities have "hallucinations" that are in a relation such that there exists a transformation between them, such as what we see in a diffeomorphism between coordinate systems, then a "common world of matter" seems to inevitably follow. (This is what the bisimulation algebra that I worked on seeks to formally represent...) The key is to think of the properties involved. What are the properties of matter that give it its "substance"? Electrostatic repulsion at the molecular level plays a huge role...
    What would be the logical dual of this effect? Pask considered this in his claim of "No Doppelgangers" http://en.wikipedia.org/wiki/Gordon_Pask#No_Doppelgangers


My view is that the same thing that gives the hallucinations significance (makes them more narrative and eidetic, more pull toward gestalt coherence) has a symmetric exhaust in the form of entropy...which is space.

    Space. as I define it, is the multiplicity of possible places that an object could be located. Time is the common perceived sequencing of events. But what is pushing the events to change in the first place? Why not just a frozen static relation? Some people have pointed to a deep level inconsistency, like the omega-inconsistency idea.
http://www.mathresources.com/products/mathresource/maa/omega_inconsistency.html

"omega inconsistency,
n. (Philosophy) the apparent paradox that occurs when the principle of induction fails, that is, when it is not possible to infer from the fact that each element of a domain has a property that all of them have it. It is so called since the paradigm case is that of the finite ordinal numbers, each of which has a finite successor while they clearly do not all have a finite successor as the set of finite ordinals is the smallest infinite ordinal, omega. A philosophical example, due to Russell, is that it is part of the concept of desire that one wants each of one's desires to be satisfied, but amongst those desires is the apparently inconsistent desire to face new challenges, that is, to leave some of one's desires unsatisfied; thus one can satisfy any of one's desires but not all of them. This paradox seems best resolved by observing a change of the scope of the universal quantifier."

This is a nice article on the subject: http://beitiathustra.wordpress.com/2005/11/30/brief-explanation-of-omega-inconsistency/

    A change in scope of the universal quantifier is still a change that has to be accounted for. What is change coming from? What if []<> itself is globally inconsistent? My dear friend Prof. Hitoshi Kitada wrote on this:

from http://arxiv.org/pdf/physics/0212092v1.pdf

"The class φ is the first world, the Universe, which is completely chaotic. In other words,
φ is “absolute inconsistent self-identity” in the sense of Kitarou Nishida, whose
meaning was later clarified by Ronald Swan in the form stated above. In this clarifi-
cation, φ can be thought “absolute nothingness” in Hegel’s sense.
The Universe φ is contradictory, and hence its truth value is constantly oscillating between
the two extremal values or poles, truth and false, or +1 and −1, or more generally, inside
a unit sphere of C. Namely, the class φ as a set of wff’s of the set theory S is countable,
but the values which the elements of φ take vary on a unit sphere. In other words, the
Universe φ is a stationary oscillation, when we see its meaning."

    This has implications for the Measure problem!

It is space/entropy which provides Cantor-set-like statistical gaps in which the various layers of realism can break down in peace. The gaps are where the dreams hide their unrealism and forgives their continuity errors. The world of matter looks like it makes perfect sense, but to our molecular selves, it may be a timeless chaos of conflicting orders.

    Indeed!!!!

Roger

unread,
Aug 20, 2012, 5:29:11 AM8/20/12
to everything-list
Hi Bruno Marchal
 
According to the Bible, belief is a product of faith or trust, and that trust
does not come from you, it is a gift from God. We have nothing to do with it,
at least that isa what we Lutherns believe. 
 
 
8/20/2012
Leibniz would say, "If there's no God, we'd have to invent him so everything could function."
----- Receiving the following content -----
Time: 2012-08-19, 08:26:10
Subject: Re: The I Ching, a cominatorically complete hyperlinked semanticfield(mind).

--
You received this message because you are subscribed to the Google Groups "Everything List" group.
To post to this group, send email to everyth...@googlegroups.com.
To unsubscribe from this group, send email to everything-list+unsub...@googlegroups.com.

Roger

unread,
Aug 20, 2012, 6:54:22 AM8/20/12
to everything-list
Hi Stephen P. King
 
The modal logic needs to aim purposefully toward the "best possible" solution.
And contain absolute as well as contingent truths. Thus there must be
some sort of mereology involved in the modalities. Maybe a new type of copula
insuring this situation to hold ?
 
8/20/2012
Leibniz would say, "If there's no God, we'd have to invent him so everything could function."
----- Receiving the following content -----
Receiver: everything-list
Time: 2012-08-20, 01:02:41
Subject: Re: Reconciling Bruno's Primitives with Multisense

Richard Ruquist

unread,
Aug 20, 2012, 11:18:57 AM8/20/12
to everyth...@googlegroups.com
Roger,

Divine selection and natural selection are sourced, 
however at differing levels of information integration,
in the "universal CYM monad subspace".

Belief can also be a product of science.
I believe science.
Richard

To unsubscribe from this group, send email to everything-li...@googlegroups.com.

Richard Ruquist

unread,
Aug 20, 2012, 11:41:02 AM8/20/12
to everyth...@googlegroups.com
My belief based on string theory is that monad logic gets applied to produce the best possible world at the level of quantum particle interactions where the best of several quantum states is chosen in every interaction in the universe by the ,monads.
Richard Ruquist PhD

--

Stephen P. King

unread,
Aug 20, 2012, 11:51:24 AM8/20/12
to everyth...@googlegroups.com
On 8/20/2012 6:54 AM, Roger wrote:
Hi Stephen P. King
 
The modal logic needs to aim purposefully toward the "best possible" solution.

Hi Roger,

    But the "best possible" can only be defined infinitely (and thus impossible to know) or finitely in a error-prone or approximate way.

And contain absolute as well as contingent truths.

    I agree.

   
Thus there must be some sort of mereology involved in the modalities.

    Yes. The actuals are mutually consistent aspects or modes of the possibilities. The key is the frame of reference of the observer. There is no finitely knowable 3p, there is is only finitely approximative 1p. Thus we choose a point of view tat allows for measurement/observation that can be converted into communicable representations. This is the canonical form!


Maybe a new type of copula insuring this situation to hold ?

    Copula? http://en.wikipedia.org/wiki/Copula ? Please elaborate...

Bruno Marchal

unread,
Aug 20, 2012, 1:03:55 PM8/20/12
to everyth...@googlegroups.com
And there are many notions of convergence. Searching a job in England
Ramanujan, just to show his ability to compute, said that he could
compute the sum of all the natural numbers 1 + 2 + 3 + 4 + 5 + ...
which gives:

-1/12,

of course :) (*)

The crazy thing is that when you compute mass of a photon in string
theory, you are naturally lead to a sum of two terms, the first one
giving 1/12, and the second being 1 + 2 + 3 + 4 + ...

Bruno


(*) It is the value of the analytical continuation of the Rieman Zeta
function on -1. But it follows also naturally from convergence
criteria not involving the zeta function. Zeta(s) is the sum of all 1/
n^s, with n natural number ≠ 0, and it is equal to the product of all
1/(1-1/p^s)) with p primes, by a famous relation of Euler.


>
>
> Saibal

Bruno Marchal

unread,
Aug 20, 2012, 4:30:22 PM8/20/12
to everyth...@googlegroups.com
On 19 Aug 2012, at 18:39, Craig Weinberg wrote:

 

On Sunday, August 19, 2012 7:57:32 AM UTC-4, Bruno Marchal wrote:


I understand that 2+2 = 4.
I still cannot explain how and why I understand "2+2 = 4".
"2+2=4" is easy.
"I understand 2+2=4" is quasi infinitely more complex.


 
To me, objects ÷ space subjects * time = realism is easy also.


You are lucky. 





You don't really 'come to the idea' at all though, you assume it from the start. There is no theory for why or how numbers would dream, only the assumption that they do.


Here I absolutely disagree. The theory is that I am a material machine. The conclusion is that matter is an hallucination, yet a lawful one. It looks like you have not yet take a deep look on UDA.

I conclude that matter is a hallucination also, but not our hallucination.

1) in which theory. I am afraid you have no idea of what is a theory if you find easy what you quote above.
2) who said that matter was our hallucination, and what do you mean by "our". usually I mean by "our" either the universal machine (in the sense of Post, Church, Turing, ...), or the Löbian machine, which are universal machine who know somehow that they are universal.




Matter begins as the (shared, intentional) hallucination of our molecular subselves.
The key though, is that the extent to which our experience is distant from our molecular sub-realities is directly proportional to the realism and involuntary nature of our experience with matter.

It's confusing to me when you say that we are a material machine, yet matter is a hallucination, so that means we are a hallucination machine - which is ok by me, but why bring matter into it at all?

Computationalism reduce the problem of quanta and qualia into a theory of "hallucination" if you want. A term like "molecules" must eventually be justified by number's dreams statistics (to be short, but this can be explained entirely from "2+2=4" and the like). 




What makes some hallucinations into matter?

UDA is supposed to explain exactly that. 




My view is that the same thing that gives the hallucinations significance (makes them more narrative and eidetic, more pull toward gestalt coherence) has a symmetric exhaust in the form of entropy...which is space. It is space/entropy which provides Cantor-set-like statistical gaps in which the various layers of realism can break down in peace. The gaps are where the dreams hide their unrealism and forgives their continuity errors. The world of matter looks like it makes perfect sense, but to our molecular selves, it may be a timeless chaos of conflicting orders.

Like molecules, space and entropy must be justified from an evaluation of the computationalist first person indeterminacy on all computations.



 



 
 


It seems shrouded in obfuscating self-reference. Why would anything that has been encoded ever need to be decoded if the machine can fluently process the encoded form?

To store what we learn. The DNA plays already such a role at the molecular level. It illustrates also a digital encoding and decoding. 

Why would storage imply any kind of encoding? Without physics to constrain resource requirements, there would be no need to compress information.

The numbers do that all the time. There is no need, but they cannot not doing it, because it follows from addition and multiplication. There is no need for Saturn's ring either.

What I am saying is that the reason they do it all the time is that numbers are different than you assume. They have no existence that is independent from private time and public space. There is no Platonia, only the sense of the whole which resides inherently in every apartness.

In your non comp theory perhaps. I see inspired and well written literature, not theory in the usual sense of the word. 




 



I can't think of any reason that a Turing machine would need to create an abstraction layer, especially if you could control the speed of the tape. If for any reason you needed subroutines to synchronize, you could run those parts of the tape faster or slower - infinitely fast if you like. Without physics, time is relative and unbounded. A Turing machine needs no programming language, it doesn't even need bytes. You can do all of your addressing on the fly with dynamically sized memory chunks. All of these formalisms, codes, architectures, etc are purely in the service of the limitations imposed by physics and human convenience.

Not if you bet that we are machine. In that cse the UD reasoning shows that what you describe is only the appernt view from inside, but that eventually the physical resource constraints arise from the mathematical computer science constraints, that is, from arithmetic.

Why would any constraints arise purely from arithmetic that would be resolved by encoding?

It formulates the problem, or a big part of the problem. It does not solve it. 
That is the interest of comp: it transforms a problem in philosophy or theology or metaphysics into a problem in arithmetic. A *very* difficult problem. That is why I added AUDA, which illustrate the beginning of a solution, and which provides already a lot of information. We can say that today, the known laws of physics (quantum mechanics) saves comp from an easy refutation in the manner of the UDA. 



 



We get tired of writing a thousand lines of code every time we want to transform some dataset, but the computer doesn't care. It will run a trillion lines of code just to add the same number to itself, it doesn't care how long it takes or how wasteful it seems to us.

Not with comp, as I am a person which manifest itself through a computer, and persons care.

Comp begs the question.

It would if it was an answer, but it is a question. It beg the question by definition. It is my working hypothesis.




At what point would a computation start to invent 'care' as a meta-computation?

Good question. But you don't need to understand the comp problem.
Then AUDA answers that question, but it is a bit technical. Read perhaps the sane04 paper. Part 1 and part 2.




If there were arithmetic resources to consider, surely the addition of encoding, decoding, and compiling  would use more resources rather than less.

It is build in. If not addition+multiplication would not be Turing universal. You should study a bit of computer science and mathematical logic to get this. 










Why would it need any other form - especially if it is all made of numbers?

Nobody needs a universe. Why do we do babies?
The "truth" is that we have them, we cannot really avoid them. It is like the prime number and the universal machine. Once you have zero and + and *, you get Platonia, and a lot of mess in Platonia. It is a logical consequence.

But a universe is not a consequence of Platonia.

It is. Or your non-comp assumption is correct, but I do not work in the theory non-comp. Actually, I have not yet seen a non-comp *theory*. Only philosophical argument against comp, but no concrete replacement.

My theory is not non-comp, it is the reconciliation of comp and non-comp. Part of it is to explain why any theory by definition can only apply to comp.

That is false. Non-comp theories exists (mathematically, nobody has really found one, still less used to explain where consciousness and matter come from). Also AUDA can be applied to most known weakening of comp, so you have to go very far in the negation of comp to get a non comp explanation of the origin of matter.





As Goethe said, “My friend, all theory is gray, and the Golden tree of life is green.”

A good reason to distinguish theory and what is the theory about.
It reminds me also of the short formulation of the mind body problem: how could a sleeping gray brain in the dark generate a colourful dream?






Since we know that we have a universe

We know only that we are conscious. How could we know that there is a (physical) universe? 

How could we not know that there is a physical (part) of the universe? We know it with the same certainty that we know 2+2=4.

I doubt this.



We can tell when we finally do wake up in the truly public space of the universe, that it is a different experience from even the most convincing dream.

I doubt this.



 
We don't even *know* that there is a mathematical universe, or even an arithmetical universe. We bet on it.


Then that relativistic bet becomes what we mean by *knowing*.

I use knowing in the classical sense, even in the Theaetetus sense. Knowing p is believing p in the case where p is true (by chance, by logic, by reason, by habit, whatever).

Here, to reassure Stephen, comp saves knowledge from occasionalism, is far as we bet the measure problem has a solution (but AUDA is technically promising that solution). 



There is only seems like, and physics is a seems like which pretends to be the opposite. Matter seems like it 'simply is.'






that means that it is Platonia which needs to be justified intellectually in terms of contributing to the universe, not the other way around.

But with comp, we do got an explanation of the physical universe(s). And we got both quanta and qualia. It might be false, but the explanation is there.

Haha, the 'it might be false' part doesn't concern you?

It concerns everybody, especially in front of a doctor, and perhaps a priest or shaman. 
We cannot know the "public truth", we can only propose theories, reasoning and experience, together with interpretation of theories, nothing is easy, in the public domain, especially when close to the very fundamental question (usually hot and full of wishful thinking).


Shouldn't the fact that it doesn't concern you concern you?

What do you mean by "it does not concern me"? Being false concern all honest people communicating.



But I don't see that comp gives you qualia. I don't see red or pain, just ineffable namespaces.

You are just confusing the terms of the theory with what those terms are supposed to describe.



A hole where qualia can be inferred, but only if we already have the example of qualia to plug into the explanation.

Exactly. Bravo. You get the point. And the beauty of it is that is what the universal machine can already explain.

What you ask for is not a theory of qualia, it is the qualia themselves. Of course that is different, like a theory of numbers is not the numbers too. 

If you want qualia you have to quit the chair and go to the movies or live adventure in virtual or physical realities.




 



 



What I am saying is that if you are going to invoke a possibility of dreams, that has to be grounded in the terms that you are laying out as primitive. Why would dreams leap out of mechanical relations?

It is a logical consequence, once you accept the idea that you might survive with a digital brain.

It's the logic of begging the question. I ask you why numbers dream and you say it's logical if you accept that dreams can survive as digital process. I don't though. If human dreams could exist in something other than humans, then they would already.

And that is the case already, indeed. That follows from the reasoning.

Where else do human dreams exist except in human experience?

Correct. Now, in the comp theory, human experiences are particular case of a much vaster set of experience. The X dreams don't exist but in the X experience.




 



On the sands of the beaches, in the odd complicated cloud formation. I would need a compelling reason to believe otherwise. Why would I give the benefit of the doubt to machines when machines have clearly shown no signs of feeling thus far?

But molecular biology, quantum mechanics, etc. suggest strongly that we are machine. The rest is the logical consequence.

They suggest a mechanistic level of what we are, but given that consciousness can't be detected externally even at the macrocosmic level, there is no hope that any other level besides the lowest quantitative mechanistic level would be revealed.

Again that is a theorem in comp. No machine can ever be sure on its level, and of what is really necessary to keep for remaining oneself. 



You can't find irony and love with a microscope. The logic is biased because it assumes a voyeuristic access to reality rather than an integrated perception of multiple subjects.

Again you are right, but mechanism prevent the normative description of one subject, and still less on many subjects, be them machine or human. So it just generalize you are idea of an integrated perception subject to the numbers, it makes physics the integration process allowing that first person plural dream sharing. There is also a cosmology, witnessing the depth of our computations, and a physics, witnessing a curious bottom linearity. Comp and computer science makes precise links there. 

To be conscious one second, you need to be a deep number,
To be conscious two second, you need a linear bottom,
That's what the universal machine already say, actually. 

In precise theoretical terms, even arithmetical, comprehensible by any schoolgirl or schoolboy, patient enough.




We are machine, but we are also the opposite of machine,

I can understand the tension.
But the ideally self-referentially simple correct machine cannot avoid that tension too (in comp + classical theory of knowledge).

When we say "I am a machine (ate level x)", we talk on the 3-I, it is necessarily, a view of oneself in a (locally) third person way, or (globally) a first person plural way. 

From the point of view of the 1-I, the 1-I is not a machine. The machine cannot even name or define it.




perhaps even a machine that specifically amplifies the opposition to mechanism to a higher degree than any other.


Hmm... You need to preserve some balance between the left and the right brain. We need both Bp and Bp & p. 
Those distinction justifies the use of art to fake, succesfully for the great artist the communication of the non communicable qualia.





 
There is no evidence for non-comp.

We are the evidence for non-comp. Not our bodies but our individuality and creativity, humor, passion, initiative, etc. The microbiology is the boring part. The interesting part is impossible to see directly because it is what we are, but it can be understood by inference of symmetry.

OK. It really looks like the Bp and Bp & p distinction. 




 
So we do have compelling reason to think that machine can have feeling. The conjunction of our own experience, and the study of our bodies.

You have a compelling reason to think that, but I see the clear error of painting ourselves into a corner based on faulty assumptions. You assume that our being identical with our body means that who we are must be reduced to body-like process, whereas the fact that we can choose to change the chemistry of our brain, move our voluntary muscles, etc, tells me that there is another side to the story: The body must also be seen as a reduced shadow of processes which are us. If we are made of neurons, then neurons, at least to some extent feel and see also,

Not necessarily. It happens that I am open that neurons can already think, as all protozoa, who are quite sophisticated mechanism, but I am open for the contrary of this, I don't know. If comp is correct, at the substitution level our consciousness is independent from the "motivation" of the elementary units of action.



and if neurons feel and see then molecules do something similar.

They interacts, but unless you talk about a quite sophisticated macromolecules, molecules does not self-represent, unlike DNA, Cells, ... universal machines.

If not you have to continue and declare the electron and the proton conscious, and then the quarks and then the strings and the membrane, and all along I am waiting for the relation with who I am etc. 





The fact that subjectivity is private by nature makes it unsurprising that we would be reluctant to consider that this is the case, especially since the body is the comp-space-matter extension that evacuates all appearance of subjectivity. Subjectivity does not survive objectification, which means that subjects can't be reduced to objects.


Absolutely so. The person cannot be reduced to its body or bodies. Eventually the contrary happen, bodies are statistical pattern in number hallucinations. 

Subjectivity cannot survive objectification, unless they are presented as choice of manner of life, like saying "yes" to the digitalist doctor (which is more a right than a truth).




But I don't want to defend comp. That's the kind of philosophy I do not work on. I just show the consequence of being material machine. The consequences are that we are already in a "matrix", whose structure can be mathematically handled, so that we can constructively derive physics, and compare it to the empirical physics, so that comp is shown testable. (To be short).


Not just that they might not have feelings, but that they are universally known for lacking any capacity for feeling whatsoever - that is what defines the nature of machines. When you understand the continuum of eidetic to entopic, figurative to literal, you will understand why this makes perfect sense

But what you say makes sense. The comp theory already explains why comp is hard (impossible, even)  to believe, and why it has to be highly counter-intuitive. 

Comp isn't hard for me to believe at all,

This means that you are either unsound, or inconsistent. But take it easy as I use "believe" in a very restricted sense. But comp is like QM, the more you study, the more it is unbelievable. 



I just think that if we play out the axioms of comp alone, we do not wind up with realism or a universe. 

At first sight it is much like a multi-dreams. Does that percolate into a multiverse, or a multi-multiverse? Open question.


We have to begin from our whole experience and make sense of arithmetic in that context and not assume that arithmetic can be its own independent context (because why would it need or want any other?).

You talk like Plotinus talk about the ONE. 
The ONE is supposed to be perfect, so why would it need or want any other?

But here the answer is that arithmetic a priori need nothing indeed. It is the person supervening on infinities of complex arithmetical relation which does the thinking.

I have no doubt that the number 2900000007800021 is either a prime number, or not, and that such a fact is independent of me. 
I thought you agree on this.







A machine is an assembly of logical forms. That does not produce any phenomenology by itself because it is constructed from the outside in. Living organisms build themselves from the inside out, from their own native sensitivities and motivations. They are not having an alien script intentionally imposed upon their behavior.

I don't know that. Genetic illustrates that nature do scripts. But math shows also that very short scripts, like "help yourself" can lead to tremendous richness and complexity. 


Nature can do scripts, but that doesn't mean that the scriptee has the level of awareness used by the scripter. Nature is full of exploitation.


Arithmetic too. Not all numbers are perfect.



 


 



Even if there was some purpose for it, how could that actually take place - what are the dreamings made of?

Ontologically: nothing take place. All the computations are there. Some emulate self-observing machine and the math explain why they have to be beffudled by existence and conscience.


That may be, but why in the world would computational befuddlement be expressed as personality and realism?

Because there is, very roughly, 60000^10000000000 brain states possible, that makes a lot of subjectivity possible, and they are multiplied and selected through a filtering on a continuum of computations (first person indeterminacy). 

That's a naive anthropomorphizing of computation though.

No, it is the consequence of the UDA. 



What does Platonia care for 'a lot'? It has infinity and eternity.


OK. Platonia does not care. But in the neoplatonist theory, Platonia is the second GOD. Not the first one, the ONE, not the third one, the inner God.

An there are tension indeed.   Eternity and infinity is not always enough to solve problems.


 






My view is that it may be the case that everything that is not matter across space is experience through time - by definition, ontologically. There is no other form or content possible in the cosmos. Numbers are experiences as they must be inferred by computational agents and cannot exist independently of them. What my formulas do is to propose a precise relation between dream-time (including logical algebras) and matter-space (including topological geometries). To do this we need to invoke a continuity between them which is a perpendicular axis which runs from the literal (tight equivalence; induction is accomplished through linear arithmetic logic) to the figurative/metaphorical (loose thematic association; induction is accomplished through linear logic as well as elliptical cross-context leaps).

I don't believe in time, space, cosmos, matter, ...
I explain their appearance by the dream property of numbers, relatively to universal numbers.


That's what I am showing you, is how the dream property of numbers (eidetic-figurative) and the number property of dreams (entopic-literal) are two polar regions of the same (involuted) continuum.

I am a simple minded logician. If you use a technical term, you have to explain it to me in a precise frame theory.
I don't ask you to formalize it in first order logic, but to succeed in explaining this enough precisely so that a poor logician like can understand that with time and patience we can get such a formalization. If not it looks like you can say anything you want, and very often, it makes sense in machine first person terms, and thus fail to convey your non-comp intuition more than the natural non-comp intuition of *all* correct machine.
Comp shows intuition locally correct, locally useful, locally unavoidable, but globally wrong.

Likewise non-comp shows the logic of 'correct and useful' to be trivially valuable but profoundly inadequate. We can use different terms, as long as one side is literally figurative (numbers are actually figures)

Hmm...


and the other figuratively literal (dreams seem like reality to some degree). I keep throwing out different terms, which ones are you comfortable with?

Oriental - Western.

OK.


Subjective - Objective.

OK


Algebraic-Topological.


I guess you meanTopological-Algebraic.

You could add:

Feminine-Masculine

Yin-Yan



Dreamtime - Spacetime.

Nope. I see what you mean, but time and space occurs differently. It is related to the duality above, by adding something. (It is an octalism, not a dualism). More later perhaps. 


Interior - Exterior.
Proprietary - Generic.
Signyfing - Generic.
Personal - Universal.
Private - Public.
Eidetic - Entopic
Narrative - Scenic
Portrait - Landscape
Vertical - Horizontal
Solipsistic - Mechanistic
I - it

ℵ-Ω

ॐ - (ॐ - ॐ)

 
What I am saying though is that *any quantitative treatment of qualitative experience is an unrecoverably catastrophic reduction*.

I agree with this. Comp prevents to do such a reduction about what a machine can be and can be capable of.
 
 

Ok, but we have to connect that irreducible quality with quantity, and I think we can do that with this symmetrical continuum of literal figures and figurative literacy.

Perhaps. But you should do it, then, or convince others to do it.


I think I am doing that. I don't know that others can be convinced though, they have to convince themselves.

Personnally I find something understandable if you can explain it to my fourteen years old niece, or better to any (enough rich) universal machines. 




 



Modal logic can make a toy model of experience, by removing all of the experience and replacing it with a meaningless variable.

Why meaningless?

Because that's what variables do, they generalize all potential content-meanings under single quantifiable term.

Hmm... OK. But the person is in the box (B), not in the variable, which is usually used for some unspecified arithmetic proposition, or memories content, etc.

If the box is B and the person is in the box, then the person is in B,

No, the person is referred through that B. Like Obama is referred through the string of symbols "Obama"/ You will not confuse Obama and "Obama", OK? 



which is a variable containing no possibility for content (other than to refer to a potential for content...B stands for a menu, not a meal).

B stand for my sun in law going to look at that menu.


 



I don't really understand what modal logic has to do with the possibility of quality.

The machine modal logic of self-reference, Bp, when conjuncted with the non nameable Truth, leads the machine to discover true propositions, in a sort of immediate way, which they cannot justify nor even describe, except by projecting them on similar machines. 
The redness quality is like that. I have no doubt that I experience redness, but I am unable to communicate it to a blind rationalist. I can point on something red, and hope my fellow has a sufficiently similar experience, so that the word "red" will have a sufficiently close meaning to mine. 

The incommunicable nature of qualities like red is not sufficient to point to anything like red though. Water is wet, but so are a lot of things. Wetness doesn't make it water and ineffability by itself doesn't generate an explanation for qualia.

Sure. But they are not just ineffable, they obey precise laws, and a part of them are the quanta, so we can test the theory. That's the point. 


 
 

 


What quality does this modality have? Is it shiny? Is it spicy? Does it get tired?

The modality has no quality. It describes qualities. You might do a confusion of level. It is a bit like the confusion between the string "s(s(0))" and the number s(s(0)).

 
How is it describing qualities? It seems to me to be describing quantitative formalism. How does one describe red logically?

By explaining it exists necessarily and that it cannot be describe logically. Modal logic, in the machine context are "meta". It does not explain the quality red. It explains that quality exists and are NOT describable quantitatively. 

The "universal soul", Bp & p, or the S4Grz logic, is a sort of mathematical tour-de-force: it is a formal logic describing the logic of content which are impossible to formalize ever. But we can formalize this at the metalevel.


 
But it formalizes red out of existence. It makes it #FF0000.

Not in the first person view. 

On the planet Nuptar, in a galaxy nearby, life evolved through silicon, due to harsh condition, but then they changed a bit and the siliconic Nuptarians discovered that they could make computer with organic molecules, and indeed they eventually build brains. Nuptarians where divided on the question, somesaid that would formalize red out of existence, it makes it <# neuronal circuits>.

You confuse again the theory and what it is supposed to describe. 

A part of the confusion is that a brain too is a biological sort of theory, and the pure qualitative part of the experience of the brain is indeed not reducible to its lower elementary part.


That is true for many machines.





Well, if I remember well, you are the one who refuse to sell a steak to my daugher's husband, who said "yes" to a doctor, and is, by construction, a machine.

I don't deny that we are machines, only that we are entirely machines.


That makes sense. Bp & p is still a machine, in a sense, but only in the outer God eyes, not in the inner God eyes. 

IF 3-I is a machine (comp), then the 1-I will not feel to be a machine, at all.



My point is that we can just as easily be described as non-machines with machine-like tendencies as the other way around. The symmetry between the two and the sense between them is what I am interested in.
 




While I admit, certainly, to being ignorant of the particular details of mathematical principles which give you a more generous view of machine consciousness than I have, I don't limit the capabilities of machines in the way that you assume. If quanta is flat qualia,

Quanta are more sort of sharable qualia.

I agree, but what I am adding to that is that the more sharable they are, the less qualitative they have to be by definition. If you want the most universal truths, they must be shallow and meaningless. If you want deep and meaningful truths then they must be personal (to match up with a lifetime's worth of personal associations and experiences).

I don't think so, as we are very close. Like the biological genome can already copy the body at some level, you can share deep and meaningful emotions and qualia by providing a link to the work of an artist who moved you. Eventually it is a personal, but the universal part is sharable, and can propagate more easily the personal things. We (the universal numbers) play that trick again, and again.




 
First person plural construct brought by the many-world internal interpretation of arithmetic by arithmetical beings.

 I think MWI is a plug for the inadequacy of comp. With sense, you don't need to formally execute all possible computations, you can use elliptical (...) nesting to fill in for infinities and therefore retain the coherence of a single-world interpretation,

I will be short. I have never believed in a single world interpretation. The term world is to fuzzy also, in this context.

keep in mind that "I" (the one who fake to believe in comp) believe only in the existence of 0, s(0), s(s(0)), etc. And then some laws. Worlds are number dream extrapolation and it is an open problem if there are 0, 1, 2, ... or continuum, or more "physical worlds". 




or as I call it, a Sole Entropy Well (single source of almost zero entropy - pure signal-ness, which perpetually 'floats on top' of all subordinate causes and conditions...like God, but without the beard.)




then machine intelligence can extend infinitely farther than organic intelligence, because of the horizontal universality of it.

Yes, indeed.



What I am saying is that you can't build qualia out of quanta,

I agree with this. Qualia emerges logically before quanta, but it seems the contrary. That can perhaps be related to Stephen-Pratt reversal of arrows in the mind body relation, or to the duality between thrid person (Bp) and the Theaetetical first person (Bp & p).


Sounds promising?

I asked Stephen if he think that the Pratt duality applied to the and -> or transform from the passage 3-view -> 1-view. There are many dualities possibles and there is a danger of confusion.




 


because there is no universal qualia.

This I doubt. More from personal reasons than comp, though.

Universal qualia I suppose is qualia of the unity/symmetry between qualia and quanta.

It is symmetrical and harmonic for the p, Bp, Bp & p modalities, but the harmony break when the soul steal a secret of Platonia, Dt, and become Bp & Dt & p. 



 


Qualia is othrogonal to quanta - it is the manifestation of non-universality itself.

Well, with comp, quanta is the tiny emerging part of the qualia-consciousness part of the arithmetical platonia, seen from inside.

May be the crux of our difference (beyond comp/non-comp) is that, as many people, you seem to believe that lind and matter are symmetrical and have same caliber. But comp is still open too full platonism, in which matter, and physical universe are more like a divine bug, a temporary misleading; hding the truth more than paving a way to it. Some platonist identify matter with bad, or the devil. this is of course an open problem with comp, but what can already been argued for is that the mindscape is vastly bigger than the physical scape. The comp mindscape or personscape is vast. *Very* vast. It escapes the math. It violates physicalism, but also mathematicalism, despite the ontology is quite little (a tiny segment of arithmetical truth). 

Bruno


I think of matter as bad i


Open problem for me. I don't know for the universal machine. There are relations indeed, but here the antic platonist might be a bit naive, I don't know.  Matter is the indeterminate, for Aristotle and Plotinus, and arguably for the Ums and Lums (universal and löbian machines).

Bruno



Bruno Marchal

unread,
Aug 21, 2012, 5:58:57 AM8/21/12
to everyth...@googlegroups.com
On 19 Aug 2012, at 21:51, Stephen P. King wrote:

On 8/19/2012 7:57 AM, Bruno Marchal wrote:


I understand that 2+2 = 4.
I still cannot explain how and why I understand "2+2 = 4".
"2+2=4" is easy.
"I understand 2+2=4" is quasi infinitely more complex.

Dear Bruno,

    As I see it, the quasi-infiitely more complex aspect of "I understand that 2+2=4" follows, at least, from the requirement that many entities capable of making such statements can point to examples of 2+2=4 and communicate about such statements with each other however far away in space and time they are from each other. We can ignore the fact that there is a collection of entities to whom the statement "I understand that 2+2=4" has a meaning.

The idea was that "2+2 = 4" can be "explained" in few lines, and refer only to the very simple objects 0, s(s(0), +, *.
But "understanding "2+2=4" refer to a thinking machine, and so refer to two universal numbers (brains, computer) in addition to "2+2=4". 


You need to get a grip on the nature of meaningfulness. Searle has tried to do this with his Chinese Room idea but failed to communicate the concept. :_(

Searles makes an obvious confusion of level of description, as already well understood by Dennett and Hofstdater (cf "Mind's I").





snip






The dreaming number are usually very big concrete number. They dream  
by encoding computational state of person, relatively to some  
universal number, which are encoding universal machine relatively to  
some other one, and the initial one can be chosen arbitrary. Those are  
not symbolic number, but real encoding number, a bit like the genome  
if you want.

Why would that result in a dream?

Because I work in the comp theory where we come to the idea that consciousness can be manifested by abstract relation between numbers, as they emulate computation. We have already said "yes" to the doctor.


You don't really 'come to the idea' at all though, you assume it from the start. There is no theory for why or how numbers would dream, only the assumption that they do.


Here I absolutely disagree. The theory is that I am a material machine. The conclusion is that matter is an hallucination, yet a lawful one. It looks like you have not yet take a deep look on UDA.

    No Bruno, you do not *just* have a theory that "I am a machine".

?



There are many assumptions buried in the UDA that are ignored. One of them is that there are a pre-existing collection of distinguishable entities capable of comprehending what is an abstract theory.

This is assumed in the idea that there is a physical reality enough rich to implement a digital brain. But after UDA, we just know that for the TOE, such digital brain are entirely provided by the number relations, but that is in the result.



This includes the blackboards on which you write your "chicken-scratch" and the internet and the computers that we are using to write, send and receive these emails.

You can assume that for the decor, but that does not change the reasoning. 



There is an entire physical world that you want to ignore, mostly, I suspect, because you do not want to deal with the interaction/concurrency problem.

You keep being deadly wrong on this issue. The point is that we CANNOT assume this AFTER having said "yes" to the doctor. We can assume a physical reality, and interaction, but in fine, we have to justify it from the numbers, and this is the formulation of the problem. I refer you to Girard "geometry of interaction" which is the closer from what we get from Gödel's self-reference. 




The conclusion that "matter is a hallucination", while correct IMHO, does not eliminate the impact of this pre-existing collection and their capacities.

How could an hallucination be pre-existing? That does not make sense.





 
 


It seems shrouded in obfuscating self-reference. Why would anything that has been encoded ever need to be decoded if the machine can fluently process the encoded form?

To store what we learn. The DNA plays already such a role at the molecular level. It illustrates also a digital encoding and decoding. 

Why would storage imply any kind of encoding? Without physics to constrain resource requirements, there would be no need to compress information.

The numbers do that all the time. There is no need, but they cannot not doing it, because it follows from addition and multiplication. There is no need for Saturn's ring either.

    Ultimately one could argue that anything is "no needed" and yet such arguments are wrong headed.

    Numbers are not isolate entities! They do not "do" anything in and from themselves.

This is too vague. You are ignoring than numbers obeys laws which makes them Turing universal, so I am not sure what you mean by isolated in this context. Of course they are digital, so they have a discrete topology, but this does not isolate them. Numbers do things, like 3 divides 6, or like a universal numbers can implement a human brain, independently of anything else.




There is not even an "self" in a number alone unless we think of them as a fixed point of a class of transformations of a quasi-infinite collection. You seem to be using a Sorites paradox type of argument here to make a false point.
http://plato.stanford.edu/entries/sorites-paradox/
"The sorites paradox is the name given to a class of paradoxical arguments, also known as little-by-little arguments, which arise as a result of the indeterminacy surrounding limits of application of the predicates involved. For example, the concept of a heap appears to lack sharp boundaries and, as a consequence of the subsequent indeterminacy surrounding the extension of the predicate ‘is a heap’, no one grain of wheat can be identified as making the difference between being a heap and not being a heap. Given then that one grain of wheat does not make a heap, it would seem to follow that two do not, thus three do not, and so on. In the end it would appear that no amount of wheat can make a heap. We are faced with paradox since from apparently true premises by seemingly uncontroversial reasoning we arrive at an apparently false conclusion.
This phenomenon at the heart of the paradox is now recognized as the phenomenon of vagueness (see the entry on vagueness). Though initially identified with the indeterminacy surrounding limits of application of a predicate along some dimension, vagueness can be seen to be a feature of syntactic categories other than predicates. Names, adjectives, adverbs and so on are all susceptible to paradoxical sorites reasoning in a derivative sense".


You might explain the relevance of this. Where do I do that error?





I can't think of any reason that a Turing machine would need to create an abstraction layer, especially if you could control the speed of the tape. If for any reason you needed subroutines to synchronize, you could run those parts of the tape faster or slower - infinitely fast if you like. Without physics, time is relative and unbounded. A Turing machine needs no programming language, it doesn't even need bytes. You can do all of your addressing on the fly with dynamically sized memory chunks. All of these formalisms, codes, architectures, etc are purely in the service of the limitations imposed by physics and human convenience.

Not if you bet that we are machine. In that cse the UD reasoning shows that what you describe is only the appernt view from inside, but that eventually the physical resource constraints arise from the mathematical computer science constraints, that is, from arithmetic.

    Sure, we have evidence that our experiences of the world are not direct contact with the objects making up the world but are a form of virtual reality generated by our brain. Therefore we can make good bets that more often than not are good such that they pay off, but the computation that the brain generates that our experiences supervene upon simply does not happen in the absence of physical processing that fall under the rule of "thermodynamics".


If you prove this, then you can add a 9th step and conclude that comp is wrong. But you keep saying this without proof or argument.



There is no knowledge for free. Arithmetic is the product of many such brains interacting with each other *and* it is a relational structure that exists in its own right.

?


We cannot remove the former of this pair and imagine that the latter is left to float free and continue with all of its abilities intact.

In the non comp theory, as step 8 shows that even if there was a physical reality, it would not be related to any form of stable consciousness.




We get tired of writing a thousand lines of code every time we want to transform some dataset, but the computer doesn't care. It will run a trillion lines of code just to add the same number to itself, it doesn't care how long it takes or how wasteful it seems to us.

Not with comp, as I am a person which manifest itself through a computer, and persons care.

    Certainly, but this is because of the vast constellation of other persons that you are capable of not only betting to exist and have some particular set of properties, but also cannot just theorize out of existence. COMP does not exist in isolation. It is a concept that exists in (all possible versions of ) our records of it (which include our internal brain memories).

Again that would contradict either comp, or comp's consequence. So you are saying that comp is false, or that there is a flaw in UDA. But as long as you don't find it, you have to accept comp's consequences. Without AUDA and the quantum I would also have taken this as an argument making comp not really plausible, but with QM nature seems to follow comp in its most startling consequences, so a non-comp conclusion is premature with respect to UDA.







Why would it need any other form - especially if it is all made of numbers?

Nobody needs a universe. Why do we do babies?
The "truth" is that we have them, we cannot really avoid them. It is like the prime number and the universal machine. Once you have zero and + and *, you get Platonia, and a lot of mess in Platonia. It is a logical consequence.

But a universe is not a consequence of Platonia.

It is. Or your non-comp assumption is correct, but I do not work in the theory non-comp. Actually, I have not yet seen a non-comp *theory*. Only philosophical argument against comp, but no concrete replacement.

    I cannot speak for Craig directly, but where we have a disagreement is in the things that COMP ignores. The larger universe within which it has meaningful expression.

How does the larger universe be responsible for meaning?
Are all the interacting people living on arithmetic's border zombies?





Since we know that we have a universe

We know only that we are conscious. How could we know that there is a (physical) universe?

    Because we can kick the wall and watch our foot rebound.

If you find this convincing, we have a problem.



There is overwhelming evidence that there is a physical universe.


I agree completely with this. With comp, that is the reason of the body problem. We must justify our belief in it without postulating it. Or there is a flaw in UDA. Where?



The very act of exchanging these emails is some of that evidence!

With comp that is an evidence only for a dream sharing, or a video game, not a primitive reality. You keep confusing level and meta-levels.



Can we be certain of what it is like in-itself? No! We cannot! But we can form theories about what it might be and test these. This fact alone tells us that there is "something" that is there and it is not just the figment of my (singular) imagination.


Of course. but with comp, what is "really there" can and must be limited to the sigma_1 truth.  



We don't even *know* that there is a mathematical universe, or even an arithmetical universe. We bet on it.

    Yes, and we can do that only because of the prior possibility of being able to represent facts about that universe to ourselves and each other.

Then comp is false.





that means that it is Platonia which needs to be justified intellectually in terms of contributing to the universe, not the other way around.

But with comp, we do got an explanation of the physical universe(s). And we got both quanta and qualia. It might be false, but the explanation is there.

    Only to you is there such a complete explanation.

No. To all universal machines/numbers. 



We simply cannot ignore the fact that we are "not-Bruno" and that only "Bruno bets correctly that comp is true".

By definition, all computationalist makes that bet. And I am agnostic on computationalism. I just show that the idea that there is a physical universe cannot explain our beliefs that there is a physical universe. 




This is like the sentence: "Stephen Paul King cannot prove that this statement is true". I can only bet on it one way or another, but anyone else can prove things about it.


 


What I am saying is that if you are going to invoke a possibility of dreams, that has to be grounded in the terms that you are laying out as primitive. Why would dreams leap out of mechanical relations?

It is a logical consequence, once you accept the idea that you might survive with a digital brain.

It's the logic of begging the question. I ask you why numbers dream and you say it's logical if you accept that dreams can survive as digital process. I don't though. If human dreams could exist in something other than humans, then they would already.

And that is the case already, indeed. That follows from the reasoning.

    :-P


On the sands of the beaches, in the odd complicated cloud formation. I would need a compelling reason to believe otherwise. Why would I give the benefit of the doubt to machines when machines have clearly shown no signs of feeling thus far?

But molecular biology, quantum mechanics, etc. suggest strongly that we are machine.

    You are not just talking about particular formal theories! You are also tacitly taking into account the vast world of observations using those theories that find them to be correct.

We never find that a theory is correct.



This does not deny the suggestion that we are machine, but it does not allow a free floating abstract representation of The Machine to exist *and* do things that only the universe of interacting entities can do.

Well, if you have a problem with 2+2=4, there is nothing I can do for you. You are stuck in the level-metalevel confusion I am afraid.
That might explain why you cannot give a theory.





The rest is the logical consequence. There is no evidence for non-comp.

    This email *is* evidence! That I can coherently  (in spite of my dyslexia) understand what you might mean with those symbols) such that I can respond in kind is evidence that not-comp. But I am only focusing on the error in step 8 that makes comp fail for me, and thus make a non-comp claim.

But please show the error. You only claim that there is one, staring from a philosophical conviction. You might be right, but then use your philosophical conviction to find it. What is wrong in step 8. Besides most of what you said already contradict step seven too.





So we do have compelling reason to think that machine can have feeling. The conjunction of our own experience, and the study of our bodies.

    No no no no no! We cannot ever have any reason, it is not 1p and thus cannot be considered as knowledge! It is not possible to exactly represent the 1p, we can only approximate it. We can get very very very close to it, but we can never get "it".

Then the doctor cannot succeed in reincarnating us through a digital brain, and all you say is that comp is wrong. 



My experience of what it is like to be me is not sharable, and thus cannot be copied, coded or transferred. I can only copy, code and share representations of it. The map is only the territory when the territory is the map. Otherwise please do not conflate or confuse the two.

Comp only conflates the fixed point, with this image.






But I don't want to defend comp. That's the kind of philosophy I do not work on. I just show the consequence of being material machine. The consequences are that we are already in a "matrix", whose structure can be mathematically handled, so that we can constructively derive physics, and compare it to the empirical physics, so that comp is shown testable. (To be short).


Not just that they might not have feelings, but that they are universally known for lacking any capacity for feeling whatsoever - that is what defines the nature of machines. When you understand the continuum of eidetic to entopic, figurative to literal, you will understand why this makes perfect sense

But what you say makes sense. The comp theory already explains why comp is hard (impossible, even)  to believe, and why it has to be highly counter-intuitive.

    Only because of the very abstract math involved, otherwise it is not complicated at all! Otherwise the "explain it to your grandmother" is impossible.

Grandmother can understand that "explaining the 1p" in term of 3p is impossible, once comp is assumed, and so it needs some act of faith.



A machine is an assembly of logical forms. That does not produce any phenomenology by itself because it is constructed from the outside in. Living organisms build themselves from the inside out, from their own native sensitivities and motivations. They are not having an alien script intentionally imposed upon their behavior.

I don't know that. Genetic illustrates that nature do scripts. But math shows also that very short scripts, like "help yourself" can lead to tremendous richness and complexity.

    Umm, this is a subject that we should elaborate on all on its own. The sentence "help yourself" is not just 12 symbols in some particular order!


 

Even if there was some purpose for it, how could that actually take place - what are the dreamings made of?

Ontologically: nothing take place. All the computations are there. Some emulate self-observing machine and the math explain why they have to be beffudled by existence and conscience.


That may be, but why in the world would computational befuddlement be expressed as personality and realism?

Because there is, very roughly, 60000^10000000000 brain states possible, that makes a lot of subjectivity possible, and they are multiplied and selected through a filtering on a continuum of computations (first person indeterminacy).
    Sure, a lot of difference that make a difference can be coded with those many brain states. But you need to show why they have the particular rules that they have.

That is the easy part of comp.



We simply cannot ignore the body problem!

Indeed, that is the conclusion of UDA. It is very hard to understand you, as you repeat what I say all the times, with a tone like if I was ignoring it. That there is a body problem is the result of the investigation as UDA makes clear.
There are no other way, if you want to communicate something.





Modal logic can make a toy model of experience, by removing all of the experience and replacing it with a meaningless variable.

Why meaningless?

Because that's what variables do, they generalize all potential content-meanings under single quantifiable term.

Hmm... OK. But the person is in the box (B), not in the variable, which is usually used for some unspecified arithmetic proposition, or memories content, etc.

    No! A person is not "in the box (B)" at all! This is a map-territory conflation without explanation!

When you say yes to the doctor, you are in the brain's plan, which is given by the arithmetical probability corresponding to that B. The map-territory are confused here as it is the comp assumption. The doctor does not put a model of you in the artificial brain. It puts you into it, by assumption.







 

It is to say 'here, instead of your baby, let's just call it 'consumer of diapers and milk' and design a nursery based on the statistics derived from that consumption.

No, it is the complete contrary. I see your point, but it is eaxtly that type of reduction that is prevented by comp. Even for machine, we can no more reduce them to their third person description. They do have a soul, even after-life, etc.
 
If machines have souls, then there has to be a way that the two are connected. There has to be some kind of rationale for it.

Nice to hear that. I agree.

    As do I.




What is needed is a way to reference phenomenological qualities which expresses not merely non-comp, but explicitly asserts quality and a view of the universe from the perspective of irreducible quality.

You have to postualte them, and to postulate matter, and to postulate a relation, and non-comp, and that is neither satisfying, nor working. It introduces difficulties where there are already enough, imo.

I don't think you have to postulate matter, you only have to formalize what you already assume about dreaming numbers.

OK. That's the point. In the comp theory.

    But comp theory does not float free of the world within which it has a meaning!

Level confusion.



Just as how "17 is prime" has a unique truth value only in a world where that set of physical pattern convey a meaning. In the absence of that world there is not meaning nor truth value.

If that was true, an explanation of 17 is prime would involved a physical reality. it does not. You are confusing level of explanation.





 



> We can’t pretend to talk about the eidetic, dream like  
> perpendicular of number logic while using the purely empirical terms  
> of arithmetic reduction. We need symbols that can only refer to  
> named qualities rather than enumerated quantities.
>
This is exactly what happen when you define the first person by the  
knower. Bp & p, or if you prefer

provable(p) and true(p),

provable and true are not first person qualities, they are epistemological quantifications. First person would be more like great(p) and superior(p).

?

A square peg either fits in a particular sized round hole or not. That is a true/false. It can be proved by actually trying to fit the peg in the hole. Those are third person public conditions. Whether one peg or hole seems 'better' than another is a first person kind of distinction.

    Sentences are true (or false) only in worlds that have demonstrations of those sentences.

Then classical logic would not apply to arithmetic.



Truth is not independent of context. It is independent of any particular representation within a set of mutually accessible worlds. We cannot imagine that entities have some particular set of properties (which include truth values) in the absence of the possibility to verify such and verification is the quintessence of the physical world, unless we are OK with incoherent thinking....

This contradicts the existence of the body problem, or makes it impossible to solve, which is premature.




 



gives a modality which can provably be shown qualitative, and non  
formalizable in arithmetic. It leads to a logic (know as S4Grz) which  
describes something which is absolutely impossible to reduce to any  
number relations or even anything third person describable notion,  
even infinite one.

That doesn't mean it is qualitative, only that it is so obscurely arithmetic that arithmetic itself cannot quantify it.

But the machine still can refer to it. So I don't see why this would not work. It would not work if you reduce a machine to its body, but the divergence between Bp and Bp & p disallow such a reduction.

I don't really understand what modal logic has to do with the possibility of quality.

The machine modal logic of self-reference, Bp, when conjuncted with the non nameable Truth, leads the machine to discover true propositions, in a sort of immediate way, which they cannot justify nor even describe, except by projecting them on similar machines.

    Truth is not an object per se. It can be identified with an object, but it itself is not. The map is only the territory when the territory is the map. Truth is not a person and thus has no particular name, thus it cannot beconjuncted with anything else. It has no peer!

Then fundamental science is impossible. I understand better why you don't propose any theory. Eventually all you say is "don't ask". This is bad philosophy, as it kills science.




The redness quality is like that. I have no doubt that I experience redness, but I am unable to communicate it to a blind rationalist. I can point on something red, and hope my fellow has a sufficiently similar experience, so that the word "red" will have a sufficiently close meaning to mine.

    You cannot communicate redness at all! Qualia are strictly 1p. We can talk about representations of 1p, but never 1p itself.

Here you do clearly the level confusion. Of course we can talk about qualia, even if we cannot define them. With S4Grz we explicitly talk about things which admits no formal description.



This is why we need something like bisimulation to represent communication and interactions in general.

And this would contradict what you just say above, or you lost me completely. 



 

What quality does this modality have? Is it shiny? Is it spicy? Does it get tired?

The modality has no quality. It describes qualities. You might do a confusion of level. It is a bit like the confusion between the string "s(s(0))" and the number s(s(0)).

 
How is it describing qualities? It seems to me to be describing quantitative formalism. How does one describe red logically?

By explaining it exists necessarily and that it cannot be describe logically. Modal logic, in the machine context are "meta". It does not explain the quality red. It explains that quality exists and are NOT describable quantitatively.

    OK, then please be consistent! Our explanations only touch the representations of the 1p, we never touch our 1p themselves.

I was just saying that. you loss me more and more. We might waste our time trying for we have different methodlogies, and as long as you don't provide a theory I can not make sense of your remarks.




The "universal soul", Bp & p, or the S4Grz logic, is a sort of mathematical tour-de-force: it is a formal logic describing the logic of content which are impossible to formalize ever. But we can formalize this at the metalevel.

    We have formal representations of representations of the concept of "universal soul"... The concept is a collectively agreed upon representation and that collective agreement only happens because of the physical world.

Which contradict the comp body problem. the body problem is that we have to explain the body, not that we have to assume it in the theory. You are the one ignoring it.








You might think I just described it, by Bp & p, or by "provable(p) and  
true(p)", but this is not the case, as I use some of your intuition  
about truth, which cannot be arithmetized by itself, by a famous  
result of Gödel and Tarski (independently).
It happens that we do have a good intuition of many truth, and machine  
can indeed describe better and better approximations of the truth  
concept, but the limit of it, used here, cannot be. So by using both  
the comp hypothesis, and by studying simple (Löbian) machine (simpler  
than us) we can develop a formal (quantitative in some sense, at some  
level, from some point of view) theory concerning the non formal, and  
even non-formalizable-at-all-by-the-machine, qualities that machine  
can still refer about. And this can be used to explain why machine are  
forced to be befuddled by the subjectively-real apparent gap between  
third and first person attributes.

It seems like you are missing the obvious. Awareness is not just about knowing and navigating a set of logical conditions. That can be accomplished easily without any awareness. Experienced qualities are orthogonal to knowledge and procedural evaluation.

I can understand that "navigating a set of logical conditions" can be done without awareness. Awareness or knowlegde arrives when the navigating embed the navigator in truth, or in a reality. There is a fixed point, and it is explained why this is felt as a personal non communicable experience.

The fact that there would be data that cannot be communicated from a fixed vector doesn't imply to me anything specifically personal or experiential. What is relevant about consciousness is not only *that* it is private, but what the content of that privacy is.

Of course.

    But please do not miss the point here, Bruno! You are ignoring the fact that representations can be of other representations but not only of other representations. Somewhere some how, there must be a persistent demonstration of the referent.

No problem. That exists in arithmetic, in the many relative ways.






The form of it is trivial, except that it helps us understand the relation to exterior realism. Using modal logic to describe experience makes a formula out of the shape of a can and presents it as a way of creating canned food.

See above.

    Yes, please do, Bruno! Maps of maps of maps of maps of ... only converge in a representation of the stream of mappings... and such a concept has a meaning because there is something that it is like that is a 1p.



 

 


> Let the ignoring and insulting begin!
>

We don't need that here, I think, nor anywhere. An insult is almost  
always equivalent with "I have no argument".

I agree.

I know. You are wise. And we agree on many things, but we are working in antipodal theories. I think you have reductionist conception of machine, to be franc.

You are wise too. I think that your view of my view of machines is more of a projection than you think.

Well, if I remember well, you are the one who refuse to sell a steak to my daugher's husband, who said "yes" to a doctor, and is, by construction, a machine.

    Umm, are you going to sue him for illegal acts of discrimination? Horrors, Craig is a criminal! Am I one as well for trying hard to actually understand what he is trying to communicate? To be serious, Bruno, plese consider that Craig and I have theories about machines and are not reductionists. We are just considering a wider context than you do with COMP. You are only considering formal theories that can be expressed in sigma_1 sentences, no?

No.




While I admit, certainly, to being ignorant of the particular details of mathematical principles which give you a more generous view of machine consciousness than I have, I don't limit the capabilities of machines in the way that you assume. If quanta is flat qualia,

Quanta are more sort of sharable qualia.

    Please explain this sentence. It assumes a theory of bodies that you have, so far, only been coy about.

UDA is the explanation here. Study it, perhaps.




First person plural construct brought by the many-world internal interpretation of arithmetic by arithmetical beings.

    For there to be a "many-world internal interpretation" there must be something that is a world and a reason for many of them - not just one. These are the sort of things that you do not discuss at length that are at the heart of my own critique of comp step 8.

I am still waiting for the critics.





then machine intelligence can extend infinitely farther than organic intelligence, because of the horizontal universality of it.

Yes, indeed.

    See! We are not reductionist.

?





What I am saying is that you can't build qualia out of quanta,

I agree with this. Qualia emerges logically before quanta, but it seems the contrary. That can perhaps be related to Stephen-Pratt reversal of arrows in the mind body relation, or to the duality between thrid person (Bp) and the Theaetetical first person (Bp & p).

    Indeed! The reversal of the arrows is a symmetry that is only possible because of the postulation that numbers and physical matter are on the same ontological level (neither is primitive). They emerge from each other.

That is not in Pratt. 





because there is no universal qualia.

This I doubt. More from personal reasons than comp, though.

    Please elaborate on this!

You need to grasp UDA first.





Qualia is orthogonal to quanta - it is the manifestation of non-universality itself.

Well, with comp, quanta is the tiny emerging part of the qualia-consciousness part of the arithmetical platonia, seen from inside.

    I would agree with this if and only if you concede that the particularity of the quanta are the result of infinitely many bisimulations.

This is 1004, in absence of a understandable theory.



This is the solution to the body problem that I propose. You need to see where bisimulation fits in comp. AFAIK, there is no room in COMP for bisimulation because it is the theory of a single monad.

?






May be the crux of our difference (beyond comp/non-comp) is that, as many people, you seem to believe that mind and matter are symmetrical and have same caliber. But comp is still open too full platonism, in which matter, and physical universe are more like a divine bug, a temporary misleading; hding the truth more than paving a way to it. Some platonist identify matter with bad, or the devil. this is of course an open problem with comp, but what can already been argued for is that the mindscape is vastly bigger than the physical scape. The comp mindscape or personscape is vast. *Very* vast. It escapes the math. It violates physicalism, but also mathematicalism, despite the ontology is quite little (a tiny segment of arithmetical truth). 


    There is no asymmetry between the mental and the material! You only think this because you ignore the uncountable infinity of possible physical worlds.

? On the contrary, this is already explained, even at step seven. You lost me, as you criticize something which is hardly related to what I try to explain. 

Bruno



It is the possibility of the thinking that "all is matter" or "all is mind" that is the problem when misunderstood. There is so much more that I need to write on this but I have no more time to write...

Bruno




-- 
Onward!

Stephen

"Nature, to be commanded, must be obeyed." 
~ Francis Bacon

--
You received this message because you are subscribed to the Google Groups "Everything List" group.
To post to this group, send email to everyth...@googlegroups.com.
To unsubscribe from this group, send email to everything-li...@googlegroups.com.
For more options, visit this group at http://groups.google.com/group/everything-list?hl=en.

Roger Clough

unread,
Aug 21, 2012, 6:12:17 AM8/21/12
to everything-list
Hi Bruno and Stephen,
 
This is the bicameral mind again. Right brain must accept left brain decisions for human safety.
 
Ought must rule over is (or else we'd all be nazis, Hume, for the safety of humanity)
Passion must rule over reason (or else we'd all be nazis, Hume, for the safety of humanity)
Acceptace of proof dominates proof (common sense psychology)
 
Thus you can objectively, mathematically prove that 2+2=4, but you still have to subjectively accept that psychologically.
Woman always gets the last word.
 
Roger Clough, rcl...@verizon.net
8/21/2012
Leibniz would say, "If there's no God, we'd have to invent him so everything could function."
----- Receiving the following content -----
Time: 2012-08-21, 05:58:57
Subject: Re: Reconciling Bruno's Primitives with Multisense

You keep being deadly wrong on this issue. The point is that we CANNOT assume this AFTER having said "yes" to the doctor. We can assume a physical reality, and interaction, but in fine, we have to justify it from the numbers, and this is the formulation of the problem. I refer you to Girard "geometry of interaction" which is the closer from what we get from G锟斤拷del's self-reference. 




The conclusion that "matter is a hallucination", while correct IMHO, does not eliminate the impact of this pre-existing collection and their capacities.

How could an hallucination be pre-existing? That does not make sense.






 
 


It seems shrouded in obfuscating self-reference. Why would anything that has been encoded ever need to be decoded if the machine can fluently process the encoded form?

To store what we learn. The DNA plays already such a role at the molecular level. It illustrates also a digital encoding and decoding. 

Why would storage imply any kind of encoding? Without physics to constrain resource requirements, there would be no need to compress information.

The numbers do that all the time. There is no need, but they cannot not doing it, because it follows from addition and multiplication. There is no need for Saturn's ring either.

    Ultimately one could argue that anything is "no needed" and yet such arguments are wrong headed.

    Numbers are not isolate entities! They do not "do" anything in and from themselves.

This is too vague. You are ignoring than numbers obeys laws which makes them Turing universal, so I am not sure what you mean by isolated in this context. Of course they are digital, so they have a discrete topology, but this does not isolate them. Numbers do things, like 3 divides 6, or like a universal numbers can implement a human brain, independently of anything else.




There is not even an "self" in a number alone unless we think of them as a fixed point of a class of transformations of a quasi-infinite collection. You seem to be using a Sorites paradox type of argument here to make a false point.
http://plato.stanford.edu/entries/sorites-paradox/
"The sorites paradox is the name given to a class of paradoxical arguments, also known as little-by-little arguments, which arise as a result of the indeterminacy surrounding limits of application of the predicates involved. For example, the concept of a heap appears to lack sharp boundaries and, as a consequence of the subsequent indeterminacy surrounding the extension of the predicate 锟斤拷is a heap锟斤拷, no one grain of wheat can be identified as making the difference between being a heap and not being a heap. Given then that one grain of wheat does not make a heap, it would seem to follow that two do not, thus three do not, and so on. In the end it would appear that no amount of wheat can make a heap. We are faced with paradox since from apparently true premises by seemingly uncontroversial reasoning we arrive at an apparently false conclusion.





> This continuum f (锟斤拷(Om)), runs from infinitely solipsistic/private  
> first person subjectivity (calling that Aleph 锟斤拷)to infinitely  
> discrete/public third person mechanism (calling that Omega 锟斤拷), so  
> that at 锟斤拷,any given dream is experienced as 99.99锟斤拷9% dream and  
> 0.00锟斤拷1% number and at 锟斤拷 (Omega), any given machine or number is  
> presented as 99.99锟斤拷9% number and 0.00锟斤拷1% dream.
>

?


I'm mapping out this literal to figurative axis, as it modifies the axis of subject to object presentations. The more an experience extends figuratively/metaphorically, the less it extends literally/mechanically.

That makes some sense.


That's what I'm saying. Quanta is qualia that has been flattened until it spreads out like with a rolling pin to the point of universality. Qualia is the dough that has no extension into public space.

OK.


 

>
> The halfway point between the 锟斤拷 (Aleph) and 锟斤拷 (Omega) axis is the  
> perpendicular axis f (-锟斤拷(Om)) which is the high and low  
> correspondence between the literal dream and figurative number (or  
> figurative dream and literal number depending on whether you are  
> using the dream-facing epistemology or the number-facing  
> epistemology). This axis runs from tight equivalence (锟斤拷=锟斤拷  
> equality) to broadly elliptical potential set membership (锟斤拷锟斤拷锟斤拷  
> ellipsis)
>
> So it looks something like this:
>
> f(锟斤拷) 锟斤拷 {锟斤拷 锟斤拷锟斤拷锟斤拷 锟斤拷 锟斤拷=锟斤拷 锟斤拷}
>
> function (Om) is superset or equal to the continuum ranging from  
> Aleph to ellipsis perpendicular/orthogonal to the inverse range from  
> equality to Omega).
>
> To go further, it could be said that at 锟斤拷(Omega), 锟斤拷 (Om) expresses  
> as 10|O (one, zero, line segment, circle referring to the  
> quantitative algebraic and geometric perpendicular primitives) while  
> at 锟斤拷 (Aleph), 锟斤拷 (Om) expresses as
> 锟斤拷锟斤拷锟斤拷锟斤拷 (tetragrammaton or yod, hay, vov, hay, or in perhaps more  
> familiar metaphor, 锟斤拷锟斤拷锟斤拷锟斤拷(clubs, spades, hearts, diamonds)
>
> where:
>
> 锟斤拷 clubs (wands) =Fire, spiritual, tactile
> 锟斤拷 spades (swords) = Air, mental, auditory
> 锟斤拷 hearts (cups) =Water, emotional, visual
> 锟斤拷 diamonds (pentacles/coins) = Earth, physical, olfactory-gustatory
>
> Note that tactile and auditory modalities tune us into ourselves and  
> each others sensemaking (selves and minds), while the visual and  
> olfactory/gustatory sense modalities are about objectifying realism  
> of the world (egos or objectified selves/self-images and bodies). It  
> should be obvious that 锟斤拷 clubs (wands) and 锟斤拷 spades (swords) are  
> stereotypically masculine and abstracting forces, while 锟斤拷 hearts  
> (cups) and 锟斤拷 diamonds (pentacles/coins) are stereotypically  

    Mhm!



 



> We can锟斤拷t pretend to talk about the eidetic, dream like  

?
result of G锟斤拷del and Tarski (independently).
It happens that we do have a good intuition of many truth, and machine  
can indeed describe better and better approximations of the truth  
concept, but the limit of it, used here, cannot be. So by using both  
the comp hypothesis, and by studying simple (L锟斤拷bian) machine (simpler  

Of course.

Roger Clough

unread,
Aug 21, 2012, 6:50:11 AM8/21/12
to everything-list
Hi Stephen P. King
 
 
Leibniz did not have an overall theory of the universe such
as seems to be wanted here. The monadology is not an overall
theory of the universe, instead it is moreorless like a living
ecology, where the parts (monads) compete and collaborate with each
other through the supremem monad (the CPU) which in effect carries
out all of the needs, states, desires, abilities, expectations, etc.   
 
So Leibniz's ToE is a sociology.
 
 
Roger Clough, rcl...@verizon.net
8/21/2012
Leibniz would say, "If there's no God, we'd have to invent him so everything could function."
----- Receiving the following content -----
Receiver: everything-list
Time: 2012-08-20, 11:51:24
Subject: Re: The modal logic needs to aim purposefully toward the"best possible" solution.

Roger Clough

unread,
Aug 21, 2012, 7:03:41 AM8/21/12
to everything-list
Hi Richard Ruquist
 
I also believe in science. But if you're trying to trash religion
with science, science hasn't a clue nor a tool nor the proper
concepts to even begin with the task. Science does not know
what the meaning of anything is. Period.
 
 
Roger Clough, rcl...@verizon.net
8/21/2012
Leibniz would say, "If there's no God, we'd have to invent him so everything could function."
----- Receiving the following content -----
Receiver: everything-list
Time: 2012-08-20, 11:18:57
Subject: Re: divine selection versus natural selection

Roger,

Divine selection and natural selection are sourced,�
however at differing levels of information integration,
in the "universal锟紺YM monad锟絪ubspace".

Belief can also be a product of science.
I believe science.
Richard
On Mon, Aug 20, 2012 at 5:29 AM, Roger <rcl...@verizon.net> wrote:
Hi Bruno Marchal
According to the Bible, belief is a product of faith or trust, and that trust
does not come from you, it is a gift from God.锟絎e have nothing to do with it,

Stephen P. King

unread,
Aug 21, 2012, 7:58:44 AM8/21/12
to everyth...@googlegroups.com
    This tells me that you did not read much of Pratt. If you had you might have noticed his references to Girard's work.





The conclusion that "matter is a hallucination", while correct IMHO, does not eliminate the impact of this pre-existing collection and their capacities.

How could an hallucination be pre-existing? That does not make sense.

    Existence does not determine properties. It cannot, only mutuality determines properties.






 
 


It seems shrouded in obfuscating self-reference. Why would anything that has been encoded ever need to be decoded if the machine can fluently process the encoded form?

To store what we learn. The DNA plays already such a role at the molecular level. It illustrates also a digital encoding and decoding. 

Why would storage imply any kind of encoding? Without physics to constrain resource requirements, there would be no need to compress information.

The numbers do that all the time. There is no need, but they cannot not doing it, because it follows from addition and multiplication. There is no need for Saturn's ring either.

    Ultimately one could argue that anything is "no needed" and yet such arguments are wrong headed.

    Numbers are not isolate entities! They do not "do" anything in and from themselves.

This is too vague. You are ignoring than numbers obeys laws which makes them Turing universal, so I am not sure what you mean by isolated in this context. Of course they are digital, so they have a discrete topology, but this does not isolate them. Numbers do things, like 3 divides 6, or like a universal numbers can implement a human brain, independently of anything else.




There is not even an "self" in a number alone unless we think of them as a fixed point of a class of transformations of a quasi-infinite collection. You seem to be using a Sorites paradox type of argument here to make a false point.
http://plato.stanford.edu/entries/sorites-paradox/
"The sorites paradox is the name given to a class of paradoxical arguments, also known as little-by-little arguments, which arise as a result of the indeterminacy surrounding limits of application of the predicates involved. For example, the concept of a heap appears to lack sharp boundaries and, as a consequence of the subsequent indeterminacy surrounding the extension of the predicate ‘is a heap’, no one grain of wheat can be identified as making the difference between being a heap and not being a heap. Given then that one grain of wheat does not make a heap, it would seem to follow that two do not, thus three do not, and so on. In the end it would appear that no amount of wheat can make a heap. We are faced with paradox since from apparently true premises by seemingly uncontroversial reasoning we arrive at an apparently false conclusion.
This phenomenon at the heart of the paradox is now recognized as the phenomenon of vagueness (see the entry on vagueness). Though initially identified with the indeterminacy surrounding limits of application of a predicate along some dimension, vagueness can be seen to be a feature of syntactic categories other than predicates. Names, adjectives, adverbs and so on are all susceptible to paradoxical sorites reasoning in a derivative sense".


You might explain the relevance of this. Where do I do that error?

    You are missing the tiny bit of reference to the physical world in each and every number.







I can't think of any reason that a Turing machine would need to create an abstraction layer, especially if you could control the speed of the tape. If for any reason you needed subroutines to synchronize, you could run those parts of the tape faster or slower - infinitely fast if you like. Without physics, time is relative and unbounded. A Turing machine needs no programming language, it doesn't even need bytes. You can do all of your addressing on the fly with dynamically sized memory chunks. All of these formalisms, codes, architectures, etc are purely in the service of the limitations imposed by physics and human convenience.

Not if you bet that we are machine. In that cse the UD reasoning shows that what you describe is only the appernt view from inside, but that eventually the physical resource constraints arise from the mathematical computer science constraints, that is, from arithmetic.

    Sure, we have evidence that our experiences of the world are not direct contact with the objects making up the world but are a form of virtual reality generated by our brain. Therefore we can make good bets that more often than not are good such that they pay off, but the computation that the brain generates that our experiences supervene upon simply does not happen in the absence of physical processing that fall under the rule of "thermodynamics".


If you prove this, then you can add a 9th step and conclude that comp is wrong. But you keep saying this without proof or argument.

    I cannot help if you cannot see it.




There is no knowledge for free. Arithmetic is the product of many such brains interacting with each other *and* it is a relational structure that exists in its own right.

?


We cannot remove the former of this pair and imagine that the latter is left to float free and continue with all of its abilities intact.

In the non comp theory, as step 8 shows that even if there was a physical reality, it would not be related to any form of stable consciousness.

    It is the stability of consciousness itself, but this does not make consciousness primitive. I truly need to present a more carefully reasoned argument for the neutrality of consciousness. You refuse to read B. Russell's stuff. OK... You will have to deal with my terrible writings...






We get tired of writing a thousand lines of code every time we want to transform some dataset, but the computer doesn't care. It will run a trillion lines of code just to add the same number to itself, it doesn't care how long it takes or how wasteful it seems to us.

Not with comp, as I am a person which manifest itself through a computer, and persons care.

    Certainly, but this is because of the vast constellation of other persons that you are capable of not only betting to exist and have some particular set of properties, but also cannot just theorize out of existence. COMP does not exist in isolation. It is a concept that exists in (all possible versions of ) our records of it (which include our internal brain memories).

Again that would contradict either comp, or comp's consequence. So you are saying that comp is false, or that there is a flaw in UDA. But as long as you don't find it, you have to accept comp's consequences. Without AUDA and the quantum I would also have taken this as an argument making comp not really plausible, but with QM nature seems to follow comp in its most startling consequences, so a non-comp conclusion is premature with respect to UDA.

    If you could just consider building a toy model of how to 1) represent a pair of amoeba with your construction and 2) model the conversation between them that is possible.





Why would it need any other form - especially if it is all made of numbers?

Nobody needs a universe. Why do we do babies?
The "truth" is that we have them, we cannot really avoid them. It is like the prime number and the universal machine. Once you have zero and + and *, you get Platonia, and a lot of mess in Platonia. It is a logical consequence.

But a universe is not a consequence of Platonia.

It is. Or your non-comp assumption is correct, but I do not work in the theory non-comp. Actually, I have not yet seen a non-comp *theory*. Only philosophical argument against comp, but no concrete replacement.

    I cannot speak for Craig directly, but where we have a disagreement is in the things that COMP ignores. The larger universe within which it has meaningful expression.

How does the larger universe be responsible for meaning?

    Meaningfulness is in the many agreements between many (Finite) minds, nowhere else.


Are all the interacting people living on arithmetic's border zombies?

    In a way yes. You are a zombie to me and I to you. WE only abduct that we have actual qualia. As you say, we "bet on it".







Since we know that we have a universe

We know only that we are conscious. How could we know that there is a (physical) universe?

    Because we can kick the wall and watch our foot rebound.

If you find this convincing, we have a problem.

    Indeed! It is from my considerations of interactions that I came to my conclusions. Nowhere in your papers is interactions between separate minds discussed.





There is overwhelming evidence that there is a physical universe.


I agree completely with this. With comp, that is the reason of the body problem. We must justify our belief in it without postulating it. Or there is a flaw in UDA. Where?

    Step 8. In the idea that one can simply not postulate a physical universe and thus make it vanish.  By the way. It is a bodies (plural) problem, not a body problem... Pratt pointed this out in his papers as the problem that Descartes completely missed.





The very act of exchanging these emails is some of that evidence!

With comp that is an evidence only for a dream sharing, or a video game, not a primitive reality. You keep confusing level and meta-levels.

    No, I am not. Primitive reality has no properties associated with it. It cannot be assumed or one gets contradictions in one's theory. It is not numebrs or matter or pink unicorns it is only necessary possibility.





Can we be certain of what it is like in-itself? No! We cannot! But we can form theories about what it might be and test these. This fact alone tells us that there is "something" that is there and it is not just the figment of my (singular) imagination.


Of course. but with comp, what is "really there" can and must be limited to the sigma_1 truth. 

    No, sigma_1 representations are only what can be non-contradictorily communicated, it is not the whole of reality. The truth values of those do not define properties, they *are* a form of property!





We don't even *know* that there is a mathematical universe, or even an arithmetical universe. We bet on it.

    Yes, and we can do that only because of the prior possibility of being able to represent facts about that universe to ourselves and each other.

Then comp is false.

    Indeed, but only step 8's mistake.   






that means that it is Platonia which needs to be justified intellectually in terms of contributing to the universe, not the other way around.

But with comp, we do got an explanation of the physical universe(s). And we got both quanta and qualia. It might be false, but the explanation is there.

    Only to you is there such a complete explanation.

No. To all universal machines/numbers.

    In your mind.





We simply cannot ignore the fact that we are "not-Bruno" and that only "Bruno bets correctly that comp is true".

By definition, all computationalist makes that bet. And I am agnostic on computationalism. I just show that the idea that there is a physical universe cannot explain our beliefs that there is a physical universe.

    You are not behaving like an agnostic. You are behaving like a "True Believer"!






This is like the sentence: "Stephen Paul King cannot prove that this statement is true". I can only bet on it one way or another, but anyone else can prove things about it.


 


What I am saying is that if you are going to invoke a possibility of dreams, that has to be grounded in the terms that you are laying out as primitive. Why would dreams leap out of mechanical relations?

It is a logical consequence, once you accept the idea that you might survive with a digital brain.

It's the logic of begging the question. I ask you why numbers dream and you say it's logical if you accept that dreams can survive as digital process. I don't though. If human dreams could exist in something other than humans, then they would already.

And that is the case already, indeed. That follows from the reasoning.

    :-P


On the sands of the beaches, in the odd complicated cloud formation. I would need a compelling reason to believe otherwise. Why would I give the benefit of the doubt to machines when machines have clearly shown no signs of feeling thus far?

But molecular biology, quantum mechanics, etc. suggest strongly that we are machine.

    You are not just talking about particular formal theories! You are also tacitly taking into account the vast world of observations using those theories that find them to be correct.

We never find that a theory is correct.

    So? We cannot prove the Goldbach conjecture either because it is impossible to test every possible prime pair. The computational resources for such a test does not exist.





This does not deny the suggestion that we are machine, but it does not allow a free floating abstract representation of The Machine to exist *and* do things that only the universe of interacting entities can do.

Well, if you have a problem with 2+2=4, there is nothing I can do for you. You are stuck in the level-metalevel confusion I am afraid.
That might explain why you cannot give a theory.

    I have no problem at all with 2+2=4. I just have problems with people that believe that entities have inherent properties.







The rest is the logical consequence. There is no evidence for non-comp.

    This email *is* evidence! That I can coherently  (in spite of my dyslexia) understand what you might mean with those symbols) such that I can respond in kind is evidence that not-comp. But I am only focusing on the error in step 8 that makes comp fail for me, and thus make a non-comp claim.

But please show the error. You only claim that there is one, staring from a philosophical conviction. You might be right, but then use your philosophical conviction to find it. What is wrong in step 8. Besides most of what you said already contradict step seven too.

    Step 8 assumes that not postulating the ontologically necessary to communicate a concept allows the concept to still be communicated without contradiction. You cannot simply have numbers as having inherent properties without a means to define what such might be.







So we do have compelling reason to think that machine can have feeling. The conjunction of our own experience, and the study of our bodies.

    No no no no no! We cannot ever have any reason, it is not 1p and thus cannot be considered as knowledge! It is not possible to exactly represent the 1p, we can only approximate it. We can get very very very close to it, but we can never get "it".

Then the doctor cannot succeed in reincarnating us through a digital brain, and all you say is that comp is wrong. 



My experience of what it is like to be me is not sharable, and thus cannot be copied, coded or transferred. I can only copy, code and share representations of it. The map is only the territory when the territory is the map. Otherwise please do not conflate or confuse the two.

Comp only conflates the fixed point, with this image.

    That is the part where you are a genius and deserve a huge prize, but you ignore the other requirements that allow the fixed point, transformation and closure.








But I don't want to defend comp. That's the kind of philosophy I do not work on. I just show the consequence of being material machine. The consequences are that we are already in a "matrix", whose structure can be mathematically handled, so that we can constructively derive physics, and compare it to the empirical physics, so that comp is shown testable. (To be short).


Not just that they might not have feelings, but that they are universally known for lacking any capacity for feeling whatsoever - that is what defines the nature of machines. When you understand the continuum of eidetic to entopic, figurative to literal, you will understand why this makes perfect sense

But what you say makes sense. The comp theory already explains why comp is hard (impossible, even)  to believe, and why it has to be highly counter-intuitive.

    Only because of the very abstract math involved, otherwise it is not complicated at all! Otherwise the "explain it to your grandmother" is impossible.

Grandmother can understand that "explaining the 1p" in term of 3p is impossible, once comp is assumed, and so it needs some act of faith.

    3p is the illusion!





A machine is an assembly of logical forms. That does not produce any phenomenology by itself because it is constructed from the outside in. Living organisms build themselves from the inside out, from their own native sensitivities and motivations. They are not having an alien script intentionally imposed upon their behavior.

I don't know that. Genetic illustrates that nature do scripts. But math shows also that very short scripts, like "help yourself" can lead to tremendous richness and complexity.

    Umm, this is a subject that we should elaborate on all on its own. The sentence "help yourself" is not just 12 symbols in some particular order!


 

Even if there was some purpose for it, how could that actually take place - what are the dreamings made of?

Ontologically: nothing take place. All the computations are there. Some emulate self-observing machine and the math explain why they have to be beffudled by existence and conscience.


That may be, but why in the world would computational befuddlement be expressed as personality and realism?

Because there is, very roughly, 60000^10000000000 brain states possible, that makes a lot of subjectivity possible, and they are multiplied and selected through a filtering on a continuum of computations (first person indeterminacy).
    Sure, a lot of difference that make a difference can be coded with those many brain states. But you need to show why they have the particular rules that they have.

That is the easy part of comp.



We simply cannot ignore the body problem!

Indeed, that is the conclusion of UDA. It is very hard to understand you, as you repeat what I say all the times, with a tone like if I was ignoring it. That there is a body problem is the result of the investigation as UDA makes clear.

    OK, let us consider that body problem. Why the singular body and not many bodies?
    How is my immaterial mind causing your immaterial mind to do things that it would otherwise not do? Got any ideas? How do immaterial entities interact?







Modal logic can make a toy model of experience, by removing all of the experience and replacing it with a meaningless variable.

Why meaningless?

Because that's what variables do, they generalize all potential content-meanings under single quantifiable term.

Hmm... OK. But the person is in the box (B), not in the variable, which is usually used for some unspecified arithmetic proposition, or memories content, etc.

    No! A person is not "in the box (B)" at all! This is a map-territory conflation without explanation!

When you say yes to the doctor, you are in the brain's plan, which is given by the arithmetical probability corresponding to that B. The map-territory are confused here as it is the comp assumption. The doctor does not put a model of you in the artificial brain. It puts you into it, by assumption.

    What role does the doctor play in the formal theory?






 

It is to say 'here, instead of your baby, let's just call it 'consumer of diapers and milk' and design a nursery based on the statistics derived from that consumption.

No, it is the complete contrary. I see your point, but it is eaxtly that type of reduction that is prevented by comp. Even for machine, we can no more reduce them to their third person description. They do have a soul, even after-life, etc.
 
If machines have souls, then there has to be a way that the two are connected. There has to be some kind of rationale for it.

Nice to hear that. I agree.

    As do I.




What is needed is a way to reference phenomenological qualities which expresses not merely non-comp, but explicitly asserts quality and a view of the universe from the perspective of irreducible quality.

You have to postualte them, and to postulate matter, and to postulate a relation, and non-comp, and that is neither satisfying, nor working. It introduces difficulties where there are already enough, imo.

I don't think you have to postulate matter, you only have to formalize what you already assume about dreaming numbers.

OK. That's the point. In the comp theory.

    But comp theory does not float free of the world within which it has a meaning!

Level confusion.

    No, I am being careful. You fail to understand that meaningfulness is contingent, and thus cannot be considered as a priori or inherent. The truth of 17 is prime is not independent of the possibility of knowledge of the fact. It is merely independent from the knowledge of any given entity. We have completely different conception of universality. Plato made a big deal about the connection between the Forms and the shadows on the wall... He even used second allegory to illustrate this: the Divided Line allegory. The Forms are not "disconnected" or separate, otherwise knowledge would be impossible.





Just as how "17 is prime" has a unique truth value only in a world where that set of physical pattern convey a meaning. In the absence of that world there is not meaning nor truth value.

If that was true, an explanation of 17 is prime would involved a physical reality. it does not. You are confusing level of explanation.

    All knowledge that you have of "17 is prime" involved a physical experience. All! The writing of the symbols is a physical act...





 



> We can’t pretend to talk about the eidetic, dream like  
> perpendicular of number logic while using the purely empirical terms  
> of arithmetic reduction. We need symbols that can only refer to  
> named qualities rather than enumerated quantities.
>
This is exactly what happen when you define the first person by the  
knower. Bp & p, or if you prefer

provable(p) and true(p),

provable and true are not first person qualities, they are epistemological quantifications. First person would be more like great(p) and superior(p).

?

A square peg either fits in a particular sized round hole or not. That is a true/false. It can be proved by actually trying to fit the peg in the hole. Those are third person public conditions. Whether one peg or hole seems 'better' than another is a first person kind of distinction.

    Sentences are true (or false) only in worlds that have demonstrations of those sentences.

Then classical logic would not apply to arithmetic.

    Nonsense! You are ignoring the necessities of communication. Your ideas work consistently, but only in a solipsistic sense. Liek I asked earlier. Try it for yourself. Build a toy model of a pair of minds and show how they interact.



Truth is not independent of context. It is independent of any particular representation within a set of mutually accessible worlds. We cannot imagine that entities have some particular set of properties (which include truth values) in the absence of the possibility to verify such and verification is the quintessence of the physical world, unless we are OK with incoherent thinking....

This contradicts the existence of the body problem, or makes it impossible to solve, which is premature.

    Nonsense. Pratt solves the body problem. Read all of his paper, not just the abstract.




 



gives a modality which can provably be shown qualitative, and non  
formalizable in arithmetic. It leads to a logic (know as S4Grz) which  
describes something which is absolutely impossible to reduce to any  
number relations or even anything third person describable notion,  
even infinite one.

That doesn't mean it is qualitative, only that it is so obscurely arithmetic that arithmetic itself cannot quantify it.

But the machine still can refer to it. So I don't see why this would not work. It would not work if you reduce a machine to its body, but the divergence between Bp and Bp & p disallow such a reduction.

I don't really understand what modal logic has to do with the possibility of quality.

The machine modal logic of self-reference, Bp, when conjuncted with the non nameable Truth, leads the machine to discover true propositions, in a sort of immediate way, which they cannot justify nor even describe, except by projecting them on similar machines.

    Truth is not an object per se. It can be identified with an object, but it itself is not. The map is only the territory when the territory is the map. Truth is not a person and thus has no particular name, thus it cannot beconjuncted with anything else. It has no peer!

Then fundamental science is impossible.

    Indeed! There is no such thing as "fundamental science" there is no such thing as a measurable foundation in the absolute sense.


I understand better why you don't propose any theory. Eventually all you say is "don't ask". This is bad philosophy, as it kills science.

    This is soo sad.







The redness quality is like that. I have no doubt that I experience redness, but I am unable to communicate it to a blind rationalist. I can point on something red, and hope my fellow has a sufficiently similar experience, so that the word "red" will have a sufficiently close meaning to mine.

    You cannot communicate redness at all! Qualia are strictly 1p. We can talk about representations of 1p, but never 1p itself.

Here you do clearly the level confusion. Of course we can talk about qualia, even if we cannot define them. With S4Grz we explicitly talk about things which admits no formal description.

    All the while you are really just imagining that you are talking to me. You are just talking to your self and I am talking to myself. There is no "we" except as a concept in our minds but minds require physicality to interact.





This is why we need something like bisimulation to represent communication and interactions in general.

And this would contradict what you just say above, or you lost me completely.

    Are you familiar with bisimulation? There are many papers on it... I have only proposed a very crude dynamic version of it.





 

What quality does this modality have? Is it shiny? Is it spicy? Does it get tired?

The modality has no quality. It describes qualities. You might do a confusion of level. It is a bit like the confusion between the string "s(s(0))" and the number s(s(0)).

 
How is it describing qualities? It seems to me to be describing quantitative formalism. How does one describe red logically?

By explaining it exists necessarily and that it cannot be describe logically. Modal logic, in the machine context are "meta". It does not explain the quality red. It explains that quality exists and are NOT describable quantitatively.

    OK, then please be consistent! Our explanations only touch the representations of the 1p, we never touch our 1p themselves.

I was just saying that. you loss me more and more. We might waste our time trying for we have different methodlogies, and as long as you don't provide a theory I can not make sense of your remarks.




The "universal soul", Bp & p, or the S4Grz logic, is a sort of mathematical tour-de-force: it is a formal logic describing the logic of content which are impossible to formalize ever. But we can formalize this at the metalevel.

    We have formal representations of representations of the concept of "universal soul"... The concept is a collectively agreed upon representation and that collective agreement only happens because of the physical world.

Which contradict the comp body problem. the body problem is that we have to explain the body, not that we have to assume it in the theory. You are the one ignoring it.

    We have to explain how separate minds can interact. Please present some consideration of this.










You might think I just described it, by Bp & p, or by "provable(p) and  
true(p)", but this is not the case, as I use some of your intuition  
about truth, which cannot be arithmetized by itself, by a famous  
result of Gödel and Tarski (independently).
It happens that we do have a good intuition of many truth, and machine  
can indeed describe better and better approximations of the truth  
concept, but the limit of it, used here, cannot be. So by using both  
the comp hypothesis, and by studying simple (Löbian) machine (simpler  
than us) we can develop a formal (quantitative in some sense, at some  
level, from some point of view) theory concerning the non formal, and  
even non-formalizable-at-all-by-the-machine, qualities that machine  
can still refer about. And this can be used to explain why machine are  
forced to be befuddled by the subjectively-real apparent gap between  
third and first person attributes.

It seems like you are missing the obvious. Awareness is not just about knowing and navigating a set of logical conditions. That can be accomplished easily without any awareness. Experienced qualities are orthogonal to knowledge and procedural evaluation.

I can understand that "navigating a set of logical conditions" can be done without awareness. Awareness or knowlegde arrives when the navigating embed the navigator in truth, or in a reality. There is a fixed point, and it is explained why this is felt as a personal non communicable experience.

The fact that there would be data that cannot be communicated from a fixed vector doesn't imply to me anything specifically personal or experiential. What is relevant about consciousness is not only *that* it is private, but what the content of that privacy is.

Of course.

    But please do not miss the point here, Bruno! You are ignoring the fact that representations can be of other representations but not only of other representations. Somewhere some how, there must be a persistent demonstration of the referent.

No problem. That exists in arithmetic, in the many relative ways.

    How does mere existence define properties? Please explain this.
    Umm, Which papers of his have you read? This one is quite good in explaing the idea http://boole.stanford.edu/pub/bridge.pdf

Stephen P. King

unread,
Aug 21, 2012, 8:01:28 AM8/21/12
to everyth...@googlegroups.com
On 8/21/2012 6:50 AM, Roger Clough wrote:
Hi Stephen P. King
 
 
Leibniz did not have an overall theory of the universe such
as seems to be wanted here. The monadology is not an overall
theory of the universe, instead it is moreorless like a living
ecology, where the parts (monads) compete and collaborate with each
other through the supremem monad (the CPU) which in effect carries
out all of the needs, states, desires, abilities, expectations, etc.   
 
So Leibniz's ToE is a sociology.

HI Roger,

    A plurality that is actually singular to God, the supremum.

--
You received this message because you are subscribed to the Google Groups "Everything List" group.
To post to this group, send email to everyth...@googlegroups.com.
To unsubscribe from this group, send email to everything-li...@googlegroups.com.
For more options, visit this group at http://groups.google.com/group/everything-list?hl=en.

Richard Ruquist

unread,
Aug 21, 2012, 8:12:42 AM8/21/12
to everyth...@googlegroups.com
Roger,
You are mistaken. The universe is based on physical laws despite the existence of a supernatural, which I take to be based in the collective set of monads. 

The way in which the monads manifest the physical laws and constants of nature is a bonified subject of science, just are the study of COMP is. They may even be related except for the multiverse aspect of COMP.
Richard

On Tue, Aug 21, 2012 at 7:03 AM, Roger Clough <rcl...@verizon.net> wrote:
Hi Richard Ruquist
 
I also believe in science. But if you're trying to trash religion
with science, science hasn't a clue nor a tool nor the proper
concepts to even begin with the task. Science does not know
what the meaning of anything is. Period.
 
 
Roger Clough, rcl...@verizon.net
8/21/2012
Leibniz would say, "If there's no God, we'd have to invent him so everything could function."
----- Receiving the following content -----
Receiver: everything-list
Time: 2012-08-20, 11:18:57
Subject: Re: divine selection versus natural selection

Roger,

Divine selection and natural selection are sourced,�
however at differing levels of information integration,
in the "universal燙YM monad爏ubspace".

Belief can also be a product of science.
I believe science.
Richard
On Mon, Aug 20, 2012 at 5:29 AM, Roger <rcl...@verizon.net> wrote:
Hi Bruno Marchal
According to the Bible, belief is a product of faith or trust, and that trust
does not come from you, it is a gift from God.燱e have nothing to do with it,

Bruno Marchal

unread,
Aug 21, 2012, 2:22:43 PM8/21/12
to everyth...@googlegroups.com
Hi Roger,

I answer your many post in one, by pity for the virtual mail boxes.


On 20 Aug 2012, at 11:29, Roger wrote:

Hi Bruno Marchal
 
According to the Bible, belief is a product of faith or trust, and that trust
does not come from you, it is a gift from God. 

We have nothing to do with it,
at least that isa what we Lutherns believe.


If it is a gift by God, why a bible?

All religions which believes that religion does not apply to machine will remain stuck on earth, the others will conquer the physical universe.


Yes, Hume was complaining about slipping  modal logic into an argument.

OK. Note that this was before Kripke, who found a nice mathematical semantic for a large class of modal logics, giving them at least mathematical sense. And that was before it was realized, notably by Kripke, that incompleteness provides transparent aritthmetical interpretations of modal logics (Gödel, Löb, Solovay).


There are indeed some similarities between Hume and van Quine. Hume
was an empiricist while van Quine sxeems to me at l,east to have been a pragmatist.
Bother woirk from the particular to the general.  Theory schmeery.

Van Orman Quine pragmatism is not so well clear cut. Comp relates theology and theo-technology, you can eventually say yes to a doctor for pragmatic reason. 
Anyway. Technically Quine's critics on modal logic is refuted by incompleteness, even on the first order extension, with the quantifiers allowed to have variable in the scope of the box. Note that this is true Peano Arithmetic but not for Zermelo Frankel set theory. Quantifying in the scope of a set predicate is hard to define.

I spent 33 years at least in the metallurgical laboratory before retiring,
so in the end, I can't help that while I enjoy and respect theory, and
and am always fascinated by it, in the end I worship data. Pragmatism.
I was born that way.

We makes sense of data through theory and experiences, but not always consciously. The brain implements many theories learned through evolution. I don't think we can separate data from theory so easily. Somehow a brain is by itself already a theory. Our bodies are divine hypotheses, somehow, assuming comp. We are words in a rational truncation of a quantum field, to take a low level.

I have no problem with pragmatism, as long as it is not used against the freedom of any inquiry, nor used as justified invalid reasoning, or lies and propaganda. Nor used as pretext to cut the funding of fundamental research, as I can give a pragmatic reasons to fund fundamental research in all direction.

Pragmatic OK, if honest. That is sometimes difficult with respect to hard question, like "what's going on?". It is normal that we develop wishful thinking, and if that works, as already suggested by the Löb formula( in some very weak and formal sense to be sure), a theory has to be assumed always in remaining open it can be false.

Sorry, I was again being a bit harsh again.  You are a kind person.
 
Can you give me a link to the sort of output a comp program would provide ?
Being a natural pragmatist, I learn best from examples.

By definition, all programs are "comp programs", so an example of output is what happens on your computer's screen right now.
BY comp, I am a program, so another example, is this post. 

There is a reason why a machine looking inward become religious. 

Hi Bruno and Stephen

I want to inform you that you are wrong in all of your writings.

Please understand how very incorrect you are about everything you
post!  Why are you so wrong.

It would help if you could be a little more specific. 


Bruno

To unsubscribe from this group, send email to everything-li...@googlegroups.com.

For more options, visit this group at http://groups.google.com/group/everything-list?hl=en.

Bruno Marchal

unread,
Aug 21, 2012, 2:28:54 PM8/21/12
to everyth...@googlegroups.com
On 21 Aug 2012, at 12:12, Roger Clough wrote:

Hi Bruno and Stephen,
 
This is the bicameral mind again. Right brain must accept left brain decisions for human safety.
 
Ought must rule over is (or else we'd all be nazis, Hume, for the safety of humanity)
Passion must rule over reason (or else we'd all be nazis, Hume, for the safety of humanity)
Acceptace of proof dominates proof (common sense psychology)
 
Thus you can objectively, mathematically prove that 2+2=4, but you still have to subjectively accept that psychologically.
Woman always gets the last word.

No problem here. That fits nicely with the Bp versus Bp & p duality, which is just the difference between "rational belief" and "rational knowledge" (true rational belief).

It took time to realize that when we define the rational belief by formal proof, which makes sense in the ideal correct machine case, although knowledge and belief have the same content (the same arithmetical p are believed), still, they obey to different logics. This is a consequence of incompleteness. Rational beliefs obey to a modal logic known as G (or GL, Prl, K4W, etc.) and true rational belief obeys to a logic of knowledge (S4), indeed known as S4Grz. 

G is 

[](p -> q) -> ([]p -> []q)
[]p -> [][]p
[]([]p -> p) -> []p

with the rules A, A->B  /  B and A / []A

S4Grz is

[](p -> q) -> ([]p -> []q)
[]p -> [][]p
[]([](p -> []p) -> p) -> p

with the rules A, A->B  /  B and A / []A

Bruno

You keep being deadly wrong on this issue. The point is that we CANNOT assume this AFTER having said "yes" to the doctor. We can assume a physical reality, and interaction, but in fine, we have to justify it from the numbers, and this is the formulation of the problem. I refer you to Girard "geometry of interaction" which is the closer from what we get from G del's self-reference. 




The conclusion that "matter is a hallucination", while correct IMHO, does not eliminate the impact of this pre-existing collection and their capacities.

How could an hallucination be pre-existing? That does not make sense.






 
 


It seems shrouded in obfuscating self-reference. Why would anything that has been encoded ever need to be decoded if the machine can fluently process the encoded form?

To store what we learn. The DNA plays already such a role at the molecular level. It illustrates also a digital encoding and decoding. 

Why would storage imply any kind of encoding? Without physics to constrain resource requirements, there would be no need to compress information.

The numbers do that all the time. There is no need, but they cannot not doing it, because it follows from addition and multiplication. There is no need for Saturn's ring either.

    Ultimately one could argue that anything is "no needed" and yet such arguments are wrong headed.

    Numbers are not isolate entities! They do not "do" anything in and from themselves.

This is too vague. You are ignoring than numbers obeys laws which makes them Turing universal, so I am not sure what you mean by isolated in this context. Of course they are digital, so they have a discrete topology, but this does not isolate them. Numbers do things, like 3 divides 6, or like a universal numbers can implement a human brain, independently of anything else.




There is not even an "self" in a number alone unless we think of them as a fixed point of a class of transformations of a quasi-infinite collection. You seem to be using a Sorites paradox type of argument here to make a false point.
http://plato.stanford.edu/entries/sorites-paradox/
"The sorites paradox is the name given to a class of paradoxical arguments, also known as little-by-little arguments, which arise as a result of the indeterminacy surrounding limits of application of the predicates involved. For example, the concept of a heap appears to lack sharp boundaries and, as a consequence of the subsequent indeterminacy surrounding the extension of the predicate ‘is a heap’, no one grain of wheat can be identified as making the difference between being a heap and not being a heap. Given then that one grain of wheat does not make a heap, it would seem to follow that two do not, thus three do not, and so on. In the end it would appear that no amount of wheat can make a heap. We are faced with paradox since from apparently true premises by seemingly uncontroversial reasoning we arrive at an apparently false conclusion.





> This continuum f ( (Om)), runs from infinitely solipsistic/private  
> first person subjectivity (calling that Aleph  )to infinitely  
> discrete/public third person mechanism (calling that Omega Ω), so  
> that at  ,any given dream is experienced as 99.99…9% dream and  
> 0.00…1% number and at Ω (Omega), any given machine or number is  
> presented as 99.99…9% number and 0.00…1% dream.
>

?


I'm mapping out this literal to figurative axis, as it modifies the axis of subject to object presentations. The more an experience extends figuratively/metaphorically, the less it extends literally/mechanically.

That makes some sense.


That's what I'm saying. Quanta is qualia that has been flattened until it spreads out like with a rolling pin to the point of universality. Qualia is the dough that has no extension into public space.

OK.


 

>
> The halfway point between the   (Aleph) and Ω (Omega) axis is the  
> perpendicular axis f (- (Om)) which is the high and low  
> correspondence between the literal dream and figurative number (or  
> figurative dream and literal number depending on whether you are  
> using the dream-facing epistemology or the number-facing  
> epistemology). This axis runs from tight equivalence (“=”  
> equality) to broadly elliptical potential set membership (“…”  
> ellipsis)
>
> So it looks something like this:
>
> f( )   {  “…” ⊥ “=” Ω}
>
> function (Om) is superset or equal to the continuum ranging from  
> Aleph to ellipsis perpendicular/orthogonal to the inverse range from  
> equality to Omega).
>
> To go further, it could be said that at Ω(Omega),   (Om) expresses  
> as 10|O (one, zero, line segment, circle referring to the  
> quantitative algebraic and geometric perpendicular primitives) while  
> at   (Aleph),   (Om) expresses as
>      (tetragrammaton or yod, hay, vov, hay, or in perhaps more  
> familiar metaphor,     (clubs, spades, hearts, diamonds)
>
> where:
>
>   clubs (wands) =Fire, spiritual, tactile
>   spades (swords) = Air, mental, auditory
>   hearts (cups) =Water, emotional, visual
>   diamonds (pentacles/coins) = Earth, physical, olfactory-gustatory
>
> Note that tactile and auditory modalities tune us into ourselves and  
> each others sensemaking (selves and minds), while the visual and  
> olfactory/gustatory sense modalities are about objectifying realism  
> of the world (egos or objectified selves/self-images and bodies). It  
> should be obvious that   clubs (wands) and   spades (swords) are  
> stereotypically masculine and abstracting forces, while   hearts  
> (cups) and   diamonds (pentacles/coins) are stereotypically  

    Mhm!



 



> We can’t pretend to talk about the eidetic, dream like  

?
result of G del and Tarski (independently).
It happens that we do have a good intuition of many truth, and machine  
can indeed describe better and better approximations of the truth  
concept, but the limit of it, used here, cannot be. So by using both  
the comp hypothesis, and by studying simple (L bian) machine (simpler  

Of course.

Bruno Marchal

unread,
Aug 21, 2012, 3:04:46 PM8/21/12
to everyth...@googlegroups.com
On 21 Aug 2012, at 13:03, Roger Clough wrote:

Hi Richard Ruquist
 
I also believe in science. But if you're trying to trash religion
with science, science hasn't a clue nor a tool nor the proper
concepts to even begin with the task. Science does not know
what the meaning of anything is. Period.

I agree. Nevertheless, by using some hypothesis science might explain why science does not know the meaning of anything.
I agree with what you say, but not as a closure of inquiry. If something seems impossible, we must favor the simplest hypothesis which explains the impossibility.

Science is not truth. Science is only a tiny lantern on a big unknown/ignorance-space.
Only pseudo-scientist "know" the public truth. Serious scientists suggests only hypotheses, and evidences or refutation. Never "the truth".

Bruno




 
 
Roger Clough, rcl...@verizon.net
8/21/2012
Leibniz would say, "If there's no God, we'd have to invent him so everything could function."
----- Receiving the following content -----
Receiver: everything-list
Time: 2012-08-20, 11:18:57
Subject: Re: divine selection versus natural selection

Roger,

Divine selection and natural selection are sourced,�
however at differing levels of information integration,
in the "universal燙YM monad爏ubspace".

Belief can also be a product of science.
I believe science.
Richard
On Mon, Aug 20, 2012 at 5:29 AM, Roger <rcl...@verizon.net> wrote:
Hi Bruno Marchal
According to the Bible, belief is a product of faith or trust, and that trust
does not come from you, it is a gift from God.燱e have nothing to do with it,

Stephen P. King

unread,
Aug 21, 2012, 3:39:08 PM8/21/12
to everyth...@googlegroups.com
On 8/21/2012 8:12 AM, Richard Ruquist wrote:
Roger,
You are mistaken. The universe is based on physical laws despite the existence of a supernatural, which I take to be based in the collective set of monads.

Hi Richard,

    Please calm down a bit and understand that it is not possible for a single finite mind to comprehend, much less, "know" in a way that can be explained to the average "grandmother", the delicate balance of the monadology. Even Leibniz himself fudged his explanation!



The way in which the monads manifest the physical laws and constants of nature is a bonified subject of science, just are the study of COMP is. They may even be related except for the multiverse aspect of COMP.

    I agree with this remark 100%!

    One brief comment on the tittle of this thread. Is it necessary for "Divine Selection" and "Natural Section" to be two mutually contradictory possible explanations? How is God not immanent in Nature? It is only when we push transcendence that we have serious problems.

    BTW, this is another version of the disagreement that I am having with Bruno. He is pushing a transcendence only theory of truth and I am arguing for immanence *and* transcendence within an over all Panentheism theory. My argument revolves around the problem of interaction between multiple minds. My solution is not very different from Spinoza's but I seek to frame it using computer science, as that allows a finite mathematical model.
    Bruno's idea seeks a reduction of all interactions to being wholly within the Supremum and all appearances or interaction and actions in general (including physics) to "dreams of numbers". The problem with this is that Transcendance models fall apart when they try to explain the necessity of finite appearance.
    Transcendence alone theories just postulate that all objects have properties in an inherent way because of they are "in reality just shadows of the Forms" and "Forms" are the essence of the properties themselves. This works and sound fine until one tries to construct a model of interactions using that theory. Doing so inevitably causes contradictions to arise that cannot be solved by appeals to measures or any other hand-waving or question-begging device. Please think about this carefully, the reasoning is very subtle, but unassailable.

    How might the "shadows of the Forms" cast shadows of their own on each other? What about shadows of shadow of shadows of shadows of ... What prevents the infinite regress? AFAIK, only the limitations of actual physical resources cut off the computations such that endless loops of self-modeling recursions never happen. This possibility was, sadly, missed by Dennett in his valiant attempt to save materialism. Computations having to actually solve an NP-Complete problem with finite resources is the requirement that eliminates Bruno's measure problem, but he refuses to see this.

Stephen P. King

unread,
Aug 21, 2012, 3:42:43 PM8/21/12
to everyth...@googlegroups.com
On 8/21/2012 2:28 PM, Bruno Marchal wrote:

On 21 Aug 2012, at 12:12, Roger Clough wrote:

Hi Bruno and Stephen,
 
This is the bicameral mind again. Right brain must accept left brain decisions for human safety.
 
Ought must rule over is (or else we'd all be nazis, Hume, for the safety of humanity)
Passion must rule over reason (or else we'd all be nazis, Hume, for the safety of humanity)
Acceptace of proof dominates proof (common sense psychology)
 
Thus you can objectively, mathematically prove that 2+2=4, but you still have to subjectively accept that psychologically.
Woman always gets the last word.

No problem here. That fits nicely with the Bp versus Bp & p duality, which is just the difference between "rational belief" and "rational knowledge" (true rational belief).

It took time to realize that when we define the rational belief by formal proof, which makes sense in the ideal correct machine case, although knowledge and belief have the same content (the same arithmetical p are believed), still, they obey to different logics. This is a consequence of incompleteness. Rational beliefs obey to a modal logic known as G (or GL, Prl, K4W, etc.) and true rational belief obeys to a logic of knowledge (S4), indeed known as S4Grz. 

G is 

[](p -> q) -> ([]p -> []q)
[]p -> [][]p
[]([]p -> p) -> []p

with the rules A, A->B  /  B and A / []A

S4Grz is

[](p -> q) -> ([]p -> []q)
[]p -> [][]p
[]([](p -> []p) -> p) -> p

with the rules A, A->B  /  B and A / []A

Bruno

Dear Bruno,

    It might help us immensely if you could tell us how to read these symbolic representations. Not all of us speak that language! There are English words for all of these symbols!

Stephen P. King

unread,
Aug 21, 2012, 3:43:50 PM8/21/12
to everyth...@googlegroups.com
Hear Hear!


On 8/21/2012 2:22 PM, Bruno Marchal wrote:
> All religions which believes that religion does not apply to machine
> will remain stuck on earth, the others will conquer the physical universe.
>


Roger Clough

unread,
Aug 22, 2012, 4:24:15 AM8/22/12
to everything-list
 
ROGER: According to the Bible, belief is a product of faith or trust, and that trust
does not come from you, it is a gift from God.

We have nothing to do with it,
at least that is what we Lutherans believe.


BRUNO: If it is a gift by God, why a bible?

ROGER: Faith helps us to believe. the Bible, particularly the Gospels, tells us what to believe (that Jesus saved us).

BRUNO: All religions which believe(s) that religion does not apply to machine will remain stuck on earth, the others will conquer the physical universe.

ROGER: ?

BRUNO: Yes, Hume was complaining about slipping modal logic into an argument.

OK. Note that this was before Kripke, who found a nice mathematical semantic for a large class of modal logics, giving them at least mathematical sense. And that was before it was realized, notably by Kripke, that incompleteness provides transparent aritthmetical interpretations of modal logics (G?el, L?, Solovay).

ROGER: There are indeed some similarities between Hume and van Quine. Hume
was an empiricist while van Quine seems. to me at least, to have been a pragmatist.
Both work from the particular to the general. Theory schmeery.

BRUNO: Van Orman Quine pragmatism is not so well clear cut. Comp relates theology and theo-technology, you can eventually say yes to a doctor for pragmatic reason. Anyway. Technically Quine's critics on modal logic is refuted by incompleteness, even on the first order extension, with the quantifiers allowed to have variable in the scope of the box. Note that this is true Peano Arithmetic but not for Zermelo Frankel set theory. Quantifying in the scope of a set predicate is hard to define.


ROGER: I spent 33 years at least in the metallurgical laboratory before retiring,
so in the end, I can't help that while I enjoy and respect theory, and
and am always fascinated by it, in the end I worship data. Pragmatism.
I was born that way.


BRUNO:  We makes sense of data through theory and experiences, but not always consciously. The brain implements many theories learned through evolution. I don't think we can separate data from theory so easily. Somehow a brain is by itself already a theory. Our bodies are divine hypotheses, somehow, assuming comp. We are words in a rational truncation of a quantum field, to take a low level.
 
ROGER: Good.


BRUNO: I have no problem with pragmatism, as long as it is not used against the freedom of any inquiry, nor used as justified invalid reasoning, or lies and propaganda. Nor used as pretext to cut the funding of fundamental research, as I can give a pragmatic reasons to fund fundamental research in all direction.
 
ROGER: "Pragmatic" used in the vernacular sense is usually another word for "practical".  As in: "Our reason for cutting the program was pragmatic. We simply couldn't afford it."
But that is not exactly what pragmatic means philosophically. Philosophically (see Peirce) pragmatic means that the (pragmatic, not traditional) truth of an issue is what results from actually carrying it out.
As the experimental result is the truth of carrying out an experimental protocol. It may not be true in the ordinary sense. Because by going from the particular to the general,
you are using synthesis, not analysis. Synthesis can provide unexpected truths, so very powerful. Just an experiment can give you totally unexpected results.

BRUNO:  Pragmatic OK, if honest. That is sometimes difficult with respect to hard question, like "what's going on?". It is normal that we develop wishful thinking, and if that works, as already suggested by the L? formula( in some very weak and formal sense to be sure), a theory has to be assumed always in remaining open it can be false.


ROGER:  [Reflecting] Sorry, I was again being a bit harsh again. You are a kind person.  But pragmatism is as honest as a carefully planned and carried out scientific experiment.


Can you give me a link to the sort of output a comp program would provide ?
Being a natural pragmatist, I learn best from examples.


BRUNO:  By definition, all programs are "comp programs", so an example of output is what happens on your computer's screen right now.
BY comp, I am a program, so another example, is this post.
 
ROGER: OK.

BRUNO: There is a reason why a machine looking inward become religious.

Roger Clough

unread,
Aug 22, 2012, 4:50:25 AM8/22/12
to everything-list
Hi Stephen P. King
 
 
Physical Laws must be supernatural.
You can't measure them. 
 
 
Roger Clough, rcl...@verizon.net
8/22/2012
Leibniz would say, "If there's no God, we'd have to invent him so everything could function."
----- Receiving the following content -----
Time: 2012-08-21, 15:39:08
Subject: Re: divine selection versus natural selection

On 8/21/2012 8:12 AM, Richard Ruquist wrote:
Roger,
You are mistaken. The universe is based on physical laws despite the existence of a supernatural, which I take to be based in the collective set of monads.

Hi Richard,

锟斤拷锟斤拷锟斤拷 Please calm down a bit and understand that it is not possible for a single finite mind to comprehend, much less, "know" in a way that can be explained to the average "grandmother", the delicate balance of the monadology. Even Leibniz himself fudged his explanation!



The way in which the monads manifest the physical laws and constants of nature is a bonified subject of science, just are the study of COMP is. They may even be related except for the multiverse aspect of COMP.

锟斤拷锟斤拷锟斤拷 I agree with this remark 100%!

锟斤拷锟斤拷锟斤拷 One brief comment on the tittle of this thread. Is it necessary for "Divine Selection" and "Natural Section" to be two mutually contradictory possible explanations? How is God not immanent in Nature? It is only when we push transcendence that we have serious problems.

锟斤拷锟斤拷锟斤拷 BTW, this is another version of the disagreement that I am having with Bruno. He is pushing a transcendence only theory of truth and I am arguing for immanence *and* transcendence within an over all Panentheism theory. My argument revolves around the problem of interaction between multiple minds. My solution is not very different from Spinoza's but I seek to frame it using computer science, as that allows a finite mathematical model.
锟斤拷锟斤拷锟斤拷 Bruno's idea seeks a reduction of all interactions to being wholly within the Supremum and all appearances or interaction and actions in general (including physics) to "dreams of numbers". The problem with this is that Transcendance models fall apart when they try to explain the necessity of finite appearance.
锟斤拷锟斤拷锟斤拷 Transcendence alone theories just postulate that all objects have properties in an inherent way because of they are "in reality just shadows of the Forms" and "Forms" are the essence of the properties themselves. This works and sound fine until one tries to construct a model of interactions using that theory. Doing so inevitably causes contradictions to arise that cannot be solved by appeals to measures or any other hand-waving or question-begging device. Please think about this carefully, the reasoning is very subtle, but unassailable.

锟斤拷锟斤拷锟斤拷 How might the "shadows of the Forms" cast shadows of their own on each other? What about shadows of shadow of shadows of shadows of ... What prevents the infinite regress? AFAIK, only the limitations of actual physical resources cut off the computations such that endless loops of self-modeling recursions never happen. This possibility was, sadly, missed by Dennett in his valiant attempt to save materialism. Computations having to actually solve an NP-Complete problem with finite resources is the requirement that eliminates Bruno's measure problem, but he refuses to see this.

锟斤拷锟斤拷锟斤拷

Richard

On Tue, Aug 21, 2012 at 7:03 AM, Roger Clough <rcl...@verizon.net> wrote:
Hi Richard Ruquist
锟斤拷
I also believe in science. But if you're trying to trash religion
with science, science hasn't a clue nor a tool nor the proper
concepts to even begin with the task. Science does not锟斤拷know
what the meaning of anything is. Period.
锟斤拷
锟斤拷
Roger Clough, rcl...@verizon.net
8/21/2012
Leibniz would say, "If there's no God, we'd have to invent him so everything could function."
----- Receiving the following content -----
Receiver: everything-list
Time: 2012-08-20, 11:18:57
Subject: Re: divine selection versus natural selection

Roger,

Divine selection and natural selection are sourced,
however at differing levels of information integration,
in the "universal锟紺YM monad锟絪ubspace".

Belief can also be a product of science.
I believe science.
Richard
On Mon, Aug 20, 2012 at 5:29 AM, Roger <rcl...@verizon.net> wrote:
Hi Bruno Marchal
According to the Bible, belief is a product of faith or trust, and that trust
does not come from you, it is a gift from God.锟絎e have nothing to do with it,
at least that isa what we Lutherns believe.
8/20/2012
Leibniz would say, "If there's no God, we'd have to invent him so everything could function."

Bruno Marchal

unread,
Aug 22, 2012, 4:56:05 AM8/22/12
to everyth...@googlegroups.com

On 21 Aug 2012, at 13:58, Stephen P. King wrote:

>> where do you think the flaw is.

 Step 8. In the idea that one can simply not postulate a physical universe and thus make it vanish. 

I ask you to provide what is invalid in a proof that P -> Q, and you give me a philosophical opinion why you estimate that Q is non sense. That is not a refutation of P -> Q.

    You are missing the tiny bit of reference to the physical world in each and every number.

There is no such references. Logicians are literalist.

If you prove this, then you can add a 9th step and conclude that comp is wrong. But you keep saying this without proof or argument.

    I cannot help if you cannot see it. 

I cannot see it if you don't show it. In science all assertion can be justified. Either they are hypothesis, or they follow from the hypotheses by the inference rules. This has nothing to do with true and false, note.

    It is the stability of consciousness itself, but this does not make consciousness primitive. I truly need to present a more carefully reasoned argument for the neutrality of consciousness. You refuse to read B. Russell's stuff. OK... You will have to deal with my terrible writings...

I did read it, but you keep repeating the contrary. So we are in a loop. 

    If you could just consider building a toy model of how to 1) represent a pair of amoeba with your construction and 2) model the conversation between them that is possible. 

This is an exercise for undergraduate. See my paper "amoeba, planaria and dreaming machine". The planaria program does this effectively for any number, or eeven constructive ordinal number of interacting cells.

No, I am not. Primitive reality has no properties associated with it. It cannot be assumed or one gets contradictions in one's theory. It is not numebrs or matter or pink unicorns it is only necessary possibility.

Sorry but this does not make sense. 

    No, sigma_1 representations are only what can be non-contradictorily communicated, it is not the whole of reality. 

Of course reality is sigma_2, sigma_3, etc. But sigma_1 can be contradictory. Indeed it can be false also.

    You are not behaving like an agnostic. You are behaving like a "True Believer"!

You should not let your cat walking on your keyboard.

You can't use philosophy to refute a technical point, Stephen. Unfair incongruities will not help. You are the one who keep pretending having seen a flaw; you are the one obliged to show it, or to retract and be more cautious. 

Bruno




Bruno Marchal

unread,
Aug 22, 2012, 6:09:43 AM8/22/12
to everyth...@googlegroups.com
On 22 Aug 2012, at 10:24, Roger Clough wrote:

 
ROGER: According to the Bible, belief is a product of faith or trust, and that trust
does not come from you, it is a gift from God.

We have nothing to do with it,
at least that is what we Lutherans believe.


BRUNO: If it is a gift by God, why a bible?

ROGER: Faith helps us to believe. the Bible, particularly the Gospels, tells us what to believe (that Jesus saved us).

I don't like too much being told what to believe. 



BRUNO: All religions which believe(s) that religion does not apply to machine will remain stuck on earth, the others will conquer the physical universe.

ROGER: ?

In a chinese legend there was a very nice king who decided to put a giant carpet on whole china so that people stop hurting their feet. Then a pragmatic councilor suggested that he would be less expensive and more feasible to cut a little piece of the carpet and attach it under the feet of each chinese people. This is how the chinese invented the shoes! (in that legend).

Likewize, we will not been able to terraform most planets around us in the galaxy. It will be far more easier to transform ourselves, notably into numbers, so that we can upload ourselves in a spreading galactic cybernet. But if your refuse to baptize those numbers because you think that "machine cannot think", then they will baptize themselves, and your religion will be stuck on earth, which at the time will be a museum of carbon, with some luck. Numbers can move at the speed of light.








BRUNO:  We makes sense of data through theory and experiences, but not always consciously. The brain implements many theories learned through evolution. I don't think we can separate data from theory so easily. Somehow a brain is by itself already a theory. Our bodies are divine hypotheses, somehow, assuming comp. We are words in a rational truncation of a quantum field, to take a low level.
 
ROGER: Good.

OK.




BRUNO: I have no problem with pragmatism, as long as it is not used against the freedom of any inquiry, nor used as justified invalid reasoning, or lies and propaganda. Nor used as pretext to cut the funding of fundamental research, as I can give a pragmatic reasons to fund fundamental research in all direction.
 
ROGER: "Pragmatic" used in the vernacular sense is usually another word for "practical".  As in: "Our reason for cutting the program was pragmatic. We simply couldn't afford it."
But that is not exactly what pragmatic means philosophically. Philosophically (see Peirce) pragmatic means that the (pragmatic, not traditional) truth of an issue is what results from actually carrying it out.
As the experimental result is the truth of carrying out an experimental protocol. It may not be true in the ordinary sense. Because by going from the particular to the general,
you are using synthesis, not analysis. Synthesis can provide unexpected truths, so very powerful. Just an experiment can give you totally unexpected results.

OK.



BRUNO:  Pragmatic OK, if honest. That is sometimes difficult with respect to hard question, like "what's going on?". It is normal that we develop wishful thinking, and if that works, as already suggested by the L? formula( in some very weak and formal sense to be sure), a theory has to be assumed always in remaining open it can be false.


ROGER:  [Reflecting] Sorry, I was again being a bit harsh again. You are a kind person.  But pragmatism is as honest as a carefully planned and carried out scientific experiment.

Not necessarily. Cannabis has been made illegal for the pragmatic reason to keep job in the oil industry, and as a simple tool for harassing the Mexicans. Big bandits are often big pragmatic. I would say that pragmatism is orthogonal to honesty/dishonesty.
Pragmatism is often invoked for justifying lies. 
No problem with a notion of pragmatism + honesty, though.





Can you give me a link to the sort of output a comp program would provide ?
Being a natural pragmatist, I learn best from examples.


BRUNO:  By definition, all programs are "comp programs", so an example of output is what happens on your computer's screen right now.
BY comp, I am a program, so another example, is this post.
 
ROGER: OK.

OK.

Bruno


To unsubscribe from this group, send email to everything-li...@googlegroups.com.

For more options, visit this group at http://groups.google.com/group/everything-list?hl=en.

Roger Clough

unread,
Aug 22, 2012, 6:14:31 AM8/22/12
to everything-list
Hi Richard Ruquist
 
Physical law is unextended, while physical objects are extended. 
 
As I understand it, Nature is extended while Supernature is not.
 
So I could call physical law supernatural.
 
 
Roger Clough, rcl...@verizon.net
8/22/2012
Leibniz would say, "If there's no God, we'd have to invent him so everything could function."
----- Receiving the following content -----
Receiver: everything-list
Time: 2012-08-21, 08:12:42
Subject: Re: Re: divine selection versus natural selection

Roger,
You are mistaken. The universe is based on physical laws despite the existence of a supernatural, which I take to be based in the collective set of monads.锟斤拷

The way in which the monads manifest the physical laws and constants of nature is a bonified subject of science, just are the study of COMP is. They may even be related except for the multiverse aspect of COMP.
Richard

On Tue, Aug 21, 2012 at 7:03 AM, Roger Clough <rcl...@verizon.net> wrote:
Hi Richard Ruquist
锟斤拷
I also believe in science. But if you're trying to trash religion
with science, science hasn't a clue nor a tool nor the proper
concepts to even begin with the task. Science does not锟斤拷know
what the meaning of anything is. Period.
锟斤拷
锟斤拷
Roger Clough, rcl...@verizon.net
8/21/2012
Leibniz would say, "If there's no God, we'd have to invent him so everything could function."
----- Receiving the following content -----
Receiver: everything-list
Time: 2012-08-20, 11:18:57
Subject: Re: divine selection versus natural selection

Roger,

Divine selection and natural selection are sourced,
however at differing levels of information integration,
in the "universal锟紺YM monad锟絪ubspace".

Belief can also be a product of science.
I believe science.
Richard
On Mon, Aug 20, 2012 at 5:29 AM, Roger <rcl...@verizon.net> wrote:
Hi Bruno Marchal
According to the Bible, belief is a product of faith or trust, and that trust
does not come from you, it is a gift from God.锟絎e have nothing to do with it,

Richard Ruquist

unread,
Aug 22, 2012, 6:56:13 AM8/22/12
to everyth...@googlegroups.com
Agreed Roger, except that the CYM monads have extension.
The physical laws you speak of are in human imagination
and seemingly not extended, 
but there is necessarily a substantial manifestation of them... 
The supernatural of course extends across the entire universe.
Richard 

On Wed, Aug 22, 2012 at 6:14 AM, Roger Clough <rcl...@verizon.net> wrote:
Hi Richard Ruquist
 
Physical law is unextended, while physical objects are extended. 
 
As I understand it, Nature is extended while Supernature is not.
 
So I could call physical law supernatural.
 
 
Roger Clough, rcl...@verizon.net
8/22/2012
Leibniz would say, "If there's no God, we'd have to invent him so everything could function."
----- Receiving the following content -----
Receiver: everything-list
Time: 2012-08-21, 08:12:42
Subject: Re: Re: divine selection versus natural selection

Roger,
You are mistaken. The universe is based on physical laws despite the existence of a supernatural, which I take to be based in the collective set of monads. 

The way in which the monads manifest the physical laws and constants of nature is a bonified subject of science, just are the study of COMP is. They may even be related except for the multiverse aspect of COMP.
Richard

On Tue, Aug 21, 2012 at 7:03 AM, Roger Clough <rcl...@verizon.net> wrote:
Hi Richard Ruquist
 
I also believe in science. But if you're trying to trash religion
with science, science hasn't a clue nor a tool nor the proper
concepts to even begin with the task. Science does not know
what the meaning of anything is. Period.
 
 
Roger Clough, rcl...@verizon.net
8/21/2012
Leibniz would say, "If there's no God, we'd have to invent him so everything could function."
----- Receiving the following content -----
Receiver: everything-list
Time: 2012-08-20, 11:18:57
Subject: Re: divine selection versus natural selection

Roger,

Divine selection and natural selection are sourced,
however at differing levels of information integration,
in the "universal燙YM monad爏ubspace".

Belief can also be a product of science.
I believe science.
Richard
On Mon, Aug 20, 2012 at 5:29 AM, Roger <rcl...@verizon.net> wrote:
Hi Bruno Marchal
According to the Bible, belief is a product of faith or trust, and that trust
does not come from you, it is a gift from God.燱e have nothing to do with it,

Craig Weinberg

unread,
Aug 22, 2012, 7:19:12 AM8/22/12
to everyth...@googlegroups.com
Here's my post for this morning. I think it relates:


God and The Gaps


If existence is physical reality, then God doesn’t exist. In fact, it could be said that it is the absence of God which allows anything to exist. Said another way, it is the unreality of the totality which maintains the conditions of real (bounded, conditional, consequential) existence. Mortality is the masking of immortal boundarylessness. A kind of cage or lattice for what insists beyond all sequence or consequence. Only because on any ultimate level of description of the universe, there is nothing there to constitute a boundary. There can’t be a boundary, because then it too would be the universe. Whether it’s the laws that make the laws that make the laws of mathematics that make the laws of physics that make the superstrings that make the quantum, or just the limits of our own imagination, on some level, there is an everything, and that everything can be considered to be, in one and only one sense, one and only one thing - a source of signal/order/sense/experience. The singularity of totality.

Existence is a combination of signal and noise. The further in space we get from our own signal, the more we lose reception and the less signal we encounter in relation to noise and space. This loss of reception is true across literal distance as well as metaphorical distance. The more unfamiliar the territory, the less we can relate. As scales get infinitesimal or immense, we objectify and mechanize to reflect the disjunction to our own subjective anchoring of perception.

Space is entropy. In both the thermodynamic sense and the information sense, space is the every gap between signals that contains only the possibility of signal detection. In the void between our body and its surroundings, there is ‘nothing there’ to respond to us on a human level, or a biochemical level, or a physical level. To us a room full of hydrogen gas is an empty space. To hydrogen however, maybe the only empty space is the flux between near collisions - moments of virtual decoherence which define a gaseous state (as opposed to a Bose-Einstein condensate approaching absolute zero). What I am calling virtual decoherence, I think, can be better understood as sense. A pantomime of universal expectation that is empirical - it develops within experience. It makes sure that the expected keeps happening and the unexpected stands out. Momentum-inertia, probability, and significance.

If there were a God, he-she-it-we-they would be zero entropy. Zero space. Zero space but all time. Almost all time. Almost no space. Almost no entropy. Almost pure signal. Because the capacity to generate signal in any sense at all is power and knowledge, and power and knowledge are relative. Relative in the sense that ‘in the land of the blind, the one eyed man is king’, but also in the sense that knowledge itself is relation, and power is the ability to relate knowledge to the self.

One signal in an empty universe is all it takes to define omnipotence and omniscience relative to the emptiness of noise. Once a signal pretends to exist, there is nowhere to put it. It has no entropy - no space in which to exist as an object, so it insists, as a becoming of its own absence. This is the single self-subdividing event that turns eternity into an infinite now (pretending to be history).

If there were a God, it would be all of us put together. The everythingness behind the nothingness behind every nested multiplicity of almost somethingness. No space, no noise, no distance - only infinite significance becoming more significant. Infinite because by definition it is the only game in town. More significance through the paradox of self-insignificance. Being humbled makes us great. A great big pile of crap. Decaying broken forms pretending to be whole and beautiful and perfect to each other for a season. For every season, over and over. It’s only the greatness however, the underlying potential wholeness, beauty and perfection that needs to pretend it is pretending though. The whips and scorns of time form the perfectly imperfect frame for eternity. A pecking order of gravity to squeeze the life out of us, crushing our unreal wilderness of warm soft fiction with the shadows of cold hard facts.

What exists to us is almost almost infinite noise across almost infinite distance, except for a sand-clock trickle of ‘now’ which contains all of the signals that can be squeezed together yet still be held distinct from each other. What exists is 99.99…9% entropy because what insists is 0.00…1% entropy. A distanceless ‘here’ that has nowhere else to be, perpetually spinning the universe around it, over and over. Almost all time (every time except now), almost no space (nowhere except here).


Roger Clough

unread,
Aug 22, 2012, 8:12:37 AM8/22/12
to everything-list
Hi Bruno Marchal
 
    You are healed by admitting you are lost, just as the Bible says.
 
    Submission (the fundamental religious/healing act)
    is required when you are sick or in too much trouble
    to save or help yourself,  so you must turn to something
    or somebody else for help.  You place your faith in them
    to heal you.
 
    40 years ago, to my good fortune, I suffered from alcoholism and
    couldn't stop drinking. My life was a mess.  So I finally turned
    myself in to AA. Even further, they say they can't help you 
    unless you turn yourself over to a Higher Power.
 
    That's what I did and it worked. Now I submit myself to
    the words of the Bible.
         
 
 
Roger Clough, rcl...@verizon.net
8/22/2012
Leibniz would say, "If there's no God, we'd have to invent him so everything could function."
----- Receiving the following content -----
Receiver: everything-list
Time: 2012-08-22, 06:09:43
Subject: Re: A dialog on pragmatism-- in religion and in science

Roger Clough

unread,
Aug 22, 2012, 8:14:10 AM8/22/12
to everything-list
Hi Craig Weinberg
 
God indeed does not physically exist.
The Creator must remain uncreated.
 
 
Roger Clough, rcl...@verizon.net
8/22/2012
Leibniz would say, "If there's no God, we'd have to invent him so everything could function."
----- Receiving the following content -----
Receiver: everything-list
Time: 2012-08-22, 07:19:12
Subject: Re: Reconciling Bruno's Primitives with Multisense

Here's my post for this morning. I think it relates:


God and The Gaps


If existence is physical reality, then God doesn锟絫 exist. In fact, it could be said that it is the absence of God which allows anything to exist. Said another way, it is the unreality of the totality which maintains the conditions of real (bounded, conditional, consequential) existence. Mortality is the masking of immortal boundarylessness. A kind of cage or lattice for what insists beyond all sequence or consequence. Only because on any ultimate level of description of the universe, there is nothing there to constitute a boundary. There can锟絫 be a boundary, because then it too would be the universe. Whether it锟絪 the laws that make the laws that make the laws of mathematics that make the laws of physics that make the superstrings that make the quantum, or just the limits of our own imagination, on some level, there is an everything, and that everything can be considered to be, in one and only one sense, one and only one thing - a source of signal/order/sense/experience. The singularity of totality.

Existence is a combination of signal and noise. The further in space we get from our own signal, the more we lose reception and the less signal we encounter in relation to noise and space. This loss of reception is true across literal distance as well as metaphorical distance. The more unfamiliar the territory, the less we can relate. As scales get infinitesimal or immense, we objectify and mechanize to reflect the disjunction to our own subjective anchoring of perception.

Space is entropy. In both the thermodynamic sense and the information sense, space is the every gap between signals that contains only the possibility of signal detection. In the void between our body and its surroundings, there is 锟絥othing there� to respond to us on a human level, or a biochemical level, or a physical level. To us a room full of hydrogen gas is an empty space. To hydrogen however, maybe the only empty space is the flux between near collisions - moments of virtual decoherence which define a gaseous state (as opposed to a Bose-Einstein condensate approaching absolute zero). What I am calling virtual decoherence, I think, can be better understood as sense. A pantomime of universal expectation that is empirical - it develops within experience. It makes sure that the expected keeps happening and the unexpected stands out. Momentum-inertia, probability, and significance.

If there were a God, he-she-it-we-they would be zero entropy. Zero space. Zero space but all time. Almost all time. Almost no space. Almost no entropy. Almost pure signal. Because the capacity to generate signal in any sense at all is power and knowledge, and power and knowledge are relative. Relative in the sense that 锟絠n the land of the blind, the one eyed man is king�, but also in the sense that knowledge itself is relation, and power is the ability to relate knowledge to the self.

One signal in an empty universe is all it takes to define omnipotence and omniscience relative to the emptiness of noise. Once a signal pretends to exist, there is nowhere to put it. It has no entropy - no space in which to exist as an object, so it insists, as a becoming of its own absence. This is the single self-subdividing event that turns eternity into an infinite now (pretending to be history).

If there were a God, it would be all of us put together. The everythingness behind the nothingness behind every nested multiplicity of almost somethingness. No space, no noise, no distance - only infinite significance becoming more significant. Infinite because by definition it is the only game in town. More significance through the paradox of self-insignificance. Being humbled makes us great. A great big pile of crap. Decaying broken forms pretending to be whole and beautiful and perfect to each other for a season. For every season, over and over. It锟絪 only the greatness however, the underlying potential wholeness, beauty and perfection that needs to pretend it is pretending though. The whips and scorns of time form the perfectly imperfect frame for eternity. A pecking order of gravity to squeeze the life out of us, crushing our unreal wilderness of warm soft fiction with the shadows of cold hard facts.

What exists to us is almost almost infinite noise across almost infinite distance, except for a sand-clock trickle of 锟絥ow� which contains all of the signals that can be squeezed together yet still be held distinct from each other. What exists is 99.99�9% entropy because what insists is 0.00�1% entropy. A distanceless 锟絟ere� that has nowhere else to be, perpetually spinning the universe around it, over and over. Almost all time (every time except now), almost no space (nowhere except here).


--
You received this message because you are subscribed to the Google Groups "Everything List" group.
To view this discussion on the web visit https://groups.google.com/d/msg/everything-list/-/xAPZRAtx_AIJ.

Roger Clough

unread,
Aug 22, 2012, 8:43:58 AM8/22/12
to everything-list
Hi Richard Ruquist
 
In my opinion, the CYM is only extended geometrically on paper.
In a theory, not physically.
 
Although they describe what actually happens physically, they themselves,
being theory, are unextended.
 
It's just like the Pythagorean Theory. It doesn't exist physically as
triangles ihn space, it only exists on paper.
 
 
Roger Clough, rcl...@verizon.net
8/22/2012
Leibniz would say, "If there's no God, we'd have to invent him so everything could function."
----- Receiving the following content -----
Receiver: everything-list
Time: 2012-08-22, 06:56:13
Subject: Re: Re: Re: divine selection versus natural selection

Agreed Roger, except that the CYM monads have extension.
The physical laws you speak of are in human imagination
and seemingly not extended, 
but there is necessarily a substantial manifestation of them... 
The supernatural of course extends across the entire universe.
Richard 
On Wed, Aug 22, 2012 at 6:14 AM, Roger Clough <rcl...@verizon.net> wrote:
Hi Richard Ruquist
 
Physical law is unextended, while physical objects are extended. 
 
As I understand it, Nature is extended while Supernature is not.
 
So I could call physical law supernatural.
 
 
Roger Clough, rcl...@verizon.net
8/22/2012
Leibniz would say, "If there's no God, we'd have to invent him so everything could function."
----- Receiving the following content -----
Receiver: everything-list
Time: 2012-08-21, 08:12:42
Subject: Re: Re: divine selection versus natural selection

Roger,
You are mistaken. The universe is based on physical laws despite the existence of a supernatural, which I take to be based in the collective set of monads.锟斤拷

The way in which the monads manifest the physical laws and constants of nature is a bonified subject of science, just are the study of COMP is. They may even be related except for the multiverse aspect of COMP.
Richard

On Tue, Aug 21, 2012 at 7:03 AM, Roger Clough <rcl...@verizon.net> wrote:
Hi Richard Ruquist
锟斤拷
I also believe in science. But if you're trying to trash religion
with science, science hasn't a clue nor a tool nor the proper
concepts to even begin with the task. Science does not锟斤拷know
what the meaning of anything is. Period.
锟斤拷
锟斤拷
Roger Clough, rcl...@verizon.net
8/21/2012
Leibniz would say, "If there's no God, we'd have to invent him so everything could function."
----- Receiving the following content -----
Receiver: everything-list
Time: 2012-08-20, 11:18:57
Subject: Re: divine selection versus natural selection

Roger,

Divine selection and natural selection are sourced,
however at differing levels of information integration,
in the "universal锟紺YM monad锟絪ubspace".

Belief can also be a product of science.
I believe science.
Richard
On Mon, Aug 20, 2012 at 5:29 AM, Roger <rcl...@verizon.net> wrote:
Hi Bruno Marchal
According to the Bible, belief is a product of faith or trust, and that trust
does not come from you, it is a gift from God.锟絎e have nothing to do with it,

Richard Ruquist

unread,
Aug 22, 2012, 9:21:12 AM8/22/12
to everyth...@googlegroups.com
Theories always refer to physical entities. Otherwise they are unless.
In string theory the monads supernatural entities
but still part of nature.

The way in which the monads manifest the physical laws and constants of nature is a bonified subject of science, just are the study of COMP is. They may even be related except for the multiverse aspect of COMP.
Richard

On Tue, Aug 21, 2012 at 7:03 AM, Roger Clough <rcl...@verizon.net> wrote:
Hi Richard Ruquist
 
I also believe in science. But if you're trying to trash religion
with science, science hasn't a clue nor a tool nor the proper
concepts to even begin with the task. Science does not know
what the meaning of anything is. Period.
 
 
Roger Clough, rcl...@verizon.net
8/21/2012
Leibniz would say, "If there's no God, we'd have to invent him so everything could function."
----- Receiving the following content -----
Receiver: everything-list
Time: 2012-08-20, 11:18:57
Subject: Re: divine selection versus natural selection

Roger,

Divine selection and natural selection are sourced,
however at differing levels of information integration,
in the "universal燙YM monad爏ubspace".

Belief can also be a product of science.
I believe science.
Richard
On Mon, Aug 20, 2012 at 5:29 AM, Roger <rcl...@verizon.net> wrote:
Hi Bruno Marchal
According to the Bible, belief is a product of faith or trust, and that trust
does not come from you, it is a gift from God.燱e have nothing to do with it,

Roger Clough

unread,
Aug 22, 2012, 9:31:31 AM8/22/12
to everything-list
Hi Richard Ruquist
 
Yes, theories REFER to physical entitires, but they are NOT the entities themselves.
 
This is kindergarten stuff, Richard, give me a break.
 
 
Roger Clough, rcl...@verizon.net
8/22/2012
Leibniz would say, "If there's no God, we'd have to invent him so everything could function."
----- Receiving the following content -----
Receiver: everything-list
Time: 2012-08-22, 09:21:12
Subject: Re: Re: Re: Re: divine selection versus natural selection

Roger,
You are mistaken. The universe is based on physical laws despite the existence of a supernatural, which I take to be based in the collective set of monads.锟斤拷

The way in which the monads manifest the physical laws and constants of nature is a bonified subject of science, just are the study of COMP is. They may even be related except for the multiverse aspect of COMP.
Richard

On Tue, Aug 21, 2012 at 7:03 AM, Roger Clough <rcl...@verizon.net> wrote:
Hi Richard Ruquist
锟斤拷
I also believe in science. But if you're trying to trash religion
with science, science hasn't a clue nor a tool nor the proper
concepts to even begin with the task. Science does not锟斤拷know
what the meaning of anything is. Period.
锟斤拷
锟斤拷
Roger Clough, rcl...@verizon.net
8/21/2012
Leibniz would say, "If there's no God, we'd have to invent him so everything could function."
----- Receiving the following content -----
Receiver: everything-list
Time: 2012-08-20, 11:18:57
Subject: Re: divine selection versus natural selection

Roger,

Divine selection and natural selection are sourced,
however at differing levels of information integration,
in the "universal锟紺YM monad锟絪ubspace".

Belief can also be a product of science.
I believe science.
Richard
On Mon, Aug 20, 2012 at 5:29 AM, Roger <rcl...@verizon.net> wrote:
Hi Bruno Marchal
According to the Bible, belief is a product of faith or trust, and that trust
does not come from you, it is a gift from God.锟絎e have nothing to do with it,

Richard Ruquist

unread,
Aug 22, 2012, 9:47:48 AM8/22/12
to everyth...@googlegroups.com
Of course theories are not the physical entities.
But the laws of physics are a good approximation
of how the universe works 
and string theory just says 
where they come from.

The way in which the monads manifest the physical laws and constants of nature is a bonified subject of science, just are the study of COMP is. They may even be related except for the multiverse aspect of COMP.
Richard

On Tue, Aug 21, 2012 at 7:03 AM, Roger Clough <rcl...@verizon.net> wrote:
Hi Richard Ruquist
 
I also believe in science. But if you're trying to trash religion
with science, science hasn't a clue nor a tool nor the proper
concepts to even begin with the task. Science does not know
what the meaning of anything is. Period.
 
 
Roger Clough, rcl...@verizon.net
8/21/2012
Leibniz would say, "If there's no God, we'd have to invent him so everything could function."
----- Receiving the following content -----
Receiver: everything-list
Time: 2012-08-20, 11:18:57
Subject: Re: divine selection versus natural selection

Roger,

Divine selection and natural selection are sourced,
however at differing levels of information integration,
in the "universal燙YM monad爏ubspace".

Belief can also be a product of science.
I believe science.
Richard
On Mon, Aug 20, 2012 at 5:29 AM, Roger <rcl...@verizon.net> wrote:
Hi Bruno Marchal
According to the Bible, belief is a product of faith or trust, and that trust
does not come from you, it is a gift from God.燱e have nothing to do with it,

Bruno Marchal

unread,
Aug 22, 2012, 2:00:31 PM8/22/12
to everyth...@googlegroups.com
On 22 Aug 2012, at 14:12, Roger Clough wrote:

Hi Bruno Marchal
 
    You are healed by admitting you are lost, just as the Bible says.
 
    Submission (the fundamental religious/healing act)
    is required when you are sick or in too much trouble
    to save or help yourself,  so you must turn to something
    or somebody else for help.  You place your faith in them
    to heal you.
 
    40 years ago, to my good fortune, I suffered from alcoholism and
    couldn't stop drinking. My life was a mess.  So I finally turned
    myself in to AA. Even further, they say they can't help you 
    unless you turn yourself over to a Higher Power.
 
    That's what I did and it worked. Now I submit myself to
    the words of the Bible.


Nobody like addiction, nor slavery. If all drugs were legal, the less addictive would be the most popular. 

Today the most toxic and addictive common one are still alcohol and smoked tobacco. The current health politics illustrates that the human sciences are still at the stone age, even if the cause of this is just mundane stealing.

Today we know plants which cures addiction rather efficiently (Tabernanthe iboga, Salvia divinorum, ...).

Anyway way I am happy for you that your method worked for you. And if the bible can inspire you, no problem, as long as you don't claim something like "that's the truth". 

It can reflect some important truth, but it can also reflect some important mistake, where mistake is relative to different conception of "the spiritual reality".

Some religion allows God's creatures to ask God for forgiveness, but apparently comp is more simple minded on this. With comp if you ask God for forgiveness you are sent to "hell", because if you ask forgiveness it means that you have sin, and that's enough. Only those not asking forgiveness have a chance to go to heaven perhaps.

Submission does not ring to well for me, but I can buy it in the sense of a "let it go", an acceptance of the inevitable, an opening to the lack of control and ignorance, which can also be an openingand a self-abandon to an higher power, but then my "God axiom", which fits very nicely with comp,  is that such a "God" has no public name, so here a legend or any text at all have not to be taken too much literally.

All moral can be taught only by example, not by summon or text. With comp, you are a text, and texts are terrestrial finite, only pointing to the infinite.

With comp there are no public intermediate between God and the Soul. The notion of "priest" is already problematic, like an insult to God, like if the Almighty was not able to manage the situation. This does not mean that community is not possible, just that there are no guru, only clowns. The theurgy is possible, but only as a gentle self-mockery, or as an occasion to try some technical spiritual path, if someone you personally trust suggest.

With comp there are still inner intermediate between God and the Soul. It is a conjecture, but evidences exists. See the Plotinus paper for the translation in arithmetic. In a nutshell, with arithmetical in front (even if technically the knower and the feeler are not arithmetical)

god === truth
noùs === proof
soul === knower
intelligible matter === observer
sensible matter === feeler 

It is the eight person views: p, Bp, Bp & p, Bp & Dt, Bp & Dt & p. This makes five but three of them splits on earth/heaven (by the G/G*) distinction, making them eight.

Just listening to the universal machine, 

Bruno

Roger Clough

unread,
Aug 23, 2012, 7:14:56 AM8/23/12
to everything-list
Hi Bruno Marchal
 
There is objective truth, meaning truth that can be proven and expressed in symbols that you can share.
Thus it is not personal or private. Scientific and computer truth is like that.
 
But the more profound truth is subjective, because it is personal, meaning that you simply feel or sense,
that you simply know without proof. That cannot be put into words without distorting or destroying.
For example, you know you are alive. Religious truth is like that. You say "proof schmoof".
Kierkegaard thus said "Truth is subjective."
 
When you prove the 2 + 2 + 4 and show it, that is objective proof.
But to get there I think you have to use intuition, which is subjective.
And the acceptance of that truth is subjective.
 
 
 
 
Roger Clough, rcl...@verizon.net
8/23/2012
Leibniz would say, "If there's no God, we'd have to invent him so everything could function."
----- Receiving the following content -----
Receiver: everything-list
Time: 2012-08-22, 14:00:31
Subject: Re: A dialog on pragmatism-- in religion and in science

no锟絪 === proof

Bruno Marchal

unread,
Aug 23, 2012, 2:17:50 PM8/23/12
to everyth...@googlegroups.com
???

The only differences with elementary propositional logic are that we have one symbol more, the box "[]", and one more inference rule. 

It is a unary operator symbol, so if X is a formula, []X is a formula, like ~X.

The inference rule is that you can derive []p from p. Careful, this does not make p -> []p true in most modal logic.

I wrote often the box [] by using the letter B.

In the axiom above, it is better to not interpret the box, as this can confuse with the representation theorem which associate "meaning" mathematically.

I have often talked about Bp and Bp & p, with Bp having the arithmetical provability meaning (Gödel 1931).
G above is the logic of Gödel's beweisbar predicate. For example the second incompleteness theorem is given by Dt -> ~BDt, or <>t -> ~[]<>t, or consistent('t') -> NOT PROVABLE (CONSISTENT 't')), with for example t = "0=0", et 't' = Gödel number of "0=0".

S4Grz above is the corresponding logic of the first person associated to the machine, given by beweisbar('p') & p, following Theatetus, and then Boolos, Goldblatt, Artemov. I have provided many explanations on this list, including an introduction to modal logic and the Kripke semantics, but you can also open some book in logic to help yourself.

G and S4Grz are the two machineries illustrating (and formalizing completely at the propositional modal) two important arithmetical hypostases discovered by the UM when looking inward. G is the logic of third person self-reference and S4Grz is the logic of the first person self-reference.

There are six other hypostases, or machine's points of view, three of them playing a role in the "creation of the collective persistent matter hallucination. Comp makes obligatory that persistence, and it can be tested, and it can be argued that the presence of p -> []<>p as a theorem in SGrz1 and Z1* and X1* confirms it in great part. Interactions can be defined in a manner similar to Girard, and then tested on those "material hypostases". I think that this is explained in the second part of the sane04 paper.
The "1" added to the system refers to the fact that we eventually limit the arithmetical translation of the sentence letters (p, q, r, ...) to the sigma_1 sentences, which "models" the UD in arithmetic.

In particular Richard Ruquist's theory that fundamental physics is given by string theory becomes testable with respect to comp, as UDA shows that the physics is entirely retrievable from the S4Grz1, Z1* and/or X1*, and their first order modal extension. 

It is not as difficult as most paper your refer to, and it is only one paper, and you got the chance to ask any question to the author :)

You recently allude to a disagreement between us, but I (meta)disagree with such an idea: I use the scientific method, which means that you cannot disagree with me without showing a precise flaw at some step in the reasoning.

You seem to follow the seven first steps, so that in particular you grasp apparently that COMP + ROBUST-UNIVERSE entails the reversal physics/arithmetic, and the explanation why qualia and quanta separate. Are you sure you got this? Step 8 just eliminates the "ROBUST-UNIVERSE" assumption in step 7.

Then AUDA translates everything in UDA in terms of numbers and sequences of numbers, making the "body problem" into a problem of arithmetic. It is literally an infinite interview with the universal machine, made finite thanks to the modal logic above, and thanks to the Solovay arithmetical completeness theorem.

You cannot both claim that there is a flaw, and at the same time invoke your dyslexia to justify you don't do the technical work to present it. 

Bruno



Stephen P. King

unread,
Aug 23, 2012, 4:26:34 PM8/23/12
to everyth...@googlegroups.com
On 8/23/2012 2:17 PM, Bruno Marchal wrote:
> You recently allude to a disagreement between us, but I (meta)disagree
> with such an idea: I use the scientific method, which means that you
> cannot disagree with me without showing a precise flaw at some step in
> the reasoning.
>
> You seem to follow the seven first steps, so that in particular you
> grasp apparently that COMP + ROBUST-UNIVERSE entails the reversal
> physics/arithmetic, and the explanation why qualia and quanta
> separate. Are you sure you got this? Step 8 just eliminates the
> "ROBUST-UNIVERSE" assumption in step 7.
>
Dear Bruno,

I claim that step 8 is invalidated by the fact that you must use
the physical medium to interact (communicate) the abstract concept. If
we take step 8 literally, this would not occur and thus obtain a
contradiction. You seem to not realize the price that you must pay for
immaterialism.

Stephen P. King

unread,
Aug 23, 2012, 4:39:18 PM8/23/12
to everyth...@googlegroups.com
On 8/23/2012 2:17 PM, Bruno Marchal wrote:
> Then AUDA translates everything in UDA in terms of numbers and
> sequences of numbers, making the "body problem" into a problem of
> arithmetic. It is literally an infinite interview with the universal
> machine, made finite thanks to the modal logic above, and thanks to
> the Solovay arithmetical completeness theorem.
>
> You cannot both claim that there is a flaw, and at the same time
> invoke your dyslexia to justify you don't do the technical work to
> present it.
Dear Bruno,

It is the body problem that is your problem. There is no solution
for it in strict immaterialism. Immaterials cannot interact, they have
nothing with which to "touch" each other. All they can do is imagine the
possibility in the sense of a representation of the logical operation of
"imagining the possibility of X" (a string of recursively enumerable
coding the computational simulation of X).
This would be fine and you do a wonderful job of dressing this up
in your work, but the body problem is just another name for the
concurrency problem. It is the scarcity of physical resources that
forces solutions to be found and this is exactly what Pratt shows us how
to work out. Mutual consistency restrictions is the dual to resource
availability!

My dyslexia prevents me from writing long strings of symbolic
logical codes, but I can write English (and some Spanish) well enough to
communicate with you and I can read and comprehend complex texts very
well. ;-)


By the way, I only asked from a verbal -> written English version
of your symbols strings, not a condensed explanation of it. I do
appreciate what you wrote, but it was not what I was asking for.

G is

[](p -> q) -> ([]p -> []q)
[]p -> [][]p
[]([]p -> p) -> []p

with the rules A, A->B / B and A / []A

S4Grz is

[](p -> q) -> ([]p -> []q)
[]p -> [][]p
[]([](p -> []p) -> p) -> p

with the rules A, A->B / B and A / []A

These symbols have verbal words associated with them, no? If you
where to read of these sentences aloud. What English sounds would come
out of your mouth? Could those words be transcribed here for the readers
of the Everything List? What word corresponds, for instance, to "->" ?
Implies?

Roger Clough

unread,
Aug 24, 2012, 6:15:44 AM8/24/12
to everything-list
Hi Bruno Marchal
 
 
Could you explain a little about Bp & p duality ? Are they both
analytic, or does one of them us synthetic logic ?
 
 
Roger Clough, rcl...@verizon.net
8/24/2012
Leibniz would say, "If there's no God, we'd have to invent him so everything could function."
----- Receiving the following content -----
Receiver: everything-list
Time: 2012-08-23, 14:17:50
Subject: Re: Male Proof and female acceptance of proof


I have often talked about Bp and Bp & p, with Bp having the arithmetical provability meaning (G锟絛el 1931).
G above is the logic of G锟絛el's beweisbar predicate. For example the second incompleteness theorem is given by Dt -> ~BDt, or <>t -> ~[]<>t, or consistent('t') -> NOT PROVABLE (CONSISTENT 't')), with for example t = "0=0", et 't' = G锟絛el number of "0=0".

S4Grz above is the corresponding logic of the first person associated to the machine, given by beweisbar('p') & p, following Theatetus, and then Boolos, Goldblatt, Artemov. I have provided many explanations on this list, including an introduction to modal logic and the Kripke semantics, but you can also open some book in logic to help yourself.

G and S4Grz are the two machineries illustrating (and formalizing completely at the propositional modal) two important arithmetical hypostases discovered by the UM when looking inward. G is the logic of third person self-reference and S4Grz is the logic of the first person self-reference.

There are six other hypostases, or machine's points of view, three of them playing a role in the "creation of the collective persistent matter hallucination. Comp makes obligatory that persistence, and it can be tested, and it can be argued that the presence of p -> []<>p as a theorem in SGrz1 and Z1* and X1* confirms it in great part. Interactions can be defined in a manner similar to Girard, and then tested on those "material hypostases". I think that this is explained in the second part of the sane04 paper.
The "1" added to the system refers to the fact that we eventually limit the arithmetical translation of the sentence letters (p, q, r, ...) to the sigma_1 sentences, which "models" the UD in arithmetic.

In particular Richard Ruquist's theory that fundamental physics is given by string theory becomes testable with respect to comp, as UDA shows that the physics is entirely retrievable from the S4Grz1, Z1* and/or X1*, and their first order modal extension. 

It is not as difficult as most paper your refer to, and it is only one paper, and you got the chance to ask any question to the author :)

You recently allude to a disagreement between us, but I (meta)disagree with such an idea: I use the scientific method, which means that you cannot disagree with me without showing a precise flaw at some step in the reasoning.

You seem to follow the seven first steps, so that in particular you grasp apparently that COMP + ROBUST-UNIVERSE entails the reversal physics/arithmetic, and the explanation why qualia and quanta separate. Are you sure you got this? Step 8 just eliminates the "ROBUST-UNIVERSE" assumption in step 7.

Then AUDA translates everything in UDA in terms of numbers and sequences of numbers, making the "body problem" into a problem of arithmetic. It is literally an infinite interview with the universal machine, made finite thanks to the modal logic above, and thanks to the Solovay arithmetical completeness theorem.

You cannot both claim that there is a flaw, and at the same time invoke your dyslexia to justify you don't do the technical work to present it. 

Bruno

Roger Clough

unread,
Aug 24, 2012, 7:55:25 AM8/24/12
to everything-list
Hi Stephen P. King
 
True, materials don't actually interact in Idealism, but the Supreme intelligence
insures that the same result happens. In other words, you can't tell the difference.
So at least in one place Leibniz says, "True, they don't actually interact,
because ideas as substances cannot interact, but there's no harm in saying
that they do."
 
 
Roger Clough, rcl...@verizon.net
8/24/2012
Leibniz would say, "If there's no God, we'd have to invent him so everything could function."
----- Receiving the following content -----
Time: 2012-08-23, 16:39:18
Subject: Re: Male Proof and female acceptance of proof

--
You received this message because you are subscribed to the Google Groups "Everything List" group.
To post to this group, send email to everyth...@googlegroups.com.
To unsubscribe from this group, send email to everything-list+unsub...@googlegroups.com.

Stephen P. King

unread,
Aug 24, 2012, 8:31:24 AM8/24/12
to everyth...@googlegroups.com
Dear Roger,

    I only see one glaring gap in your explanation here: the chain of non-interaction leads all the way up to the supremum where God is essentially and effectively (not)interacting with itself. Is this not the very definition of Solipsism? How is the problem of solipsism not even infinitely more acute for God? God has no peers, so it naturally implies that the ordinary problem of solipsism - what does one human solipsist say to another? - is a mute point, but somewhere and somehow the appearance of plurality of entities must appear in order for us to explain appearences. This is the very same question that I keep asking Bruno and he seems to not understand the question: How does a plurality of minds emerge from the One such that they have an appearance of interactions without falling into the morass of allowing for everythign and thus, ultimately, explaining nothing?
    It seems to me that Leibniz was working out the Everything vs. Nothing problem of existence from a different point of view with the monadology.

Richard Ruquist

unread,
Aug 24, 2012, 8:54:31 AM8/24/12
to everyth...@googlegroups.com
Stephan,
I find it interesting that according to my Roman Catholic professor theologian friend,
 God has intention but but intelligence. That would seem to be consistent with what you say below. I'll have to ask him if the church came to that viewpoint do to the " ordinary problem of solipsism".


--
You received this message because you are subscribed to the Google Groups "Everything List" group.
To post to this group, send email to everyth...@googlegroups.com.
To unsubscribe from this group, send email to everything-li...@googlegroups.com.

Roger Clough

unread,
Aug 24, 2012, 9:00:15 AM8/24/12
to everything-list
Hi Stephen P. King
 
No, God communes with us (and the entire universe) and we also commune with him,
depending on our clarity of "vision" and intelligence, and perhaps desire, don't know yet.
According to Lutheran orthodoxy (L was a Lutheran),  God, since He causes all, can
cause us to commune with Him and have faith. I suppose wiping out sins is also
there, but so far no mention of Jesus. My conjecture is that Jesus is between
man and God, but since He is both, it may be more complicated.
 
Roger Clough, rcl...@verizon.net
8/24/2012
Leibniz would say, "If there's no God, we'd have to invent him so everything could function."
----- Receiving the following content -----
Receiver: everything-list
Time: 2012-08-24, 08:31:24

Roger Clough

unread,
Aug 24, 2012, 9:09:00 AM8/24/12
to everything-list
Hi Richard Ruquist
 
According to Aquinas. God IS intelligence.
 
 
Roger Clough, rcl...@verizon.net
8/24/2012
Leibniz would say, "If there's no God, we'd have to invent him so everything could function."
----- Receiving the following content -----
Receiver: everything-list
Time: 2012-08-24, 08:54:31
Subject: Re: Male Proof and female acceptance of proof

Stephan,
I find it interesting that according to my Roman Catholic professor theologian friend,
锟紾od has intention but but intelligence. That would seem to be consistent with what you say below. I'll have to ask him if the church came to that viewpoint do to the " ordinary problem of solipsism".


On Fri, Aug 24, 2012 at 8:31 AM, Stephen P. King <step...@charter.net> wrote:
Dear Roger,

锟斤拷� I only see one glaring gap in your explanation here: the chain of non-interaction leads all the way up to the supremum where God is essentially and effectively (not)interacting with itself. Is this not the very definition of Solipsism? How is the problem of solipsism not even infinitely more acute for God? God has no peers, so it naturally implies that the ordinary problem of solipsism - what does one human solipsist say to another? - is a mute point, but somewhere and somehow the appearance of plurality of entities must appear in order for us to explain appearences. This is the very same question that I keep asking Bruno and he seems to not understand the question: How does a plurality of minds emerge from the One such that they have an appearance of interactions without falling into the morass of allowing for everythign and thus, ultimately, explaining nothing?
锟斤拷� It seems to me that Leibniz was working out the Everything vs. Nothing problem of existence from a different point of view with the monadology.
锟斤拷锟斤拷锟絀t is the body problem that is your problem. There is no solution
for it in strict immaterialism. Immaterials cannot interact, they have
nothing with which to "touch" each other. All they can do is imagine the
possibility in the sense of a representation of the logical operation of
"imagining the possibility of X" (a string of recursively enumerable
coding the computational simulation of X).
锟斤拷锟斤拷锟絋his would be fine and you do a wonderful job of dressing this up
in your work, but the body problem is just another name for the
concurrency problem. It is the scarcity of physical resources that
forces solutions to be found and this is exactly what Pratt shows us how
to work out. Mutual consistency restrictions is the dual to resource
availability!

锟斤拷锟斤拷锟組y dyslexia prevents me from writing long strings of symbolic
logical codes, but I can write English (and some Spanish) well enough to
communicate with you and I can read and comprehend complex texts very
well. ;-)


锟斤拷锟斤拷锟紹y the way, I only asked from a verbal -> written English version
of your symbols strings, not a condensed explanation of it. I do
appreciate what you wrote, but it was not what I was asking for.

G is

[](p -> q) -> ([]p -> []q)
[]p -> [][]p
[]([]p -> p) -> []p

with the rules A, A->B / B and A / []A

S4Grz is

[](p -> q) -> ([]p -> []q)
[]p -> [][]p
[]([](p -> []p) -> p) -> p

with the rules A, A->B / B and A / []A

锟斤拷锟斤拷锟絋hese symbols have verbal words associated with them, no? If you
where to read of these sentences aloud. What English sounds would come
out of your mouth? Could those words be transcribed here for the readers
of the Everything List? What word corresponds, for instance, to "->" ?
Implies?

--

--
You received this message because you are subscribed to the Google Groups "Everything List" group.
To post to this group, send email to everyth...@googlegroups.com.
To unsubscribe from this group, send email to everything-li...@googlegroups.com.
For more options, visit this group at http://groups.google.com/group/everything-list?hl=en.

Richard Ruquist

unread,
Aug 24, 2012, 9:28:37 AM8/24/12
to everyth...@googlegroups.com
Hi Roger,

Then my friend is either blasphemous or the church has evolved since then.
Recent history of the church suggests that it evolves but rather conservatively.
Richard

On Fri, Aug 24, 2012 at 9:09 AM, Roger Clough <rcl...@verizon.net> wrote:
Hi Richard Ruquist
 
According to Aquinas. God IS intelligence.
 
 
Roger Clough, rcl...@verizon.net
8/24/2012
Leibniz would say, "If there's no God, we'd have to invent him so everything could function."
----- Receiving the following content -----
Receiver: everything-list
Time: 2012-08-24, 08:54:31
Subject: Re: Male Proof and female acceptance of proof

Stephan,
I find it interesting that according to my Roman Catholic professor theologian friend,
燝od has intention but but intelligence. That would seem to be consistent with what you say below. I'll have to ask him if the church came to that viewpoint do to the " ordinary problem of solipsism".


On Fri, Aug 24, 2012 at 8:31 AM, Stephen P. King <step...@charter.net> wrote:
Dear Roger,

牋� I only see one glaring gap in your explanation here: the chain of non-interaction leads all the way up to the supremum where God is essentially and effectively (not)interacting with itself. Is this not the very definition of Solipsism? How is the problem of solipsism not even infinitely more acute for God? God has no peers, so it naturally implies that the ordinary problem of solipsism - what does one human solipsist say to another? - is a mute point, but somewhere and somehow the appearance of plurality of entities must appear in order for us to explain appearences. This is the very same question that I keep asking Bruno and he seems to not understand the question: How does a plurality of minds emerge from the One such that they have an appearance of interactions without falling into the morass of allowing for everythign and thus, ultimately, explaining nothing?
牋� It seems to me that Leibniz was working out the Everything vs. Nothing problem of existence from a different point of view with the monadology.
牋牋營t is the body problem that is your problem. There is no solution
for it in strict immaterialism. Immaterials cannot interact, they have
nothing with which to "touch" each other. All they can do is imagine the
possibility in the sense of a representation of the logical operation of
"imagining the possibility of X" (a string of recursively enumerable
coding the computational simulation of X).
牋牋燭his would be fine and you do a wonderful job of dressing this up
in your work, but the body problem is just another name for the
concurrency problem. It is the scarcity of physical resources that
forces solutions to be found and this is exactly what Pratt shows us how
to work out. Mutual consistency restrictions is the dual to resource
availability!

牋牋燤y dyslexia prevents me from writing long strings of symbolic
logical codes, but I can write English (and some Spanish) well enough to
communicate with you and I can read and comprehend complex texts very
well. ;-)


牋牋燘y the way, I only asked from a verbal -> written English version
of your symbols strings, not a condensed explanation of it. I do
appreciate what you wrote, but it was not what I was asking for.

G is

[](p -> q) -> ([]p -> []q)
[]p -> [][]p
[]([]p -> p) -> []p

with the rules A, A->B / B and A / []A

S4Grz is

[](p -> q) -> ([]p -> []q)
[]p -> [][]p
[]([](p -> []p) -> p) -> p

with the rules A, A->B / B and A / []A

牋牋燭hese symbols have verbal words associated with them, no? If you
where to read of these sentences aloud. What English sounds would come
out of your mouth? Could those words be transcribed here for the readers
of the Everything List? What word corresponds, for instance, to "->" ?
Implies?

--

--
You received this message because you are subscribed to the Google Groups "Everything List" group.
To post to this group, send email to everyth...@googlegroups.com.
To unsubscribe from this group, send email to everything-li...@googlegroups.com.
For more options, visit this group at http://groups.google.com/group/everything-list?hl=en.

--
You received this message because you are subscribed to the Google Groups "Everything List" group.
To post to this group, send email to everyth...@googlegroups.com.
To unsubscribe from this group, send email to everything-li...@googlegroups.com.
For more options, visit this group at http://groups.google.com/group/everything-list?hl=en.

Stephen P. King

unread,
Aug 24, 2012, 12:44:04 PM8/24/12
to everyth...@googlegroups.com
Dear Roger,

       I agree with what you are saying regarding the communion concept, but I am interested in some kind of explanation for it that is not just some appeal to authority.


On 8/24/2012 9:00 AM, Roger Clough wrote:
Hi Stephen P. King
 
No, God communes with us (and the entire universe) and we also commune with him,
depending on our clarity of "vision" and intelligence, and perhaps desire, don't know yet.
According to Lutheran orthodoxy (L was a Lutheran),  God, since He causes all, can
cause us to commune with Him and have faith. I suppose wiping out sins is also
there, but so far no mention of Jesus. My conjecture is that Jesus is between
man and God, but since He is both, it may be more complicated.
 
Roger Clough, rcl...@verizon.net
8/24/2012

Bruno Marchal

unread,
Aug 25, 2012, 8:15:08 AM8/25/12
to everyth...@googlegroups.com
To invalidate a step in a proof, you must mention what is not valid in
the derivation. Here you just introduce a statement without proof, nor
definition (your statement that we have to use a primary physical
medium to interact), and you avoid the reasoning. This is not a valid
use of philosophy in science. You might refute the theory of evolution
by saying that it is quite well, and that it explains well how birds
and plants evolves, but that it fails miserably to explain how God
made the world in six days.

The work shows that immaterialism is the price for computationalism.
The price of immaterialism itself is the object of the whole
reasoning: to extract the appearance of a physical medium without
assuming a physical medium.


At first it looks simple, as Tegmark and Schmidhuber proposal might
suggest, given that physical realities, with interacting bodies are
easy to show to exist in arithmetic (take for example the program
emulating the SWE of the galaxy). But it is part of the work to show
that such explanation omit the first person indeterminacy, and so
can't work, as we have to justify the relative stability, from the
first person point of view, with respect to the comp multiplication of
computations, a priori different from QM. But the logic of self-
reference gives the constraints which are needed, and the rest is math
and physics, to test the comp hypothesis.


Bruno




>
> --
> Onward!
>
> Stephen
>
> "Nature, to be commanded, must be obeyed."
> ~ Francis Bacon
>
>
> --
> You received this message because you are subscribed to the Google
> Groups "Everything List" group.
> To post to this group, send email to everyth...@googlegroups.com.
> To unsubscribe from this group, send email to everything-li...@googlegroups.com
> .
> For more options, visit this group at http://groups.google.com/group/everything-list?hl=en
> .
>

http://iridia.ulb.ac.be/~marchal/



Bruno Marchal

unread,
Aug 25, 2012, 8:42:49 AM8/25/12
to everyth...@googlegroups.com

On 23 Aug 2012, at 22:39, Stephen P. King wrote:

> On 8/23/2012 2:17 PM, Bruno Marchal wrote:
>> Then AUDA translates everything in UDA in terms of numbers and
>> sequences of numbers, making the "body problem" into a problem of
>> arithmetic. It is literally an infinite interview with the
>> universal machine, made finite thanks to the modal logic above, and
>> thanks to the Solovay arithmetical completeness theorem.
>>
>> You cannot both claim that there is a flaw, and at the same time
>> invoke your dyslexia to justify you don't do the technical work to
>> present it.
> Dear Bruno,
>
> It is the body problem that is your problem.


No. It is the problem of all computationalists. That is the result of
the work. Then I show how to translate that problem in arithmetic.


> There is no solution for it in strict immaterialism.

Proof?



> Immaterials cannot interact,

Proofs? (btw, this is not needed, we need only dreams of interaction,
but then immaterial can interact, as it is obvious with comp: in the
arithmetical simulations (thus truncated digitally) of the galaxies,
they interact through gravitation, or you are coming up with
metaphysical primary sort of material interaction which nobody has
ever proved the existence.



> they have nothing with which to "touch" each other. All they can do
> is imagine the possibility in the sense of a representation of the
> logical operation of "imagining the possibility of X" (a string of
> recursively enumerable coding the computational simulation of X).
> This would be fine and you do a wonderful job of dressing this up
> in your work, but the body problem is just another name for the
> concurrency problem.

It is much vaster. We have to justify appearance of space, time,
force, physical constant, the quantum, etc. Concurrrency is easy to
explain, compared to gravitation. But it remains hard to justify the
stability if any of this. The only way to do that is in justifying
some phase randomization from only the self-reference logic. Here the
"p-> BDp" is a sort of arithemtical miracle, because it explains
already the less trivial part.



> It is the scarcity of physical resources that forces solutions to be
> found and this is exactly what Pratt shows us how to work out.
> Mutual consistency restrictions is the dual to resource availability!
>
> My dyslexia prevents me from writing long strings of symbolic
> logical codes, but I can write English (and some Spanish) well
> enough to communicate with you and I can read and comprehend complex
> texts very well. ;-)

This contradicts what you say about UDA.



>
>
> By the way, I only asked from a verbal -> written English version
> of your symbols strings, not a condensed explanation of it. I do
> appreciate what you wrote, but it was not what I was asking for.
>
> G is
>
> [](p -> q) -> ([]p -> []q)
> []p -> [][]p
> []([]p -> p) -> []p
>
> with the rules A, A->B / B and A / []A
>
> S4Grz is
>
> [](p -> q) -> ([]p -> []q)
> []p -> [][]p
> []([](p -> []p) -> p) -> p
>
> with the rules A, A->B / B and A / []A
>
> These symbols have verbal words associated with them, no? If you
> where to read of these sentences aloud. What English sounds would
> come out of your mouth?

?
It is logic. Whatever english sentence you give will be for the
intended meaning, or the intended meaning of some mathematical
intepretations of it. I gavce them just to illustrates a machinery.
you must read

[](p->q)->([]p -> []q) in the follwing literal way:

box left parenthesis p implies q right parenthesis ...

It is like giving a picture of DNA molecules "ATTCAGTTAAACTCCGTA ..." .

In logic we don't interpret the formula. You must look at "[](p->q)-
>([]p -> []q)" at a non interpreted finite molecule. And then you can
look at an ference rule like A, A->B/B as an enzyme which will take
two molecules, like [](p->q) and [](p->q)->([]p -> []q), and catlyse a
reaction leading to ([]p -> []q).



> Could those words be transcribed here for the readers of the
> Everything List? What word corresponds, for instance, to "->" ?
> Implies?

It might, and it is, depending what you mean by "implies", it can be
birds and frogs in other interpretation, and it does not matter,
because the machinery is build so that the reasoning will not depend
from the interpretation. That is the whole what logic is about.
Interpretation is defined mathematically, and provides another
chapter in logic. Then in applied logic, another layer of
interpretation is given, and this one can be rendered in english, but
to give it before can only be confusing.

Bruno


http://iridia.ulb.ac.be/~marchal/



Bruno Marchal

unread,
Aug 25, 2012, 9:51:28 AM8/25/12
to everyth...@googlegroups.com
On 24 Aug 2012, at 12:15, Roger Clough wrote:

Hi Bruno Marchal
 
 
Could you explain a little about Bp & p duality ? Are they both
analytic, or does one of them us synthetic logic ?

I void using "synthetic" and "analytic". Bp is a modal formula and its interpretation here is provable('p') where provable is Gödel provability predicate, entirely defined in arithmetic,, and " 'p' " is for the arithmetical description of some sentence of arithmetic, but 'p' is for the arithmetical proposition itself, which cannot be described as such. Bp & p means that the machine can prove that she can prove p, and that it is the case that p, or, if you prefer that p is true for the machine (believed by the machine).

By incompleteness, the machine cannot prove Bf -> f (provable "0=1" implies that 0 = 1). That is equivalent with ~provable (false), which is equivalent with "I am consistent", with a third person description of I.

So, if the machine is sound, we do have Bf -> f, but the machine cannot know that. Yet, Bf & f typically implies false (as p & q implies p). So, although Bp & p is equivalent with Bp (both proves the same proposition of arithmetic) they obeys very different logic. Bp obeys to the modal logic G, and Bp & p obeys to the modal logic of knowledge S4.

You cannot define "Bp & p" in arithmetic, by a general arithmetical predicate, and this makes the machine first person "I" quite non analytical, as no third person description can ever be used for it, from the machine perspective.

This explains why the mind-body problem befuddled the machines until they realized their own universality and the incompleteness which follows. That is quickly the case for universal machine believing in the induction axiom, (having them as pre-wired axioms) which provides them very deep cognitive abilities.

So you can see Bp as analytical, having third person description, like in arithmetic, and you can see Bp & p as synthetic, as it cannot be defined in term of the ontological element of the theory, nor any third person construction made on them.

Bruno



I have often talked about Bp and Bp & p, with Bp having the arithmetical provability meaning (G鰀el 1931).
G above is the logic of G鰀el's beweisbar predicate. For example the second incompleteness theorem is given by Dt -> ~BDt, or <>t -> ~[]<>t, or consistent('t') -> NOT PROVABLE (CONSISTENT 't')), with for example t = "0=0", et 't' = G鰀el number of "0=0".

S4Grz above is the corresponding logic of the first person associated to the machine, given by beweisbar('p') & p, following Theatetus, and then Boolos, Goldblatt, Artemov. I have provided many explanations on this list, including an introduction to modal logic and the Kripke semantics, but you can also open some book in logic to help yourself.

G and S4Grz are the two machineries illustrating (and formalizing completely at the propositional modal) two important arithmetical hypostases discovered by the UM when looking inward. G is the logic of third person self-reference and S4Grz is the logic of the first person self-reference.

There are six other hypostases, or machine's points of view, three of them playing a role in the "creation of the collective persistent matter hallucination. Comp makes obligatory that persistence, and it can be tested, and it can be argued that the presence of p -> []<>p as a theorem in SGrz1 and Z1* and X1* confirms it in great part. Interactions can be defined in a manner similar to Girard, and then tested on those "material hypostases". I think that this is explained in the second part of the sane04 paper.
The "1" added to the system refers to the fact that we eventually limit the arithmetical translation of the sentence letters (p, q, r, ...) to the sigma_1 sentences, which "models" the UD in arithmetic.

In particular Richard Ruquist's theory that fundamental physics is given by string theory becomes testable with respect to comp, as UDA shows that the physics is entirely retrievable from the S4Grz1, Z1* and/or X1*, and their first order modal extension. 

It is not as difficult as most paper your refer to, and it is only one paper, and you got the chance to ask any question to the author :)

You recently allude to a disagreement between us, but I (meta)disagree with such an idea: I use the scientific method, which means that you cannot disagree with me without showing a precise flaw at some step in the reasoning.

You seem to follow the seven first steps, so that in particular you grasp apparently that COMP + ROBUST-UNIVERSE entails the reversal physics/arithmetic, and the explanation why qualia and quanta separate. Are you sure you got this? Step 8 just eliminates the "ROBUST-UNIVERSE" assumption in step 7.

Then AUDA translates everything in UDA in terms of numbers and sequences of numbers, making the "body problem" into a problem of arithmetic. It is literally an infinite interview with the universal machine, made finite thanks to the modal logic above, and thanks to the Solovay arithmetical completeness theorem.

You cannot both claim that there is a flaw, and at the same time invoke your dyslexia to justify you don't do the technical work to present it. 

Bruno

--
You received this message because you are subscribed to the Google Groups "Everything List" group.
To post to this group, send email to everyth...@googlegroups.com.
To unsubscribe from this group, send email to everything-li...@googlegroups.com.
For more options, visit this group at http://groups.google.com/group/everything-list?hl=en.

Bruno Marchal

unread,
Aug 25, 2012, 10:10:59 AM8/25/12
to everyth...@googlegroups.com
On 24 Aug 2012, at 14:31, Stephen P. King wrote:

Dear Roger,

    I only see one glaring gap in your explanation here: the chain of non-interaction leads all the way up to the supremum where God is essentially and effectively (not)interacting with itself. Is this not the very definition of Solipsism? How is the problem of solipsism not even infinitely more acute for God? God has no peers, so it naturally implies that the ordinary problem of solipsism - what does one human solipsist say to another? - is a mute point, but somewhere and somehow the appearance of plurality of entities must appear in order for us to explain appearences. This is the very same question that I keep asking Bruno and he seems to not understand the question: How does a plurality of minds emerge from the One such that they have an appearance of interactions without falling into the morass of allowing for everythign and thus, ultimately, explaining nothing?

And this is what I explain with all details since years on this list, refering to peer reviewed papers, using standard terms of the theories in use. 
But either you philosophize on it without addressing what I say, or you justify by contingencies why you don't address it. 

Your question has an easy part, and a difficult part. 

- The easy part is the explanation of why interactions exist. This is easy, because all theories of interactions, and their models,  are emulated by arithmetic, like with the example of the simulation of the galaxy: it occurs in the UD.

- The difficult part is that such theories admits a continuum of consistent extension, including those which will lead to aberrant interactions, and we have to justify why they seem rare (relatively rare) in our extension, and that is the measure problem, which we cannot avoid with comp.

Then comp explains "easily" the quanta and qualia separation, has lived by each machine.

Bruno





--
You received this message because you are subscribed to the Google Groups "Everything List" group.
To post to this group, send email to everyth...@googlegroups.com.
To unsubscribe from this group, send email to everything-li...@googlegroups.com.

For more options, visit this group at http://groups.google.com/group/everything-list?hl=en.
Reply all
Reply to author
Forward
0 new messages